Location via proxy:   [ UP ]  
[Report a bug]   [Manage cookies]                

Highschool Mathemat 01 Univ

Download as pdf or txt
Download as pdf or txt
You are on page 1of 460

siilii;:

iliiiiil

iiii

i
iijiii

iiiiiif

lii'

^iiiililiiiiii
E, R.A FlY
OF THE
U N I VERS ITY
or ILLINOIS

v.l

_jjg,.^k*»t«r-
Return this book on or before the
Latest Date stamped below.
EDUCATION. PHIIOSOPHY,
PSYCHOLOGY LIBRARY
University of Illinois Library

""^ '
I oTi^bb
rBo4

JUN Q ''QQ.A X
•p^^v'^'

m T. -1964

LI61— H41
Digitized by the Internet Archive
in 2011 with funding from
University of Illinois Urbana-Champaign

http://www.archive.org/details/highschoolmathemat01univ
HIGH SCHOOL MATHEMATICS

Unit 1.

THE ARITHMETIC OF THE REAL NUMBERS

UNIVERSITY OF ILLINOIS COMMITTEE ON SCHOOL MATHEMATICS

^ MAX BEBERAAAN, Direefor

^ HERBERT E. VAUGHAN, Editor

'O

^
II

^^ UNIVERSITY OF ILLINOIS PRESS • URBANA, 1959


-0

^>
C 1959 by lh« Board of Truiteei of the Univeriity of Illinois.
AAonufoclured in the United Slates of America.
f

Instructions for integrating the green pages with the white pages

The green sheets of theCOMMENTARY occur in blocks [each consist-


ing of one or naore sheets] each of which contains discussions referring
to a single page or a group of consecutive pages of the text (white
sheets). The pages of the text to which a block of green sheets refers
are listed in brackets at the foot of each sheet. Thus, 'TC[l-A, B, C]*
indicates that this sheet refers to pages 1 - A, 1-B, and 1-C. And,
'TC[l-J]a', 'TC[l-J]b', and 'TC[l-J]c' indicate members of a block
of green sheets all of which refer to page 1-J. Each block should be
inserted among the pages of the text so that the printed side of its
first sheet faces the first text page to which it refers. For example,
the sheet labeled 'TC[l-A, B, C]' should face page 1 - A of the text.
And, the sheet labeled 'TC[l-J]a' should face page 1-J of the text, and
be backed up by the sheets labeled 'TC[l-j]b' and 'TC[l-J]c'. The
final result should be that, on turning to a page of the text, one has
opposite it the first of the green pages [if any] which refer to it. And,
if there are several such pages, one obtains a view of the second by
turning the first, etc.

An exception to these instructions is that the introductory green pages


of each unit are to follow the title page of that unit.

For your convenience, we list the first several pages of the completely
integrated Teachers' Edition.

Title page, blank], [l/l, blank], [l/2, blank], [l/3, blank],


'l/A, blanlc], [1/5, blanlc], [i, ii], [iii, iv], [blank, l/6],
l-A, 1-B], [1-C, 1-D], [1/7, blank], [l-E, 1-F], [1-G, 1-H],
l-I, 1-J], [1/8, blank], [l/9, blank], [l/lO, blank], [blank, l/l 2],
blank, l/l 1], [1-K, l-L], [l/l 3, blank], [l/l4, blank],
blank, l/i7], [blank, 1/16], [blank, l/l5], [1-M, 1-N],
1/18, blank], [blank, l/20], [blank, l/l9], [l-O, blank],
"blank, l/22], [blank, l/21],[l-l, 1-2], [l/23, blank],
;i-3, 1-4], [1/24, blank], [blank, l/25], ...

Note The numerals 'l/l\ 'l/2', ... are useful for assembling the
:

green pages in order for those who do not want to integrate


them with the white pages.
1/1

TEACHERS COMMENTARY

Introduction
The text materials for the UICSM program are produced by high
school teachers and mathematicians on the staff at the University of
Illinois. Since 1951, we have been debating the issues:
What mathematical ideas should be taught in high school?
What are the nnost effective ways to teach these ideas?
We feel that teachers using the materials ought to "take part" in
these discussions. We hope this can be done by means of the
TEACHERS COMMENTARY.
The COMMENTARY brings you experiences and suggestions of
teachers who have worked with us during the past six years. We
have used their daily reports not only in revising the student's
materials but in adding good teaching suggestions to the previous
edition of the COMMENTARY.
We continue to welcome all kinds of suggestions from you- -your
ideas, students' reactions, sanriples of their work, complaints and
praise from parents, and even reports of the times when you or the
students felt that "the Illinois people were just crazy".

Our operating principles


We believe that students should be given an opportunity to dis-
cover a great deal of the nnathematics which they are expected to
learn. Mathennatical ideas which a student discovers make sense to
him. Discovery will lead him to feel that mathematics is a human
and growing subject. Contrast the attitude of a student \vho has dis-
covered for hinnself rules for operating with real numbers ["signed"
numbers] with the attitude of one who has been given these rules by
the teacher or the textbook. The first student is eager to try these
rules; he needs no justification of their value to society; the very
fact that they pernnit him to solve interesting problems more effi-
ciently is sufficient justification for their use. The second student
may wonder how the teacher or textbook knew about these rules; his
notion that mathematics is a subject which is contained in books or
in the heads of teachers is reinforced; he needs to be told over and
over again that the attainment of skill in the use of these rules is
important in his vocational plans. And, although many ninth graders
will acquiesce to this last dictum, they may approach mathematics

[Unit 1]
1/2

as if it were just another of the bits of drudgery that are involved in


the process of growing up. We believe firmly that the learning of
mathematics should be a delightful experience for youngsters, and
that this delight is the reward for hard work. We also believe that
the amount of pleasure a student derives from his learning of mathe-
matics depends on the extent to which he finds opportunity for
creative activity.

So, one of the ground rules you will have to establish early in
the teaching of UICSM courses is that the student is expected to con-
tribute ideas, principles, and rules. It may be necessary to over-
come some initial prejudices which students have toward mathematics.
At first, they may insist that they be given a formula or a rule for
doing a problem. Since the text does not give it to them, and since
you will not give it to them, they may turn to their parents for such
help. Hence, it is important that parents understand the kind of
attitude we are trying to create. It has been our experience that as
soon as a student encounters success in formulating his own rules
and short cuts, the initial prejudice is removed. Students who were
demanding rules at the beginning of the year turn into students who
literally beg the teacher to give them "more time to work it out".

Our notion that the learning of mathematics ought to be an enjoy-


able experience has been interpreted by some people to mean that the
mathematics classroom is a place where parties are held, where
everybody does as he pleases, and where no one is really expected
to learn anything! Needless to say, this interpretation is made by
people who have had no experience in teaching UICSM courses. They
may have forgotten that there is such a thing as joy in intellectual
activity, and that to smile in a mathematics classroom does not result
in destruction of character.

Our experience in teaching the "early adolescent" has shown us


that he is interested in many things which the adult considers frivo-
lous. So-called "real life" applications for the adult may not be real
life applications for the young learner. Also, many of the things in
which the young learner is interested are hardly in accord with the
adult's view of real life. Since we believe that interest is a necessary
condition for learning, we have tried to set the development of mathe-
matical ideas in situations which are inherently interesting t£ young
people. Thus, one of our standard devices when approaching a new
idea is to create a fanciful situation which embodies or illustrates it.
Even though a student is aware of the fact that these situations are
fanciful, he can easily imagine thenn. And, the very fact that they
are fanciful appeals to the youngster's interest in the make-believe
and the fantastic.

[Unit 1]
[i]

1/3

Another common misconception concerning delight in leai-ning is


that any course which purports to provide youngsters with intellectual
"fun" must be a course in which the content has been watered down.
Some people believe that mathematics can be made interesting to
youngsters only by diluting it. But, as a matter of fact, diluted mathe-
matics cannot be interesting at all. Those of you who have had
experience with watered down courses know that such courses are
time -wasters for both the talented student and the less able one. We
have tried to provide in our material an intellectual challenge for a
wide range of ability. Nevertheless, the mathematical content of our
courses is sound mathematics. The fact that it is sound naathematics
does not mean that it is "rigorous", dull, and lacking in appeal to
intuition and imagination. There is a time in the student's mathemati-
cal career when he needs to bring rigor into the pursuit of mathematics,
and the mathematics he learns prior to this time must be so orgcuiized
that when rigor is finally introduced, it will not be necessary for him
to throw out anything he has learned at an earlier level. One of the
ways in which the nnathematician contributes heavily to these courses
is in the establishment of this sound mathematical background.

Now, in developing a course in which student-discovery is the all


important element, it is necessary to adopt certain practices which
will permit the students to make discoveries. One of these practices,
and this is an obvious one, is that the textbook not give the game away.
It is all too common to find textbooks in which a series of discovery
exercises is given on one page and the thing to be discovered is stated
in boldface type on the next page. We realize that textbooks may have
to cater to the needs of teachers who do not believe in the discovery-
method of teaching as well as to the needs of those teachers who do.
However, the UICSM courses are being developed only for teachers
of the latter persuasion. Therefore, you will not find rules displayed
on pages immediately following discovery exercises. We believe that
it is part of the teacher's job to determine when the students have
discovered correct generalizations. And we have included devices
in the text which will permit the teacher to make these determinations
without compelling the student to give precise verbalization. In fact,
we eschew verbalization on the part of the student at the time of dis-
covery, especially during the early part of the course, since we believe
that a painfully-arrived-at verbalization may impede the utilization of
the generalization. [See the articles by Gertrude Hendrix entitled "A
New Clue to Transfer of Training" in the December 1947 issue of
The Elementary School Journal, and " Prerequisite to Meaning" in
the November 1950 issue of The Mathematics Teacher. See also

FUnit n
1/4

"The Transfer Value of Given and Individually Derived Principles"


by G. M. Haslerud and Shirley Meyers in the December 1958 issue
of The Journal of Educational Psychology ] Later in the course, when
.

students have developed considerable linguistic facility, it will be


found that they can give precise statenaents of discoveries, including
those which they made earlier in the course.

Another practice which facilitates discovery is that of using pre-


cise lajiguage in exposition. Whenever we talk about sonaething
directly [rather them by giving examples], we want our exposition to
be precise. [Unless we say precisely what an equation is, it is fool-
hardy to expect students to discover procedures for solving equations. ]
But, note carefully, this precision in exposition is something we
expect of the textbook and the teacher, rather than of the learner.
Precise communication is a characteristic of a good textbook and a
good teacher; correct action is a characteristic of the good learner.
It is important that you observe this distinction as you go through the
course. You can begin to expect more precision from the student
only after he has been exposed to careful language for a considerable
period of time. [For a recent discussion on precise language in the
teaching of mathennatics, see Chapter 8 of the 24th Yearbook of the
National Council of Teachers of Mathematics, The Growth of Mathe -
matical Ideas (Washington, D. C. : The Council, 1959).]

COMMENTARY
Style and format of the
The COMMENTARY pages have been designed so that they can be
integrated with the white pages of the student -edition. Each green
page contains a discussion relevant to one or more white pages. For
example, the green page TC[1-A, B, C] contains a discussion concern-
ing the content of the white pages 1-A, 1-B, and 1-C. Since the first
of these white pages is a righthand page, the green page TC[1-A, B, C]
has holes punched along its right margin so that it will face the white
page 1-A. On the other hand, green page TC[1-D, E, F] has holes
punched along its left margin so that it will face white page 1-D which
is a lefthand page.

You nnay find it helpful in getting ready to teach a unit to read all
of the white pages fairly rapidly, and then read the white pages together
with the accompanying green pages. The white pages will give you
some notion of what is expected of the student. The green pages will
describe these expectations more carefully, cind will contain mathe-
matical background for you as well as pedagogical suggestions for
accomplishing the goals set for the student. Naturally, you will want
to read smaller sections of the green and white pages each day as you
prepare for your classes.

[Unit 1]
Acknowledgments

The UICSM Project Staff thanks the teachers in cooperating


schools for their detailed written reports, for the suggestions and
criticisms they gave us in conversations and in training conferences,
and for the ideas that came to us as we worked with the students in
their classes during our visits.

Special recognition is due Miss Eleanor McCoy, Associate


Teacher Coordinator in the Project, for the constructive criticisms
she made of the text materials for this edition (and also for those
made in the early years of the program when she taught at Pekin
Community High School), for the immense amount of help she has
given in the preparation of this and the last editions in the tedious
jobs of proofreading, page -fitting, and indexing, and for her even
more important assistance in making tests, constructing exercises,
and writing developments for the COMMENTARY and the text.

We also owe much to a succession of highly proficient typists:


Mrs. Naomi Konecky, Mrs. Helen M. Sakamoto, Mrs. E. June
Harris, and Mrs. A. Ruth Fifer, and to the staff artists: Mr. Roy
Nagao cind Mr. Roy Yamachi.

We
take this occasion to list former members of the Project
Staff who contributed
significantly to earlier versions of the text:
Professors Daniel S. Babb, Gerhard P. Hochschild, Bruce E.
Meserve, Eugene D. Nichols, David A. Page, Robert E. Pingry,
and Charles R. Stegmeir,

And, finally, we thank the Carnegie Corporation of New York


for its generous support.

Max Beberman, Director


Herbert E. Vaughan, Editor
University of Illinois Committee
on School Mathematics.

May 1, 1959
Urbana, Illinois

[Unit 1]
[i]

TABLE OF CONTENTS

Introduction
Arithmetic by mail [1"A]
Things and the nances of things [l"E]
Numbers and numerals [1"K]

1.01 Distance and direction [1"1]


Numerals for real numbers [1"2]
Using real numbers to measure trips [l"3]
Using real numbers to locate points with respect
to a given point [1"4]
Positive and negative real numbers [1"7]

1.02 Addition of real numbers [1"8]


Using real numbers to measure changes [1-10]
Trips of distance 0--the real number [1-13]
Nonnegative and nonpositive real numbers [1-15]

1.03 Multiplication of real niimbers [1-17]


A pump, a tank, and a movie [1-17]
Exploration Exercises --tables and operations [1-24]

1.04 Nvimbers of arithmetic and real numbers [1-29]


Shorter names for positive numbers [1-31]
Interpreting ambiguous numerals and words [1-31]

1.05 Punctuating numerical expressions [1-33]


Using parentheses, brackets, and braces [1-35]
Conventions for omitting grouping symbols [1-37]

1.06 Principles for the numbers of arithmetic [1-44]


The commutative principle for multiplication [1-45]
The commutative principle for addition [1-45]
The associative principle for addition [1-46]
The associative principle for multiplication [1-46]
Another principle [1-50]
The distributive principle for multiplication over
addition [1-51]
[ii] (CONTENTS]

More principles [1-53]


The principle for adding [1-53]
The principle for multiplying by 1 [l"53]

The principle for multiplying by [1-55]


Using the principles for short cuts [1-55]

1.07 Principles for the real numbers [1"59]


Investigating the principles for the real numbers [1-60]
Using the principles for short cuts [1-62]
Exploration Exercises --
Subtracting undoes what adding did [1-63]
Multiplying by the reciprocal undoes what
multiplying did [1-64]
Reciprocals [1-65]

1.08 Inverse operations [1-66]


Operations [1-67]
Finding out what an operation is --pairs of numbers [1-67]
Finding out what the inverse of an operation is [1-68]
Subtracting a number is the inverse of adding that
number [1-69]
Dividing by a nonzero number is the inverse of
multiplying by that number [1"69]
Exploration Exercises --
Adding the opposite undoes what adding did [1-72]
Adding the opposite of a real number is the inverse
of adding that number [1-73]
Opposites [1-73]

1.09 Subtraction of real numbers [1-75]


Subtracting a real number is the inverse of adding
that number [l"75]
Subtracting is adding the opposite [1"77]
The principle for subtraction [1"79]
Chajiging subtraction problems to addition-of-
the -opposite problems [1"79]
[CONTENTS] [iii]

1. 10 Opposites ;i-80]
The principle of opposites 1-80]
The operation oppositing 1-81]
Using a minus sign for oppositing 1-81]
Using the principle of opposites :i-83]
New names for negative numbers ;i-86]
Three uses of the minus sign ;i-87]
More names numbers
for positive ;i-88]
The operation "sameing" ;i-88]
Three uses of the plus sign ;i-88]

1.11 Division of real numbers ;i-92]


Dividing by a nonzero real number is the inverse of
multiplying by that number 1-92]
Ways of naming a quotient :i-94]
Numerator and denominator of a fraction 1-94]
Multiplying by has no inverse 1-95]

1.12 Comparing numbers :i-95]


Comparing numbers of arithmetic :i-95]

Using the symbols ' > '


and ' < '
1-96]
Comparing real numbers :i-97]

Testing by adding a positive number ;i-97]

1.13 The number line 1-99]


" Lining up" the real numbers in order ;i-99]
Using the symbols '
ff
'
and '
^^
'
1-100]
Using the symbols '
> '
and '
< '
1-101]
Uniform scale 1-102]
Absolute value operation 1-103]
Distance between real numbers 1-103]
Absolute value of a read number 1-104]
Using absolute value bars 1-106]
Does absolute valuing have an inverse ? 1-107]
Operations and their inverses 1-108]
Ambiguous numerals 1-110]
[iv] [CONTENTS]

Miscellaneous Exercises 1-111]

A. Operations with real numbers 1-111]

B. Absolute value and comparisons 1-111]

C. Trips along the number line 1-112]


D. Using real numbers to measure changes 1-113]
E. Guess -the -number problems 1-115]
F. Sorting numerals 1-115]
G. Pairs of numbers and operations 1-116]
H. Problems with averages 1-117]
I. Line chart problem 1-118]
J. Bar chart problem 1-119]
K. Using single quotes 1-120]

Test 1-121]

Supplementary Exercises 1-126]

A. Using single quotes 1-126]


B. Adding real numbers 1-126]
C. Using real numbers to measure changes 1-128]
D. Adding real numbers 1-132]
E. Multiplying real numbers 1-133]
F. Simplifying abbreviated expressions 1-133]
G. Recognizing instances of principles 1-134]
H. Using principles for short cuts 1-135]
I. Subtracting real numbers 1-136]
J. Adding and subtracting 1-137]
K. Adding and subtracting 1-137]
L. Simplifying 1-139]
M. Dividing real numbers 1-140]
N. Comparing real numbers 1-140]
O. Comparing real numbers 1-141]
[1-A]

\
[iv]

Miscellan'

Pages 1-A through 1-E are intended as an introduction to an extremely


important pedagogical idea which will conne up tiiTie and again in teach-
ing UICSM materials. The idea is developed further in pages 1-E
through 1 -O. But it may be possible to get at the heart of the matter
through the humorous interchauige between Al and his "tutor" Stan.

The Project Staff has expressed its point of view concerning the im-
portamce of distinguishing between symbols and their referents in the
article " Words, 'Words', "Words" " which appeared in the March
1950 issue of The Mathematics Teacher .The 1958 Inglis Lecture,
An Emerging Program of Secondary School Mathematics contains a
,

more detailed rationale for our making this distinction. [The Inglis
Lecture series is published by the Harvard University Press.] An
article which treats this topic from the point of viev/ of the elementary
school is Frank Wolf's " "1" and " 1" is "11" " which appeared in
the April 1958 issue of The Arithmetic Teacher .The monograph
Godel's Proof by Ernest Nagel and James R. Newman (Nev/ York:
New York University Press, 1958) contains several interesting sections
on distinguishing between symbols and their referents [In particular,
see pages 28-31 and footnote 24 on pages 82 ff. ].

We suggest that you select a student who has sonae flair for dramatics
to read aloud the exchamge of correspondence between Al and Stan.
[You might find it necessary to read the letters aloud yourself. ] Un-
doubtedly, you will find students who disagree with Al's justifications
and with Stan's justifications. This disagreement should be encouraged
even if it comes close to splitting hairs.

'I*

Students ought to see that Al is perfectly consistent in the way he has


responded to Stain's questions. Let the students tell why Al answered
as he did for each item before they read his response to Stan.

Be graphic describing why '3' goes into '8' twice. You might want
in
to bring a paper 8' into class and actually cut it into two
'
3's
'
Let
!

the students devise other questions and answer them as Al would, even
questions outside of mathematics such as:

Is MARY bigger than Mary?

TC[1-A, B, C]
[Introduction] [1"A]

Arithmetic by mail --Stan Brown had a pen pal, Al Moore, who lived
.

in Alaska. Stan and Al corresponded quite frequently. Stan liked to


receive letters from Al because he wrote about interesting things like
hunting and fishing and prospecting for gold. Al enjoyed hearing about
the things Stan did, especially about school, for Al had had very little

opportunity to attend school. One day, Al wrote to ask if Stan would


mind teaching him some arithmetic. Stan agreed but decided he needed
to know how much Al already knew. So, in his next letter to Al he

included a simple test, and asked Al to write in the answers and to


return the test to him. Al sent the test back immediately; he said it

was very easy and asked Stan to send some harder questions next time.

Turn the page to see what Al's test looked like when he returned it.
M.gi [Introduction]

1. Take 2 away from 21

2. What is half of 3 ?

3. Add 5 to 7. . . . A 7. . . .

X rur
4. Does 2 42" equal 9?

5. Which is larger, . 000065 or . 25 ? .:??<??A?'.

6. How many times does 3 go into 8 ? . . Y^'^^. . .

7. How many times does 9 go into 99? . .P^yf^. . .

8. V/hich is larger, 3 or 23? 23

9. What is a number smaller than 4 ?

10. What is a number larger than 4 ?

Stan was flabbergasted when he looked at Al's answers. Was


this a joke ? But Al had seemed so serious about wanting to learn
arithmetic. Stan decided that Al needed a lot of help. He would
start by telling Al about the errors he had made.

Dear Al,

You sure have some funny ideas about numbers.


But it won't take long to straighten you out (I hope).
I've enclosed your test with this letter so that
you could follow my explanation. Look at the first
question. I can see how you thought that the answer
was 1 because if you do take 2 away from 21 you are
left with 1. But when you take 2 away from 21, you
get 19. See? Take the second question. Your answer
isn't even a number. What you wrote is half of 3 but

half of 3 isn't that, it's Ij. In the third question, you


put 6 and 7 together, but adding numbers isn't putting
them together. It's, er... well, I'll explain that in
[Introduction] [1-C]

your next lesson. In the fourth question, I can see


that you thought that Z X 4 r- was not the same as 9,
1
but 2 X 4— IS equal to 9, so your answer is wrong.
Now, in question five, it's true that . 000065 is

bigger than . 25, but really . 25 is larger than . 000065.


You see, . 25 is a bigger number than . 000065 even
though . 25 looks smaller than . 000065.
I guess I can give you credit for question six,

although you really should have said that 3 goes into


8 twice with 2 left over. I guess you just made a
careless mistake in dividing in question seven, be-
cause 9 goes into 99 eleven times. Question eight
you did correctly, so I guess you know how to count.

You nnade the san">e kind of mistake in question

nine as you did in the very last question. Even


though what you wrote for question nine is smaller
than 4, I asked for a number smaller than 4 and the
one you picked is the same size as 4. Same idea for
question ten.
I hope that my explanations of what you did wrong
are helpful to you. Let me know when you want your
next lesson. Make it soon because we have lots of work
to do.

Your pal,

Stan

Al was a little disappointed with Stan's letter. He wrote the


following in return.

Dear Stan,
I sure appreciate what you are trying to do
for me, but I don't think that you can help me at

all. Are you sure you understand this stuff?

Sure I got questions 6 and 8 right. I did


them just as I did the others. Anyone can see
, , [Introduction]

neatly, too,
that 3 goes into 8 twice, and pretty
without any 2 left over, either. You put 3 into
8 the regular way and then you turn another 3

around and put it in on the other side of the 8.

In question 8, you can just see that 23 is

larger than 3 because 23 already has a 3 in it

and a 2 added on in front. You don't need to

know how to count to tell that!

I really laughed at what you said about


question 7. If there's one thing I do know it's

how to count, and eleven 9's make 99999999999


and not 99. You said that .000065 is bigger

than .25. I knew that because I even checked


with a ruler. Then you cross yourself up and
say that .000065 is really smaller. And in
question 4 you don't even need a ruler to tell
that 2 X 4y is different from 9.

There's no use in trying to learn arithmetic,

I guess. I think I'll stick to hunting.

Your friend,

Al

EXERCISES
A. Do you think Stan did a good job explaining Al's errors? What
seemed to be Stan's difficulty in doing so? Can you defend
what Al did?

B. In certain print shops the printing press is prepared for opera-


tion by picking individual type pieces out of boxes and placing
the pieces into the bed of the press. Such a print shop has type
of various styles and sizes. Here are two examples of remarks
you nnight hear a printer make to his helper:
1/7

By careful questioning, try to elicit from the students the notion


that what Stan needs is a writing system which will permit him to
indicate when he is talking about things and when he is talking
about the names of things. The students should notice that Stan
was not attennpting to teach arithnnetic to Al in his letter. He is
just trying to point out to Al the nature of Al's mistakes. But Al
could not understand this just because Stan kept confusing numbers
with their names.

In connection with Part B, here are further amusing statements


which might be overheard in a print shop:

Find a two bigger than this eight.

Bring me type.

0. 00001 is certainly bigger than 14.

When you emphasize that the marks are not the numbers thennselves,
perceptive students may ask what the numbers are. They write the
name 'Mickey Mantle' and know what this name stands for because
they know this New York Yankee baseball player. On the other hand,
when they write the name '2', they cannot see or touch the number 2.
The question 'What is the number 2?' is a very profound one, and is
one which has come to be answered in a satisfactory way only recently.
The number 2, just like justice, is an abstraction; it is not something
which can be pointed at.

An excellent book which deals with the question of what numbers are
isHugh A. Thurston's The Number -Systena (London: Blackie, 1956).
Another is Irving Adler's The New Mathematics (New York: John
Day, 1958).

TCfl-D, E, Fl
[Introduction] [1"E]

Bill, this seven isn't big enough; get me a bigger


seven.

I asked you to bring me two threes and you brought


me three twos. I don't care what you learned
in school, three twos are not the same as two
threes.

Make up more examples of confusing statements you might


hear in this print shop.

Things and the names of things . --It is easy to see what Al had in
mind when he took Stan's test. Al was confusing numbers with the
marks that were written on the paper. And even though Stan may
have realized this, he certainly didn't get the idea across to Al. It

might have helped Al if Stan started by saying something like this :

Look, Al, if I asked you to tell me something


about Alaska, you wouldn't tell me that it started
and ended with the same letter, would you? You
might tell me how big it was, or how many people
lived there, or what its capital city was. So, when
I ask you a question about numbers [for example,
to take 2 away froin 21], I don't expect you to tell
me about the marks on the paper. Instead, I expect
you to do something with the numbers those marks
stand for. You see, those marks aren't numbers.
They just stand for numbers. They're really names
of numbers, just as the word:

Alaska

is the name of the place in which you live.

Whenever a person writes about numbers, he puts marks on paper;


if he talks about numbers, he makes certain sounds. But the marks
on paper and the sounds are not the numbers themselves. The marks
and the sounds are nctmes of numbers. In order to write about a
thing, you must write a name of the thing.
[i-p] [Introduction]

Ineveryday conversation, one hardly ever confuses a thing


with a name of the thing, because in so many cases you can point
to the thing and point to its name, and you can see that they are

different. But, there are some things which we talk about but

which we can't point to. Take justice, for example. You would
have a hard time trying to point to something, and saying that
that thing is justice. Also, if you were having a serious discus-
sion about justice, you would get pretty annoyed with someone who
claimed he could show you what justice is and did it by handing you
a piece of paper like this:

/ J/^c^iZicjC.

Most people would agree that justice isn't something you can touch
or see or hear. Numbers are like that, also. Someone can show
you 2 apples or 2 toys or a hat and coat, but none of these pairs of
things is the number 2 itself. But, just as people can think about
justice even though they can't see or touch it, they can also think
about the number 2. And, in talking about and working with the
ideas of justice and 2, they use marks such as :

but they don't [at least they shouldn't] think that the marks them-
selves are justice and the number 2.

Now, in reading a mathematics textbook or in listening to


someone who is trying to teach you mathematics, it is very im-
portant that you be able to tell when the book or person is talking
about numbers and when the book or person is talking about the
marks which are names of numbers. Al had trouble in making
sense out of Stan's explanations because Al couldn't tell when
[Introduction] [1-G]

Stan was talking about nnarks on paper and when he was talking about
the numbers which the marks named. And you wouldn't have been
able to tell this from Stan's letter either if you didn't already know a
lot about arithmetic. Al's trouble with Stan's explanation was that

(1) in writing out the test, Stan had used the mark:

and the mark:


21

asnames for numbers [so, his first question was about the
number two and the number twenty-one], but

(2) Al thought that Stan's first question was about the marks
themselves, and

(3) in trying to explain Al's errors to him, Stan began by using


the marks as names for themselves ["if you do take 2 away
from 21 you are left with 1 "], and then used them again as
names for numbers ["when you take 2 away from 21 you get
19"].

No wonder Al was confused Iii order to write about


! the marks Stan
needed names for them He might have written:
.

Al, let's user

Tom
as a name for the mark:
2.

and use:
Dick
and:
Harry
as names for the mark:
21
and the mark:
1.

Now, if you do take Tom away from Dick, you


are left with Harry. But when you take 2 away
from 21, you get 19.
ri-j-j] [Introduction]

Or, he might have told Al that he would write a name for a mark by
drawing a loop around it. When Al saw a loop, he would know that
it, together with the mark inside it, was a word, and that this word
was a name for the mark. So, Stan might have written:

I can see how you thought that the answer was


(l) because if you do take fzj away from (Zl) ,

you are left with (T) . But, when you take 2

away from 21, you get 19.

Of course, this statement wouldn't teach Al how to subtract, but it

would at least tell him that Stan realized that Al was confusing num-
bers with names of numbers.

Can you think of other devices you could use to write names for

marks ?

In this book, the device we shall use most frequently to write

names for marks is to enclose marks in single quotes ['. . . ']. When
you see a mark [such as a word or a sentence, for exannple] with
single quotes around it, you are seeing a name for the nnark which is

written between the single quotes.


So, the sentence:

' 3* is a nanae for a number


is about the mark :

which is named in the sentence. On the other hand, the sentence:

3 is aji odd number

is about the number which the same mark names.


To decide whether or not to put single quotes around a mark, ask
yourself:

Am I talking about the mark, itself?

or

Am I talking about the thing to which the mark refers ?


[Introduction] [1~I]

When you are talking with a person, you can sonnetinnes get along
without using names for the things you are talking about. You do this
by pointing at a thing and saying 'this'. For instance, Stan could have
made his explanation to Al, if they had been together, by pointing, first
at the '2' in the '21', then at the '21', and then at the '1', aind saying:

If you take this away from this, you are left with this.

Sometimes in writing we shall avoid using a name for a mark by a kind


of pointing at the mark. [In fact, we have already done so. ]

We can point at a mark by displaying it on a separate line, and


putting a colon at the end of the preceding line. Think of the colon as
the writer's finger pointing at the mark which follows it.

Sometimes we want to point at a word just to call special attention

to it, either because it's especially important or because it's a new


word.

(1) 'John' is a word but John


, is a boy .

(2) Names of numbers are called numerals.

(3) Notice that in '[8 X(3 + 2)] X 5' we use both parentheses and
brackets.

In sentence (1), the words 'word* and 'boy' are underlined nnerely
because we wish to emphasize a contrast. In sentence (2), the word
'numeral' is anew word, and we want to call your attention to it.

Actually, the sentence is about this word and, so, should be written:

Names of numbers are called 'numerals'.

But, because the word is underlined in (2), and follows 'are called',
our carelessness in onnitting the single quotes is not likely to cause
confusion. In sentence (3), the word 'brackets' is xinderlined because
it is a word which has not been used before. But [unlike the word
'numeral' in (2)] the sentence isn't about this word, so it would be
incorrect to enclose it in single quotes.
fl-j] [Introduction]

Read the following paragraphs carefully and note the use of


various signals to tell you when names and marks are being dis-
cussed.

This morning a girl named Ruth received four

letters. Four friends had written to Ruth, and each


of them wrote 'Ruth' on the envelope. Each letter

started with:

Dear Ruth,

Notice that Ruth has four letters and that 'Ruth' has
four letters.
Ruth sometinnes wished her name were longer
so that she could give herself a short nickname. Her
friend Margaret was often called Peg . 'Peg' is short
even though Peg is tall. Ruth often wondered how you
get 'Peg' out of 'Margaret'.

EXERCISES

A, Some of the following sentences make sense and some do not.


In the case of each sentence which does not, put single quotes
around sonne of the words in the sentence so that the resulting

sentence does make sense.


1. Bill has a dog.

2. Bill found a dog in a book.

3. I have trouble with my pen when I nnake a 3.

4. Mary is a part of Maryland.

5. He erased the 5 and put a 4 in its place.

6. John has four letters.

7. Ada is taller than Penelope but Ada is shorter than Penelope.

8. Mr. is an abbreviation for Mister.

9. 6 is number and 6 is a place holder.


a Similarly, is a
number and is a place holder.
1/8

You may want to agree upon some device with your class for pro-
nouncing single quotation nnarks. For example, we on the Project
Staff would read the second sentence in the displayed paragraph as:

Four friends had written to Ruth, and each of


them wrote s quote Ruth uns quote on the envelope.

However, the purpose of reading this paragraph is to see how it is


written [in addition to what it says]) and the purpose of reading it
aloud is to focus the student's attention on how the paragraph is
written. Ordinarily, in contrast to this, the purpose of reading
aloud is to focus attention on what the written language says .One
is then translating written language into spoken language rather
than, as here, using spoken language to describe written language.
Ifone wished to use spoken language to convey the meaning of the
second (written) sentence in the displayed paragraph, one would make
the sounds indicated by:

four friends had written to ruth and each of


them wrote the word ruth on the envelope.

[Here, the underlining indicates stress. ] This has implications (for


your vocal activities in the classroom) which are discussed on
TC[1-L].

Single quotation marks are used to write names of marks. Obviously,


they can be used in writing names of names of mg-rks, etc.

John is a boy,
'John' is a boy's name,
'
'John' ' is a name for a boy's name.

There are some excellent exercises of this type in Alfred Tarski's


Introduction to Logic (New York: Oxford University Press, 1954).
Incidentally, you will find much material in this book which will be
helpful in handling the ideas in the UICSM program.

'I*

TCn-Jla
[1-J]
Answers for Part A.

1. Bill has a dog.

2. Bill found a 'dog* in a book.

3. I have trouble with my pen when I make a ' 3'.

4. 'Mary' is a part of 'Maryland'.


5. He erased the '5' and put a '4' in its place.

6. John has four letters, [or: 'John' has four letters.]


7. Ada is taller than Penelope but 'Ada' is shorter than 'Penelope'.
8. 'Mr. '
an abbreviation for 'Mister'. [An abbreviation of a
is
mark is mark which is used in place of the first mark
a second
because it is convenient to do so. In most cases the convenience
is due to the fact that the abbreviation is shorter than the mark
which it abbreviates. But the former need not be a piece of the
latter. Example: 'won't' for 'will not'.]
9. 6 is a number aind '6' is a place holder. Similarly, is a num-
ber and '0' is a place holder.

•Exercise 9 may cause some difficulty. In fact, there may even be


students who have not heard the expression 'place holder'. But,
many students have been taught that the number is not really a
number at all but just a place holder in a place value system for
writing numerals. This is utter nonsense, and is undoubtedly a
cause of many of the so-called zero -difficulties that students en-
counter in arithmetic. There is a number and there is a symbol
'0'. The symbol '0' [sometimes called 'cipher'] is a name for the
number 0. In discussions of the decimal system of numeration the
symbols '0', '1', '2', ..., '9' are called digits None of thenn is .

a number, but each is a decimal nanne for a number. The rules for
the decimal system of naming numbers tell one how to combine digits
to obtain decimal names for numbers. This process is analogous to
that of combining letters of the alphabet to obtain nouns in the English
language. [The analogy is not too strict, because the digits are them-
selves nouns, while the letters of the alphabet are not nouns; and,
while the meaning of a decimal numeral depends on the mecinings of
the digits of which it is composed (and on the order in which they

TC[l-J]b
]

1/10

occur in the numeral), letters of the alphabet have no meanings which


contribute to the meaning of an English word built out of them. ] So,
while it makes sense to say that the digit *0' (or amy other digit) is a
place holder in a decimal numeral, the number is no more a place
holder thaji is the number 6 or the nunnber 84.75. [Incidentally,
while it makes sense to say that '0' is a place holder in the numeral
*809', this is no more helpful than is saying that *e' is a place holder
in the word 'net'. What may be helpful in explaining decimal nota-
tion is to say that in *809' the '9' holds (better: occurs in) the units
place, the '0' holds the tens place, and the *8' holds the hundreds
place.

TC[l-J]c
[1-K]
[1-J]

Rep

1/12

Also, from another teacher came a report of a student who naentioned


that his grandmother, who had been an arithmetic teacher for many
years, assured him that was not a number even though sonae people
claimed it was

Some students may not be familiar with the Roman numeral 'IIII*.
They may claim that it is not a name for 4 because the Roman numeral
they know is *IV'.

You should call your students attention to the use of single quotes
in the third line from the bottom of page I-K. Ask why they were
put there. [Answer: Because the writer wanted to refer to a num-
eral for 4, so he had to use a name of this numeral.] Throughout
the text, you will find opportunities for questions like this. By doing
so, you will help the student understand the use of the single quota-
tion marks device. However, you should not insist that he use this
device consistently in his own written work. It is not easy to become
proficient in this, eind the attainment of such proficiency is not a
purpose of the course.

TC[l-K]b
[1-K]
.

1/U

Here is a punctuated version of the paragraph in Part B.


Ialways put milk on my cereal. Otherwise, it is
too dry to eat. But, I have never put 'milk' on my
cereal. It would be much easier to put 'nrxilk' on cake
frosting than to put 'milk' on corn flakes.
As an extra-credit follow-up to Part B you may ask students to
punctuate Stan's letter to Al.

This display of numerals for 4 is the beginning of our campaign


to get students to understand that a numeral such as '3 + 1' is
not a command to add 3 and 1 but is merely a name for 4. In
fact, the synnbol '3 + 1' is a name of the sunn of 3 and 1. V/e
think that students who understand that nunnbers have many
names will not make contradictory statements like:
The sum of two real numbers is a real
number, but you can't add 2 and nTZ,
and:
The sum of two rational numbers is a

rational number, but you can't add , and -r

Students in conventional algebra courses seem to have a hard


time understanding that '2 + -v/^' is_ a name for the sum of 2
and the ^ 2 and that we do not have a shorter name for this
number.
,

'11' name 1
Similarly, t + T is a for the sum of •=• and
' ^
1 . . , ' 8
_8_
•r; a shorter name , .

for this sunn is


.

15

'I*

V/e included the numeral *4 X 0' in the list in order to get students
to look at each of these numerals and assure themselves that the
numerals did stand for 4. Be prepared for possible displays of
gross misunderstanding concerning "zero facts". For example,
one teacher reported to us that a student in his class said:

My parents told me that 4 X is 4, and that


X 4 is 0; you can tell which answer is which
by loolcing at the number that comes first.

TC[l-K]a
[Introduction] [1-K]

B. Use single quotes in punctuating the following paragraph in

order to make sense out of it.

I always put milk on my cereal. Otherwise,


it is too dry to eat. But, I have never put nailk
on my cereal. It would be much easier to put
milk on cake frosting than to put milk on corn
flakes.

[More exercises are in Part A, Supplementary Exercises. ]

Numbers and numerals --We have said that numbers are things
.

that we can think about, talk about, and write about. In order to

talk and write about numbers, we use nanmes for them. Names
of numbers are called numerals or, sometimes, numerical ex -
pressions . Here are a few of the numerals which are names of

the number 4. [There is one numeral among these which doesn't


name 4 ; find it !
]

the product of 12 and ^


1 + 3

A number has nnany, many numerals. Some of a number's


nunaerals are simpler looking than others. For example, '4' is

probably the simplest looking of all of 4's numerals. Certainly,


it is simpler looking than the numeral '628.424 -r 157. 106'. Yet,
n_Li [Introduction]

both of the numerals :

4
and:
628.424 f 157. 106

are names of the same number, the number 4.

A short way of saying that each of two nunnerals names the Scime

number is to write an equality sign between the nunnerals. Thus,


when we write the sentence '5 + 2 = 6 + 1', we are saying that 5 + 2

is same number as 6 +
the Our sentence is true because '5 + 2*
1 .

and '6+1' are numerals for the same number. If we write the sen-
tence '9 + 3 = 4+7', our sentence is false because '9 + 3' and '4 + 7'
are numerals for different numbers. A short way of saying that
'9 + 3' and '4 + 7' are numerals for different nunnbers is to

write an '/' between the numerals. [Pronounce '/' as you would


pronounce 'is not equal to'.] Thus, the sentence '9 + 3 / 4 + 7' is

true.
Some of the following true sentences are about numbers, and
some are about names of numbers. Be sure that you see that each
sentence is true.

(1) 4 + 8 = 9 + 3.

(2) 4 is an even number.

(3) '4' is a numeral for 4.

(4) '4' is not a number.

(5) '8 - 3' and '10 + 2' are numerals for 5.

(6) '2 + 2' is a name for 4.

(7) '2 + 2' is a name for 2 + 2.

(8) '2 + 2' is a naime for 3+1.


(9) 2 + 2 is the sum of 2 and 2.

(10) 2 + 2 is the sum of 3 and 1.


1/13

If one reads aloud the paragraph beginning 'A short way', the pur-
pose should be to direct attention to the meaning conveyed by the
written words, rather than to the written language itself. So, in
reading the paragraph one should make the sounds indicated by:

a short way of saying that each of two numerals names the


same number is to write an equality sign between the
numerals, thus, when we write the sentence five -plus -two-
equals- six-plus -one, we are saying that the number five-
plus-two is the same number as six-plus -one. our sentence
is true because the numeral five -plus -two and the numeral
six-plus -one are numerals for the same number, if we write
the sentence nine -plus -three-equals -four-plus -seven, our
sentence is false because the numeral nine -plus -three and
the numeral four -plus -seven are numerals for different num-
bers, a short way of saying that the numeral nine -plus -three
and the numeral four-plus -seven are numerals for different
numbers is to write an equals-with-a-slash-through-it
between the numerals, pronounce an equals-with-a-slash-
through-it as you would pronounce the phrase is-not-equal-to.
thus, the sentence nine -plus-three-is-not-equal-to-four-plus-
seven is true.

Questions concerning the written sentences (1) - (15) will be easily


settled if these sentences are translated in the above manner into the
spoken language.

'i*

A correct, and sometimes enlightening, way to read '9 + 3 = 4+7'


is as you would read *9 + 3 is 4 + 7*. Similarly, '9 +3^4
+ 7' may
be read as '9 + 3 is not 4 + 7', or as '9 + 3 is different from 4+7'.
Note that when the word 'is' occurs between two proper nouns, such
as '9 + 3' and '4 + 7', it expresses identity. But, contrast this with
the use of 'is' in sentences (2), (3), and (4) on page 1-L,. Here, and
in some other sentences in this group, the form is:

[proper noun] is [common noun],


and the word 'is' expresses being one of a kind. In the first usage
the word'is' is sometimes called the 'is of identity in the second
'
;

usage it is called the 'is of predication


'
. One logician has claimed
that these are just two of 27 different 'is's !

TC[l-L]a
both of
[1-M]

1/14

The discussion on true and false sentences is quite important. A


sentence is something which you look at, something which is written
on paper or on the blackboard. Sentences such as:
5 + 2 = 6 + 1, and: 9 + 3 = 4 + 7
convey information to the reader. The information conveyed to the
reader by the first sentence is correct information. In such a case,
we say that the sentence is true. The second sentence conveys
incorrect information, and we say that such a sentence is false.
Later in the course --much later --we shall call sentences with
equality signs in them 'equations'. Some equations are true and
some equations are false. [The students will also learn about sen-
tences which are neither true nor false. These are sentences like
*x + 5 = 9' and *x + y = y + x'. ] After going through this discussion
with the class you may want to proceed immediately to the exercises
in Parts D and E on page 1 -O.

The words 'true' and 'false* are commonly used with many meanings.
In this course, we try to use them with just one of these meanings. So,
as above, we say of a sentence that it is true, or that it is false, or
that it is neither true nor false. For example, we might write:
*5 + 2 = 6 + 1' is true,

or, for that matter:


'5+2 = 6 + r is false.

Each of these sentences is about the sentence The '5+2 = 6 + 1*.


first conveys correct information concerning this sentence, and the
second conveys incorrect information. The sentence '5+2 = 6 + 1',
itself, is about certain numbers and the operation of addition. It con-
veys correct information concerning these numbers and this operation.
One may, for emphasis, write [in place of '5+2 = 6 + 1']:
it is the case that 5+2 = 6 + 1

or:
it i s a fact that 5+2 = 6 + 1,

But, if one writes:


it is true that 5+2 = 6 + 1,

or:
5 + 2 = 6 + 1 is true,

one is using 'true' in the rather confusing sense which the word has
in the sentence 'this fact is true* [confusing, because a fact just i^].

TC[l-L]b
[1-L]

both
[1-M]
[1-L]

both
1/17

what you did. Your paper should now look like this

You can continue this exercise

CaJUJ.a-i^^ri.4,.CL.

r9 uAjCiO.'CaJ^f'^^'^^^f^

TC[l-M]c
\
[1-M]
1/16

Or, if you're lucky, it might even look like this

In technical works on logic, the notion of distinguishing in written


language between symbols and their referents is often called
'distinguishing between use and mention '. [See, for example,
Willard V. O. Quine, Methods of L ogic (New York: Henry Holt and
Company, Inc., 1950), pp. 37-38.~J In order to mention something
you must use a symbol which nannes it. For example, in the sen-
tence :

'4' is pronounced as 'for',

the referents mentioned are two symbols, and the symbols used are
names of the synnbols which are mentioned. Students who have a
good understanding of the use and mention distinction will have no
trouble in seeing that the foUov.'ing sentence is true:

'California' does not have single quotation marks around it.

This example is very much like Part B of the exercises. To see that
this sentence is true, do the following. Take a piece of paper and
write on it so that the paper looks like this:

Then, write on the same sheet of paper a sentence which describes

TC[l-M]b
[1-M]
. 1

[1-L]

both o'"

1/15

Part B isprobably the most difficult exercise in the Introduction.


We included it in order to catch the very kind of error referred to
in the exercise. When you look at the correct paper, what you see
is a numeral for the number 4; you don't see the number 4 itself.
When you look at the incorrect paper, what you see is a name of a
numeral for 4. The symbol on the incorrect paper is made up of
three parts --two single quotation marks and a numeral for 4. This
three -part symbol is a name of a name for 4.

What may confuse you here is that you are not accustomed to stare
at names. You are more accustomed to use names in order to
write about the things they name. You might use the symbol you
see on the incorrect paper in a sentence in which you wished to write
about the numeral which is named by this symbol. [For examples,
consider sentences (3) and (4) on page l-L, and sentences (12) and
(15) on page 1-M.] In other words, you might use *4* in a sentence
' '

in which you wished to write about *4', because *4' is a name for
' '

this numeral. On the other hand, you might use the symbol you see
on the correct paper in a sentence in which you wished to write about
the number which it names. [For example, consider sentences (2),
(3), and (6) on page 1-L, and sentences ( 1 1), (13), and ( 5) on page
1-M.] In other words, you might use '4' in a sentence in which you
wished to write about 4, because *4' is a name for this number.

One way of clarifying this matter in class is to do the following.


Write on the blackboard a numeral for 46. The blackboard should
look like this;

Then, ask a student to go to the board and write a sentence which


describes what you just did. The blackboard might then look like this;

'^' art tAjt. -d^u-AUrD-ix/ui

TC[l-M]a
[Introduction] [1-M]

(11) 4+1/4X1.
(12) You can put a '4' on paper.

(13) You can't put 4 on paper.

(14) ——
:; and - and '2' are nunnerals for tne same
number, but '2' is the simplest of these numerals.

(15) You can't write 4 but you can write a *4'.

EXERCISES
A. Write 5 numerals for 6 and 5 nunnerals for 0.

B_. A student was asked to write on a sheet of paper a numeral for


4. This is what his paper looked like :

He was wrong. His paper could have looked like this and he
would have been correct:

Explain. [Did you make this error in Part A?]


flntroduction]

numbers. Although the list con-


C The following is a long Ust of
numbers are listed. Rearrange
tains many numerals, only three

the list into threecolumns such that all of the numerals in a


column stand for the same number.
If someone asks you for
[Notice how the word 'list' is used.

a list of the students in your


mathematics class, he doesn't ex-
collect all the students and to
march them to him.
pect you to
What he hopes is that you will write down the names of the stu-

him a sheet of paper with names


dents, and that you will then hand
This sheet of paper is a list of the students .
And
written on It.

if you wanted to make yourself unpopular, you could write down


names. You
nicknames for each student as well as their given
would still be listing the students, but the
person reading the
in the class. So,
list might think there were many more students
say that although the following list contains many
it makes sense to

numerals, only three numbers are listed. ]

^^^ -W~^
3X(7 - 1)
3 + T "Sir

12X.05
7 + 3 52 ?q
^^ "
11
^^
1-15 15 + 3
j^-r^ -
J^^rz 5

l50
1 X
^ J_W/.^
Z5r\b
^
+ ±\
50/
^ 5
+ 1°
+ 5
A.
18
(3
^
+ 5)+
' 10 + 18

_L 8-8 (74 -
70) - 4 35 - (10 + 7) 6 +
10 10+0
3^'^'^S i 1^ X + (5 X 2)
Q 72 + a.
9.72 8 28
-itt
.
5
- (4
V 2)/ V
^ g.7g 3 87

(9^3) - (2X4.5) -(3X6) i)x(o. 3 x


- f (7 -
18
^
(% X 9/
1 .
5)
(18 3) 4) ' I
\
[l-O]

1/18

This kind of exercise occurs frequently throughout the UICSM text


materials. We felt that it was necessary to explain carefully how
we were using the word 'list'. This exercise tends to reinforce the
idea that a number has many nannes. You may be surprised by the
fact that students have little difficulty in interpreting the grouping
symbols in these numerals. Certainly, they will have more trouble
in doing the calculations than in interpreting the grouping symbols.

If a student points to the '9' in *9 X 2' and claims that the number 9
is listed, you might ask if John Jones is listed in a list which contains
the entry:
Mrs. John Jones.

Here are the three columns for Part C.

9X2 9 - 6 3 - 3
10-5 842
3 X(7 - 1)
12 X . 05
4 50
"^
7 + 3 52
3 3 7 4
"
15-5 26 X 2
5 5
29-11
15+3 1°

60 30
9o'i)'(i'i)
5 5 8 - 8
_6_
(3 + 5) + 10 10
(74 - 70) - 4
+ 18
6 +
2 58
35 - (10 + 7) 10+0 3
"
87
9.72 + 8. 28
3 X 978
(4 X 2) + (5 X 2) 5 X 978 18 - (3 X 6)

90 X |j X (0. 3 X 1.5) (9 + 3) - (Z X 4.5)


(18 - 3) + (7 - 4)

TC[1-N]
[1-N]

C. Th
[l-O]

\
[1-N]

1/20

G X ^ 0, X =

in ansvwer to this exercise. Neither answer complies with the in-


structions. One student wrote:
'^^ X = 0.

This, of course, is nonsense since -r- does not stand for a number.
6
In such a case, you might ask the student what number ?r ^^* ^^
should be that number which when multiplied by is 6. There ain*t
no such number']

For your own infornnation, please note that the sentence:

X =

is an open sentence. It is neither true nor false.

Answers for Part E.


1. (a) Any numeral for 8

(b) A numeral for any number other than 8

2. (a) Any numeral for 2

(b) A numeral for any number other than 2

3. (a) A numeral for any number other than 16


(b) Any numeral for 16

4. (a) A numeral for any number other than 2


(b) Any numeral for 2

5. (a) Any numeral for 793


(b) A numeral for any number other than 793
6. (a) A numeral for any number
(b) No numeral will convert this into a false sentence.

TC[l-0]b
[l-O]
1/19

The exercises inPart D illustrate several important ideas. We


have found elementary school students [grade 4] who felt that the
sentence in Exercise 7 was false because "you just wouldn't write
something like that". This illustrates a common misconception
that the relation of equality is not symmetric. These grade school
children felt that the equality sign acted like an arrow which pointed
to the correct answer. You can readily see the importance of
rennoving this misconception.

The sequence of Exercises 9, 10, 11, and 12 is important. A student


will say 'true' for Exercise 9, and jump to the conclusion that the
sentence in 10 is true also. What is happening here is that the student
thinks he is looking at instances of the connnnutativity principles in
these exercises. When he finds that the sentence in Exercise 10 is
false, he then attacks the sentence in Exercise 11 with more care, and
may be surprised to find that it is true, after all. By the time he
gets through with Exercise 12, he is willing to go back over all four
exercises and take another look, this time a more careful one.

'I-

Here are answers for Part D.

1. F 2. T 3. F 6.

7. 9. 10. 11. 12.

The exercises in Part E are typical of the kinds of completion exer-


cises which occur throughout the text. The purposes of these exercises
are to build the concept of equation-solving, and to establish awareness
at a nonverbal level of some of the basic principles of arithnnetic.
[Please do not mention these principles at this time.] Exercises 5
and 6 illustrate this purpose. After students have protested the
"unfairness" of Exercise 6(b), point out to them that they will find
many such exercises in this text. We expect students to be alert when
they do exercises. A correct answer for Exercise 6(b) is:
No numeral will convert this into a false sentence.

[Some teachers have reported that students submitted sentences

TC[l-0]a
.

[Introduction] [l-O]

D, True or false?

1. 7+9=4X5 2. 3+2=4+1
3. 8 + 7 = 8-7 4. 9X5/ 40+5
5. 6 X 2 / 10 X 2 6. 8 + 5 = 5 + 3

7. 8 = (2 + 2) + (2 + 2) 8. 2 + (2 + 2) = 2 X 2

9. 52 + 68 = 58 + 62 10. 52 X 68 = 58 X 62

11. 73 + 92 = 92 + 73 12. 73 X 92 / 92 X 73

Each of the following exercises contains a pair of sentences,


and each sentence has a blank space in it. For the first of

each pair of sentences, write a numeral in the blank space so


that the resulting sentence is true. For the second of the pair,

write a numeral in the blank space so that the resulting sen-


tence is false.

Sample 1 (a^) 9 + = 7 + 8

(b) 9 + = 7 + 8

Solution. (a-) 9+6 = 7 + 8 .Ja^ujl^

(b) 9+79 -7 + 8 ^<iPajl^

Sample 2. (a) 8 X / 12 X 2
(b) 8 X / 12 X 2

x 5"
Solution , (a) 8 /^^ 12 X 2 .JaaaJu

(b) 8 X J ;^ 12 X 2 JJUjl.

(a) 3 +_ = 5 + 6 2. (a) 7 - = 3 + 2

(b) 3 + = 5 + 6 (b) 7 - = 3 + 2
_

3. (a) 11 +. _ /^ 20 + 7 4. (a) 6 + / 15 - 12

(b) 11 + _ / 20+7 (b) 6 + / 15 - 12

(a) + 984 = 984 + 793 6. (a) X =

(b) + 984 = 984 + 793 (b) X =


[1-1]

has
-s
1/22

two measures by saying that 5 is 3 greater than 2. The relation 3^


greater than is the real nunnber *3. This relation holds between
numbers a and b of arithmetic when a = b + 3. It will be seen in
Unit 5 that a [binary] relation is a set of ordered pairs --the set of
those ordered pairs of objects between which the relation holds.
So, for example, the relation > for nunnbers of arithmetic is the
set consisting of those ordered pairs (a, b) such that, for some
number c of arithmetic, a = b + c; and the relation *3 is the set of
those ordered pairs (a, b) such that a = b + 3. The real number "2
is the relation over the numbers of arithmetic consisting of those
pairs (a, b) such that a + 2 = b--that is, "2 is the relation 2 less than .

The application of real nunnbers to measuring trips rests on the fact


that one can choose an origin on the road which is west, say, of all
points you wish to consider and compare the distance between this
origin and the ending point and starting point of a trip. The result-
ing measure of comparison will be a positive number if the ending
point is east of the starting point. If you wish to measure trips to
the west by positive nunnbers [and trips to the east by negative num-
bers] choose an origin east of all points to be considered.

[Recall that this discussion of real numbers was for your information
only. In particular, do not enlarge on the text by suggesting that
students choose an origin and lay out a scale. Students don't need a
scale to see the appropriateness of using real numbers to measure
trips. ]

You will notice that Unit 1 is devoted to an investigation of the arith -


metic of the real numbers, and that we do not discuss what is usually
called "algebra" until Unit 2. Our reason for proceeding this way
is that in earlier grades students have learned very little about num-
bers aside from algorisms and other computational tricks; in
particular, they have learned little if anything about general properties
of number systems [we are not referring to systems of numeration].
We believe it is essential that students learn something about such
feneral properties as associativity, commutativity, and distributivity
along with the inequality relation], before they attempt an "algebraic"
treatment of real numbers. Also, in Unit 1 we lay the foundation for
appreciating and constructing deductive proofs by asking students to
derive statements about particular real numbers from instances of
the basic principles. This preparatory work is exceedingly important,
and is best carried out apart from any initial confusion caused by a
too abrupt introduction of variables.

TC[l-l]b
[1-1]

has
-s
1/21

We cannot here define the numbers of arithmetic. We can only hope


that students have previously become acquainted with the numbers of
arithmetic through measurements of magnitudes such as lengths,
areas, volunnes, weights, speeds, etc. The numbers of arithmetic
are precisely the numbers which one uses as measures [with respect
to appropriate units] of such magnitudes. For example, the number
2 of arithmetic is

the inch-measure of a certain


length- -the length 2 inches,
the foot-nneasure of another
length- -the length 24 inches, and
the square -foot-measure of a certain
area- -the area of a rectangular
region of length 2 feet and width 1 foot.
The number nZ of arithmetic is the inch -measure of a certain length-
the length of a diagonal of a square of side 1 inch. The number n of
arithmetic is, among other things, the foot-measure of a certain
length- -the length of a circle whose diameter has the length 1 foot.
[The number of arithmetic is, perhaps, an exception to the state-
ment that each number of arithmetic is used as a measure of magni-
tudes. Whether it is depends on one's concept of magnitude.]

We introduce real numbers as numibers which can be used to measure


trips, that is, to measure directed changes. Again, this is an attempt
to acquaint students with a type of number through a physical applica-
tion rather than by definition.

However, for your own informiation, we shall sketch definitions of,


say, the real numbers *3 and "2 in terms of the numbers of arithmetic.
The basis of the definitions is the fact that one can compare two mag-
nitudes of the same kind by comparing their measures with respect to
a chosen unit.

For example, we say that the length 5 feet is longer than the length
24 inches because, comparing their foot-measures, 5 is greater than
2. The relation greater than holds between numbers a and b of arith-
,

metic when there is a number c of arithmetic [other than 0] such that


a = b + c. In the example above, 5 is greater than 2 because 5 = 2 + 3
[and 3 / 0]. We could express this more precise comparison of the

TC[l-l]a
[1.01] [1-1]

1.01 Distance and direction .


- -Imagine an east-west road which has
markers placed one mile apart at the side of the road. The markers
are labeled with letters. If you ride a bicycle from A to G, you say

K R A Q M O w B

-> East

that you have made a trip of 2 miles to the east; if you ride from S
to B, you again say that you have made a trip of 2 miles to the east.
Describe three other 2-miles-to-the-east trips on this road. Describe
three 2-miles-to-the-west trips on this road.
The 2-miles-to-the-east trips and the 2-miles-to-the-west trips
are alike in one important way. The length of each trip is 2 miles
[or, for each trip, the distance in miles between starting and ending
points is 2]. But, the trips are also different in an important way.
The trips to the east are made in a direction opposite to that of the

trips to the west.


Suppose you and a friend are at the point G on this road. Each
of you decides to make a trip that will take you two miles from G.
How many miles apart will you be at the end of your trips ? It is

easy to see that you cannot give a definite answer to this question.

It is not enough to know just the distances for the trips; you also have
to know something about the directions.

The numbers with which you have been working in school since
the first grade can be used in measuring distances for trips along
this road. But, using these numbers [let's call them the numbers
of arithmetic] doesn't tell whether the trips have been made in the
same direction or in opposite directions. If two people start at G
and make 2 -mile trips, we don't know whether they will be together
at the end of their trips or whether they will be four miles apart.
In order to measure trips which are made in one of two opposite
directions, we need numbers which will take into account both distance
[1-2] [1.01]

and direction. There are such numbers. They are usually called
real numbers? In this unit, you will learn many things about real

numbers --you will learn how to compute with them, you will learn

how to use them in solving problems, and you will learn that some
of them "act like" our fanniliar numbers of arithmetic.

NUMERALS FOR REAL NUMBERS


In order to work with real numbers we shall need some system
of naming them. We must have numerals for them so that we can
talk and write about them. Let us make up a system of numerals
which no one we know of has ever used before. After we have seen
that it is possible to work with numerals which we have invented,
we'll switch to a kind of numerals for real numbers which most other
people use. Our ideas of real numbers will not change in switching
from one system of numerals to another --only the names will change.

We want to use real numbers to measure trips. Since one im-


portant aspect of a trip is the distance between starting and ending
points, and since our "old" numbers of arithmetic do serve quite
well in measuring distances, we shall use as part of each numeral
for a real number a numeral for a number of arithmetic. Then,
because we need to include direction in measuring trips, we shall
complete the numeral for the real number by including an arrow.

K R Q M O w B

East

Suppose, again, that you and your friend decide to take 2-nnile trips
starting at G. If your trip is measured by the real number"? and

*Real numbers are not any more (or less) real than other kinds of
numbers. In a later course [when you learn about complex nunn-
bers] you will see why, historically, the word 'real' came to be
used. Sometimes real numbers are called directed numbers or
signed numbers .
[1-3]

U be
c

1/23

Note the careful description of how numerals for real numbers are
formed. We do not wamt the student to get the notion that "a real
number is a number of arithmetic with a plus sign or a minus sign
stuck in front of it". We can't stick a plus sign in front of a number
since a number is an abstraction and has neither front nor back nor
top nor bottom! It is for this reason that w£ do not use the term
*signed nunnbers' when referring to the real numbers.

'1^

As soon as the student encounters the numeral he will want to


'
2 ',

pronounce it. This will certainly be the case if he is reading aloud.


Let the class agree on a pronunciation of this numeral as well as the
numeral which includes the arrow pointing to the left at the top of
page 1-3. We give a pronunciation just before the exercises on page
1-3. If the students did not suggest a pronunciation which is in
agreement with the one we gave, they can use their own or switch to
ours. This is a good opportunity to point out the arbitrariness of
pronunciations of mathematical symbols. Since these nunnerals will
be dropped after a page or so, it is not important which pronuncia-
tion is used. However, for a symbol for which there is a "standard'
pronunciation, the students should see that it is usually better to
adopt it than one of their own making.

Answers for Part A [on page 1-3].

1. (a) r (b) r (c) r cd) r (e) r (f) t


2. (a) K to S, T to W, R to B (b) B to R, W to T, S to K
(c) Kto M, T to O, R to S. etc. (d) B to G, W to Q, S to A, etc,

(e) KtoT, etc. (f) K to the point halfway


between R and A, etc.

TC[l-2, 3]
[1-2]
[1.01] [1-3]

your friend's by the real number 2, you can be sure that you will be
4 miles apart at the end of your trips. How far apart will you be if

each of you makes a trip of 2 miles ? If each makes a trip of 2 miles ?

Suppose you take a trip of 3 miles and your friend takes one of 5

miles, starting from the same point. What real number measures
the trip which your friend will have to take to get from where he is

to where you are ?

Notice that we have not yet said whether the real number 2 meas-
ures a trip-to-ths-east or a trip-to-the-west. This is something
which should be decided for each problenn in which you want to meas-
ure trips made in one of two opposite directions.
In working with numbers like 3 and 9, you will want to talk about
them as well as write about them. So, you need to decide upon the
pronunciation of their numerals. Let's agree to pronounce '3 'as you
would pronounce 'right three' and '9' as 'left nine'.

EXERCISES
A. Let us agree that the number 2 measures a 2-miles-to-the-east
trip. Then 2 measures a 2-mile trip in the opposite direction.

K R Q M O s w B

-^ East

Give the real numbers which measure the trips listed.

(a) KtoR (b) R to G (c) W to O


(d) B to Q (e) Q to B (f) G to S

List three trips which are measured by each of the given


real numbers

(a) 8 (b) 8 (c) 6

(d) T (e) t (f) 275


[1-4] [1.01]

B. Consider taking trips along a north-south N


road. Let 2 measure a 2-miles -to-the- 1 ?7
south trip. Then 2 measures a 2-mile
trip in the opposite direction.

1. Give the real numbers which measure


these trips.

(a) T to K (b) Y to R
(c) P to A (d) A to P "5?

2. List three trips which are measured /0/


by each of these real numbers. 'Si
(a) (b) (c) 3.5

Imagine trips taken along an east-west road, and trips to the

east measured by right real numbers. Each of the following


statements gives the starting and ending points of a trip and
the real number which measures the trip. Use them to fill in

the miarkers on the map.

nD[ya[pp[pnnn
-> East

(1) a trip from A to G is measured by 4.

(2) a trip from J to G is measured by 2.

(3) J to B. 3 (4) I to E, T
(5) F to C, 2 (6) G to D, 5

(7) L to H, 2 (8) B to A, T
(9) AtoF, T (10) E to C. ?
(11) L to F, 1 (12) K to E. T
o^ o* o^
'1^ 'I- 'I"
[1-5]

1/24

The main purpose of Part B is to provide variety in the use of real


numbers, and to emphasize the direction-distance aspect of trips.
The use of the arrow -numerals might lead students to believe that
the maps used had to be oriented left to right.

Answers for Part B,

1. (a) (b) (c) (d)

2. (a) Y to N, QtoK, R to S, etc.

(b) Ato P, T to S, J to K, etc.


(c) Ato the point halfway between P and S,

T to the point halfway between S and K,


J to the point halfway between K and N, etc.

Part C emphasizes that each trip has an initial point and a terminal
point. The real number used to measure the trip tells you the direc-
tion of the terminal point from the initial point as well as the distance
between the initial and terminal points. A
thought -provoking question
to ask when students have finished Part C is How would you fill in :

the markers if trips to the east were measured by left real numbers ?

'I-

Answer for Part C.


When the markers are correctly filled in, the map will look like this,

B I
G] [E] [J] [T] H ^ fHJ [p ITl ® Ok|

East

TC[l-4]
[1-5]

hat
[1-4]

B. Cor

1/25

The purpose of Part D is to develop the concept of addition of real


numbers. Do not tell students the purpose of this exercise since it
is better pedagogy to have the students fully aware of the concept
before naming it. Certainly, it will not increase their effectiveness
to be told that they are actually adding real numbers; in fact, you
nnight detract from this effectiveness since they may bring to the
solution of the exercises misleading connotations of the word 'addition*.
By the time students have done a quarter of the exercises on page 1-6,
they will have developed their own rules for addition. Sonne students
may want to tellyou and the class the short cut they have discovered.
Do not allow them
to meike a public statement. Announce that you
expect students to find short cuts, but that you don't want the game
spoiled for others in the class by having a student tell what his short
cut is. If a student is overcome with the desire to let you know that
he can tell you the rule, then go to his desk, lean over, cind have him
whisper the rule to you. You will, of course, hear sonae kind of
jumble of words in which the ideas of numeral and number are un-
fortunately mixed, but do not try to get the student to formulate his
rule precisely. It is best to avoid any verbalization at all at this
time because it is much too difficult a task for the student. [The
rules are given formally in Unit 2.}

Suppose, in checking answers to these exercises, a student gives a


wrong answer for, say. Exercise 16. If you want to work this exer-
cise for the class, you must use the same physical interpretation as
was given in the samples. In other words, it should always be possible
for a student to find answers to these exercises by using the physical
interpretation. Theoretically, he does not need the short cut addition
rules. However, no student will be satisfied with the long, drawn
out method of solving these problems. He will start an active search
for a short cut. The trick is to get him to discover this short cut by
himself. This is probably the first opportunity in the UICSM program
for a student to use his own resources in finding a short cut, and the
problem is easy enough so that each student will meet with success.
If someone else gives the game away [teacher, fellow student,
or
parent], then you have lost am opportunity to build the attitude in the
student that mathematics is a subject which makes sense because it
"comes from" the student himself.

TC[l-5]
[1.01] [1-5]

One of the important things you learned in Part C was that


if you knew the direction of trips measured by right real num-
bers, you could then tell the location of a second point with
respect to a first point just by knowing the real number which
measures the trip from the first point to the second point.

Each of the following exercises is a list of real numbers. These


are measures of successive trips along an east-west road. [Right
real numbers are used tomeasure trips -to-the-east. ] The first

real number in each list measures


a trip starting at a point A.

The second measures a trip whose starting point is the ending


point of the trip fronn A. The third measures a trip whose start-
ing point is the ending point of the second trip. Etc.
Your job in each exercise is to tell the location of the ending
point of the last trip with respect to A by giving the real number
which measures the trip from A to that ending point.

Sample 1 . 3, 5.

Solution . One way to solve this make a


problem is to

sketch of an east-west road and mark a point A.

-> East
Then mark the ending point of the trip measured by 3.

This is a point 3 units to the right of A.

A B
-• —— I I •-
East

Now mark the ending point of a 5 -unit trip whose starting


point is B. This is a point 5 units to the right of B.

A C
-• — — —B— — — —
I I • I I I
I •
East
[1-6] [1-01]

So, C is 8 units to the right of A. This means that


the ending point of the last trip is 8 units east of A.
The real number which measures the trip from A
to the ending point of the last trip in the succession
is 8. Answer : 8

Sample 2 . 2, 7, 9.

Solution. B
-• — —A
I •-
-> East

B
-• — —A— — — — —C
I • I I I I •-
-> East

DBA
— — — — —— — — —
-• I • I • I I I •
C»-
•> East

So, the real nunnber which nneasures the trip


froni A to ending point of the last trip in the succession
is 4. Answer : 4

[Note : Use drawings for doing these exercises as long as you

need to. You may be able to find short cuts so that you

can give the answer immediately.]

1. 2, 5 2. 3, 6

4. T, "8 5. T2, 7 6. T, "s

7. T, 7, ? 8. 7, 7, T 9. T, T, To
10. "5 , 7, "a 11. 7, 7, 7 12. T, T, X
13. 7, To, 7. T 14. T, 7, 15, 7b 15. T, T, 7, Ti
16. 78, "83 17. ?5 foT" 18. ToT, "95

19. To, 98, "97 20. 74 fo". 7? 21. 41. 5. 57, 44. 5
[1-7]

1/26

A teacher in one of our pilot schools suggested that the Solution for
Sample 2 would be nnore effective if pictured like this :

H ^
B ^East
B
-•
I
• — 1
\
1—C •-
East
D C
-• — — — —•—I
I • I
1 1
1 —«—
-East

You may want to use this idea for diagramnning other trips on the
blackboard.

'1^

Please note that none of these exercises and none of the exercises on
pages 1-9 and 1-10 involve the real number 0. This is a tricky notion
which requires careful explanation. It is introduced on pages 1-13
and 1-14. If a student raises the question of at this time, compli-
ment hinri on asking a good question, and tell him that the matter will
be taken up in a day or so.

Answers for Part D [on pages 1-5 and 1-6].

1. T 2. T 3. T 4. T 5. T
6. 1" 7. 15 8. 10 9. 18 10. T
11. T 12. T 13, 10 14. 16 15. 16

16. T 17. 12 18, 12 19. 49 20, 11

21. 60

TC[l-6]
[1-6]
[1-7]

%
[1-6]

1/27

Students who have been exposed to positive and negative numbers in


the past may point out that they used plus and minus signs in the con-
ventional position. After their experience with the arrow-numerals,
they should be prepared to accept the idea that the fornnation of
numerals is am arbitrary process, and that this text chooses to do it
this way rather than the other way. The discussion starting on
page 1-80 will give you the rationale for our selection of the super-
script signs in place of the conventional signs. You will have no
difficulty in training yourself to use the superscript signs. In fact,
you will probably find yourself using the superscript signs in your
conventional classes. [On the other hand, you may find it difficult
to train yourself to say, for example, 'negative three' instead of
'minus three'. Just keep trying !]

Be sure students understcuid the procedure for forming a numeral


for a real number. As mentioned on TC[l-2, 3], we have taken pains
to give a careful description to forestall the student's tendency to say
that "a real nunnber is anumber of arithmetic with a plus or a minus
sign attached to it". If more explanation is needed, point out to stu-
dents that Mrs. Tonn Jones is not Tom Jones with a 'Mrs. attached
'

to him. [Even though 'Mrs, Tom Jones' i_s 'Tom Jones' with 'Mrs. '

attached to it. ] [Seepage 1-108.]

The Exploration Exercises at the bottom of this page provide a traji-


sition from the exercises on page 1-6 to the work on addition on
page 1-8. A student can translate to the arrow-numerals and proceed
as on page 1-6. After a few such translations, he will be able to do
the exercise without translating.

You will find sets of exploration exercises scattered throughout the


text. The purpose of exploration exercises is to help students build
a concept prior to the discussion which follows the exploration exer-
cises. We want students to keep saying to themselves, as they read
the text discussion, things like:

Of course ! I knew that long ago.

Answers for Exploration Exercises.

1. *9 2. "12 3. -3 4.

TC[l-7]
[1.01] [1-7]

POSITIVE AND NEGATIVE REAL NUMBERS


In the preceding exercises you worked with real numbers, using
them to measure trips. V/e invented numerals for these numbers.
We could continue to use these numerals throughout all of our work
with real numbers, but it is important that you become familiar with
the more standard ones.
The real numbers which we have called 'right real numbers' and
'left real nunnbers' are connmonly called positive numbers and negative
numbers , respectively. Positive and negative numbers come in pairs.

Each pair contains a positive number and a negative number, and both
numbers correspond with the same number of arithmetic. The numbers
in each pair can be used to measure trips over the same distance but in
opposite directions . The direction of trips measured by positive num-
bers is called the positive direction , and the direction opposite to the

positive direction is called the negative direction .

~*
In naming positive numbers we shall use a '
"^ '
instead of the ' '.

For example, we shall write


' ^3' instead of '
3 *

In naming negative numbers, we shall use a ' ~ '


instead of the '
"'" '.

So, we shall write

* -3' instead of ' 3 '

We form a numeral number by prefixing a '* or a ~ to a


for a real ' ' '

numeral for the corresponding number of arithnnetic. [The numerals


'*3' and '"3' are pronounced as 'positive three' and 'negative three',
respectively. ]

EXPLORATION EXERCISES
Each of the following exercises gives the measures of successive
trips. Find a real number which measures the direct trip from the
starting point of the first trip to the ending point of the second trip.

1. *7, *2 2. "3, "9 3. *6, "9

4. "10, *7 5. "11, *20 6. *20, "11


[1-8] [1-02]

1.0 2 Addition of real numbers. --In the Exploration Exercises


you practiced finding the nneasure of a trip made from the start-

ing point of the first of two successive trips to the ending point
of the second. For example, consider a *3-mile trip followed by
a *5-mile trip. The direct trip which takes you from the starting
point of the *3-mile trip to the ending point of the *5-mile trip is a
trip of *8 miles. And, similarly, a direct trip which takes you
fronn the starting point of a "9 -mile trip to the ending point of the
"10-nnile trip which follows it is a trip of "19 miles.
The idea of a pair of trips one of which is tacked onto the other
suggests the notion of "adding". And, this suggests that what you
did with the measures of two such trips to find the measure of the

direct trip should be called addition of real numbers . The exajnples


above, then, show that

*3 + *5 = *8

and that "9 + "10 = "19.

Take still another example.

*9 + "5 = ?

We have decided that the sum of a first real number and a second
real nunnber is the measure of a direct trip from the starting point
of one trip to the ending point of a following trip, the trips being
measured by the first and second real numbers, respectively. So,
*9 + '5 is the measure of a direct trip which takes us from a starting
point to that point which would be reached by a *9-mile trip followed
by a 'S-mile trip. Do you see that such a direct trip is a *4-mile
trip ? We say that
*9 + -5 = *4.

Can you solve the problem:

"7 + *2 = ?
1/28

The use of the 'addition' in naming an operation with real num-


word
bers is the cause of a confusion which is not usually recognized in
conventional textbooks. The first confusion of this type occurred
when we used the word 'numbers' in connection with real nunnbers.
The word 'numbers' is used by a student in elementary school, first,
with reference to natural numbers. Then the word 'numbers' is used
to refer to rational numbers, which are different entities from natural!
numbers. Then the word 'numbers' is used to refer to entities such
as ir and n/Z and other irrational numbers, entities of still another
kind. Finally, we use the word 'numbers' to refer to real nunnbers,
things which are different from all the other things a student has
learned to call 'numbers*. [And, in a later unit, he will use the word
'numbers' for still different things, the complex number s. ] So, the
student is continually "enlarging" the meaning of the word 'numbers'.
Now, this is a perfectly natural process and is a conconnitant of grow-
ing up. However, the teacher needs to realize that the student may
have difficulties in these successive enlargements of the meaning of
a word. [An indication of the reluctance of children to make this
enlargement is seen in the use of the word 'fractions' to designate
rational numbers. You may find some students who will claim that
a rational number is not a number at all. These students still think
that 'numbers' refers only to the natural numbers. ]

A similar enlargement of meaning takes place with the word 'addition'.


We have the operation of addition of natural nunnbers, the operation
of addition of rational numbers, and the operation of addition of num-
bers of arithmetic. These are distinct operations. * But the word
which names the operation is the same in each case. The elementary
school student does not seem confused by this enlargement of nneaning
because in all three cases the notion of addition involves the notion of
increasing. [On the other hand, there is much confusion in connection
with the word 'multiplication'. The student first learns this word in
connection with an operation with natural numbers. In that case the
operation is closely related to the operation of addition, and 'multi-
plication' has the connotation of increasing. But, students frequently

An operation a set of ordered pairs. [See pages 1-67 and 1 -107. ]


is
The operation adding 2 defined on the set of natural numbers contains
such pairs as (1, 3), (2, 4), (3, 5), etc. The operation adding Z defined
on the set of rational numbers contains (0, 2), (1/2, 5/2), (1, 3),
(5/3, 11/3), etc. These are evidently different sets of ordered pairs,
and so are two operations, even though both go by the same name,
adding 2^.

TC[l-8]a
[1-9]

1/29

protest the use of 'multiplication' in connection with rational numbers


when they find that multiplication of one number by another can lead
to a smaller number. ]

In connection with real numbers, the operation of addition does not


involve the idea of increasing. In fact, a student may think the use
of the word 'addition' for this operation with real numbers is "crazy"
because lots of times he finds that his short cut tells him to "subtract"
when he is supposed to add! So, what we must do when we introduce
the term 'addition of real numbers' is to indicate that we are dealing
with an operation involving these new numbers, and that since the
physical interpretation of this operation suggests the notion of "adding"
in the sense of "tacking on", we shall use the same word 'addition'
to name this operation as we used in the case of numbers of arithmetic.
But, the operation of addition of real numbers is different from the
operation of addition of numbers of arithmetic. Therefore, since the
operations are different, it is not surprising that the new operation
does not have the same properties as the old. The feeling the student
experiences is that addition of real numbers is a "combination" of
the operations of addition and subtraction of numbers of arithmetic.

Ifa student has difficulty in finding the sum of two real numbers be-
cause he has not yet formulated a rule for himself, go back to the
arrow-numerals and the trips along a road in order to help hinn find
the sum.

Note that when we speak of adding ^5 to *3, we write *3 + ^5 [rather


'
'

than ^S + "^3']. Similarly, when later we introduce multiplication of


'

real numbers, the numeral to the right of a multiplication sign will


name the multiplier So, when we speak of multiplying *3 b^ *5, we
.

shall write '"3 X ""S [rather than *5 X ^3'].


'
'
Like all conventions of
notation, these are arbitrary. Our main reason for choosing these
particular conventions is that they enable students to associate + *5
'

and X *5
'
with the operations adding *5 and multiplying by *5 in the
'

same way as they associate - *5' and v '^5 with the operations sub-
' '
'

tracting *5 and dividing by *5.

TC[l-8]b
[1-8]

1.02 f

you
\
[1-8]

1/30

Answers for Part A [on pages 1-9 and 1-10],

1. M 2. Ml 3. "5 4. "1 5. 2

6. *6 7. *7 8. "10 9. M 10. MO
11. M 12. *8 13. M 14. *35 15. "1

16. '35 17. MO. 18. *5.6 19. "6.9 20. -21

21. "419 22. *9 23. *5 24. M 25. -1

26. *6 27. M5 28. M5 29. *27 30. *7

31. "8 32. M 33. "4 34. *6 35. M


36. *5 37.
M 38.
*5
39. "8 40. M
5

41. "2 42. M 43. "1 44, M 45. "4

46. *1 47. M 48. *6 49. M 50. "74


-2 -5
51. 36 52. M06 53. M06 54.
5
55.
7

56.
4
57. -1 58. M.4 59. -4 60. *1444

61. M444 62. *6 63. "5 64. M5 65. "129

66. M87 67. "1482 68. *3

[Note of warning! In all parts of the Miscellaneous Exercises except


Parts 1 and J, we have used the oppositing symbol — when naming '
'

negative numbers. Therefore, you should not assign exercises from


the Miscellaneous Exercises section of the book until after the "New
Names for Negative Numbers" subsection [pages 1-8^'ff, ] has been
studied. ]

TC[l-9, 10]
[1.02] [1-9]

EXERCISES
A. Simplify each of the following numerals.

Sample . ""S + *4

Solution .
*1

1. % + -2 2. *3 + "8 3. "2 + -3

4. '3 + -4 5. ^3+ -5 6. ^8 + -2

7. *6 + *1 8. -2 + -8 9. ^9 + -8

10. *3 + *7 11. m + -7 12. ^3 + *5

13. *18 + -17 14. nS + *17 15. -18 + *17

16. -18 + -17 17. ^4.3 + *5.9 18. "2.7 + *8.3

19. *12.4 + -19. 3 20. *81 + -102 21. -765 + *346

22. (*6 + *7) + -4 23. ( "3 + *6) + *2

24. "4 + "3 25. *7 + "8 26. *9 + "3

27. ""5 + *10 28. *3 + M2 29. *12 + ^15

30. *2 + *5 31. 4 + "4 32. '3 + *7

33. "7 + *3 34. *2.5 + *3. 5 35. -1 + ^2

*9 -^6
^1 + '^
36.
3
,

3
37.
5 5
38. 3^2
39. '3 + "5 40. n + *3 41. *10 + "12

42. *2 + -1 43. "2 + n 44. *1 + -3

45. *3 + -7 46. -6 + "7 47. ^8 + -9

48. *21 + '15 49. -12 + ^3 50. -32 + -42

51. *17 + *19 52. "181 + ^75 53. *181 + "75

2 +"1
54. 5^5 55. 7^7 56. 4^2
(continued on next page)
[1-10] [1.02]

57. -4-4 58. M.6 + "3.2 59 '4 ^ ^4

60. *3875 + "2431 61. "2431 + *3875 62. ("4 + "3) + *5

63. (-15 + *3) + *7 64, ("15 + *28) + *2

65. ('29 + "98) + "2 66. (*87 + *64) + "36

67. ("997 + "482) + "3 68. (*3 + -7) + (^9 + "2)

[More exercises are in Part B, Supplementary Exercises .


]

B. Bill's father gave him $3 to start and operate a flower business


for one week. His father told him to use the $3 to buy flowers
the first day and to sell as much as he could each day. He also
told him to use all of the money he collected on one day to buy
flowers the next morning. Although spending all of his money
each morning might not be the best business procedure, his father
wanted to see how far up he could "run" the $3.

Here is a record of his week's business.

Expenses Sales Outcome

Monday 3.00 4.00

Tuesday 4.00 5.20

Wednesday 5. 20 4.80

Thursday 4.80 4.90

Friday 4.90 4.70

Saturday 4.70 6.80

On Monday, Bill's assets changed by $1, and this change was


an increase in his assets. That is, the outcome of business on
Monday was a profit of $1. The outcome of business on Friday-
was a loss of $0. 20.
\
[1-11]
[1-10]

1/31

The outcome of a day's business is a chanp;e in assets. This may be,


for exannple, a $*1.00 change [read as *a positive one dollar change'],
or a $"0.20 change. The nraeasures of the outcomes are real numbers,
*1.00 and "0.20. [Strictly, these are the dollar -measures of the out-
comes. The cent-measures would be *100 and "20, respectively.
This is entirely analogous to the case of a length, say, the length 1
foot, which has the number 1 of arithmetic as its foot-measure and
the number 12 of arithinetic as its inch-measure.] In the table on
page 1-10 we have listed the measures of the expenses and sales,
and expect the students to list the measures of the outcomes. "We
shall, as in the table headings, usually avoid the awkward phrase
'measure of the outconne'. By 'outcome' we are referring to the
real number which is the dollar -measure of outcome.

The word 'predict' in Exercise 2 of Part B does not mean guess or


estimate. [Some students have incorrectly interpreted the word in
that manner.] When one has become aware [verbally or nonverbally]
of a generalization, he uses it as the basis for prediction. This is
one of the ways in which we deternnine that a student has achieved a
nonverbal awareness of a generalization.

Answers for Part B [on pages 1-10 and 1-11],

1. Outcomes: Monday, *1 00; Tuesday, *1 2C Wednesday, "0.40;


. . ;

Thursday, *0. 10; Friday, "0.20; Saturday, *2-10.


2. Sum of outcomes: *3.80.

Answers for Part C [on pages 1-11, 1-12, ajid 1-13].

1. If we decide to use positive numbers to measure profits, then


measure of a $3 profit is *3,
the
measure of a $6 loss is "6, and
the
measure of a $5 profit is *5.
the
The dollar -measure of the total outcome is (*3 + "6) + *5, that
is, *2. So the total outcome was $*2.00, a profit of $2.00,
2. [(*6.80 + *2.55) + "5.42] + *1.53 = -^5.46. $5.46 profit, or $*5. 46.

TC[1-11]
[1.02] [1-11]

$0
@ ill] 1^1]
$6 $7

Loss Profit

The outcome of a day's business for Bill caii be regarded as a


trip, for it involves "distance" [difference in assets between
the opening and closing of the business day] and "direction"
[increase or decrease in assets]. So, the outcome of a day's
business cam be measured by a real number. If we decide that

a profit is to be measured by a positive number then the outconne

of Monday's business is $*1. 00, and the outcome of Friday's


business is $"0.20. In dollars, the measures of the two out-

comes are * 1. 00 and "0. 20.

1, Complete the table given on page 1-10 by listing the [measure


of the] outcome of each day's business.

2. Bill started with $3. 00 on Monday morning and ran it up to


$6. 80 by Saturday evening. Predict the sum of the outcomes
for the six days of business, and check your prediction by
actually adding the real numbers listed in the outcome colunnn.

Solve the following problems. Use real numbers wherever they


apply.

1. Phil made 3 dollars profit the first day of business, lost 6


dollars the second day, and made 5 dollars profit the third
day. What was the outcome of the total business for the
three days ?

2. Ed made $6. 80 profit the first day of business, made $2. 55


profit the second day, lost $5. 42 the third day, ajid made

$1. 53 the fourth day. What was the outcome of the four days
of business ?

(continued on next page)


[1-lZ] [1.02]

3. Zabranchburg High's football team gained 3 yards the first


down, lost 4 yards the second down, gained 5 yards the
third down, and gained 7 yards the fourth down. Did they
make a first down?

4. John and Fred are playing a game. John wins 3 points in


the first round, loses 4 points in the second round, ajid
wins 5 points in the third round. What is his score at the
end of the third round?

5. A department store has 6 floors above the ground floor arid


2 floors below the ground floor. The first floor above the
ground floor is called 'mezzanine', the next floor above the
mezzanine is called 'first floor', the floor next above the
first floor is called 'second floor', etc. The first floor below
the ground floor is called 'first basement' and the floor below
that one is called 'second basement'. An operator makes the
following trip: ground floor to mezzanine to second floor to
first floor to third floor to fourth floor to mezzanine to first
basement to third floor to second basement to first floor,

(a) If you use real numbers to measure the separate trips,


what is the sum of these real numbers ? Could you have
predicted your answer without adding ?

(b) If the floors are 17 feet apart, how mciny feet did the
operator travel during the entire trip?

(c) Which traveled the greater distance, the operator's head


or his feet ?

6. Two cyclists start from the same home at the same time.
John travels 4 miles east, then 2 miles west, then 3 miles
east. He then travels west until he meets Walt. Walt
starts by traveling 3 miles west, then 1 mile east, then 3
miles west, then east vmtil he meets John.
1/32

3. If we decide to use positive numbers to measure gains, then the


measures of the successive "trips" are *3, "4, *5, and *7. The
measure of the net result is [(*3 + "4) + *5] + *7, that is, *11. So,
there was a net gain of 11 yards. Since the requirement for a
first down is a gain of at least 10 yards, the team did make a
first down. [If we decide to use negative numbers to measure
gains, then the measures of the successive trips are "3, *4, ~5,
and ~7. The measure of the net result is "11, aund the result is
still, of course, a gain of 1 1 yards. ]

4. If we use positive numbers to count points won, then the results


of the successive rounds are *3, "4, cind *5. John's score at the
end of the third round is (*3 + "4) + *5, or *4. Students may want
to say that John's score is "4 points to the good".

5. (a) If we use *1 as the measure of a trip upward from one floor


to the next, the description of the successive trips as given
in the problem could be represented by:

n, *2, -1. *2, n. -4. -2, ^5. -6, *4.

The sum of these real nunnbers Students should be


is *2.
able to predict this, since the last trip terminated 2 floors
above the starting point of the first trip.
(b) To determine the distance traveled, the student must consider
how many times a trip of 17 feet was nnade. Thus:
ground floor to mezzanine: 1 time,
mezzanine to second floor: 2 times,
second floor to first floor: 1 time,
first floor to third floor: 2 times,
third floor to fourth floor: 1 time,

fourth floor to mezzanine: 4 times,


mezzanine to first basennent: 2 times,
first basement to third floor: 5 timies,
third floor to second basement: 6 times,
second basement to first floor: 4 times,

or a total of 28 times that a trip of 17 feet was made. So,


the operator traveled a total of 476 feet.

(c) The operator's head and feet both traveled the same distance !

[You may wajit to ask students whether they could illustrate


this graphically. One of our participating teachers reported
that a seventh grade boy in her class did this (to convince
others of the correct ajiswer) by using a pencil and a yard-
stick. He slid the pencil up the yardstick a given number of

TC[l-12]a
[1-13]

1/33

units to show that the point moved just as far as the eraser.
Another student suggested the example of the hood ornament
and door handle of a moving car.]

This problem usually creates a lot of interest; it may be several


days before nnost of the students are sure of the correct answers,
and some nnay not be able to determine them at all. After most
of the class have determined the correct answers, you may want
to have diagrams drawn on the board to show what is involved.
Here is one way it could be done. [You may have students who
can explain the problem, by using diagrams of their own.] The
first seven miles covered by each boy :

Walt John
— 1 H—
1
1— — 1

1
1
1_ —-_— ,
1
-H

0)
East - >
H
-i —

i
!1
-1 1 1
1
— i
- ... -4--

Here it can be seen that V/alt was 5 nniles west of home, and
John was 3 miles east of home, after traveling 7 miles each.
Then, Walt turned and started home, but John continued travel-
ing east for 2 miles. The next 2 miles of travel is pictured:

Walt John
-H -+-

O —>- -H East
X

and it is seen that V/alt was just 3 miles west of home, but

TC[l-12]b
[1-12]

3.
[1-13]

John was 5 miles east of home, after these 2 miles of travel.

Finally, to show the part of the road traveled by each boy until they
meet:
V/alt John
+ 0)
+
->->->f* ( C C ( 's C ( East

5 4 3 2 1 2 3 4 5 6

place where they meet


So, the answers to the questions are

(a) 1 mile (b) west (c) 14

'I"

Notice that each of Exercises 5 and 6 is marked by a five-pointed


star. This our indication of an optional problem [or subsection],
a problenrx which can be omitted without destroying the total develop-
ment. It is usually used with a problem that presents a genuine
challenge to the very best students.

TC[l-12]c
[1-12]

3.
[1.02] [1-13]

(a) If both cyclists travel at the same speed, how far from
home do they meet ?
(b) In which direction must they travel if they head directly
for home together?

(c) How many miles has each cyclist traveled by the time
they reach home ?

[More exercises are in Part C, Supplementary Exercises .


]

TRIPS OF DISTANCE

The problems in adding real numbers up to now have not included


such problems as finding the sum of *3 and "3, or finding the sum of
*7 and "7. Just as it is possible to find the sum of any pair of numbers
of arithmetic, we would like it to be the case that there is a sum for
every pair of real numbers. Our problem, then, is to give an interpre-
tation of, say, '*3 + ~3' so that '"^3
+ ~3' is a numeral for a real nunnber.
Let us try our trip interpretation. Using the picture on page 1-1,
' *3 + "3' should be a numeral for a real number that measures the
trip whose starting point is that of the trip, say, from A to M and
which has tue same ending point as the trip from M to A. In other
words, * 3 + "3 should measure the trip from A to A. This is an
unusual trip! For even though it involves the distance aspect, it

does not involve a direction. In fact, we wonder if we can even


consider this as a trip at all. If we don't consider it as a trip, and

if we wish to continue thinking of real numbers as nunnbers which

measure trips, then we must admit that collections of marks such as

' *?+-?' and '*3+-3' and '


"4 + M
are not numerals for real numbers. In other words, we would have
to admit that there are pairs of real numbers which do not have

sums. Rather than do this, we prefer to stretch our imaginations

a bit and regard a "trip from A to A" as a trip which can be

measured by a real number. Naturally, the real number to be


used in measuring such a trip is neither a positive number nor a
negative number. So, it must be one which we have not yet discussed.
[1-14] [l.OZ]

A simple numeral which we shall use for this real number is a


numeral which is also used for a number of arithmetic. It is the

numeral '0'. Thus, each of the numerals '*7+"7', '*^3+"3', and


' "4 + *4' can be simplified to '0'. Each of these numerals names
the real number 0.

Now, just as we do not confuse the positive and negative real


numbers with numbers of arithmetic [we don't think, for example,
that the real number ~3 is the same as the number 3 of arithmetic],
so we must not confuse the real number with the number of

arithmetic. It may seem difficult to avoid this confusion since both

numbers have the same nanne, and you may wonder why we did not
invent a new nunneral for this real number. We could have used
' -6- '
or '
([) 'or '
X '
or even '
f ', but everyone else uses '0', so
we shall also. Actually, it will not be too hard to keep the idea of
thenumber of arithmetic separate from the idea of the real
number 0, since any problem in which either number is to be used
will tell you which meaning to give to the nunneral '0'. For example,
the realnumber measures the outcome of a day's business in which
expenses were the same as sales, whereas the nunnber of arith-
metic measures the content of the money-box when the box is empty.

EXERCISES

A. Simplify.

1. *9 + ~9 2. "9 + "9 3. *18 + "18

4. *3 4 5. + ^3 6. + "3

7. "3+0 8. *20 + *30 9. *30 + "50

10. (*9 + "7)+*7 11. {M9+M2)+-12


12. ("843 + *726) + "726 13. ( M87 + -851 ) + "487

[More exercises are in Part D, Supplementary Exercises .]


[1-15]

V/e have now introduced the real nunaber 0, and have extended the
meaning sum of two opposite real nunnbers
of 'addition' so that the
is aixi that the sunn of an ordered pair of real nunabers, one
component of which is 0, is the other. We can now properly speak
of addition as an operation because each ordered pair of real
numbers has a unique sum which is itself a real number. The
fact that the cum of a real number and a real number is a real
number is sometimes expressed by saying that the set of real
numbers is closed under addition. [The fact that we use the
expression the sum' indicates our belief that no pair of real
*

numbers has two sums. This is confusingly expressed by 'equal


numbers added to equal numbers give equal numbers'. Of course,
there are no such things as equal numbers.]

Answers for Part A.

1. 2. 3. 4. M 5. *3

6. "3 7. -3 8. ^50 9. "20 10. "9

11. *19 12. -843 3. -851

[Some students may not use or see short cuts in doing Exercises
10 - 13. They will have airvple opportunity to discover such short
cuts later; it is not necessary to give more exercises than those
in Part A on page 1-14 and in Part D of the Supplementary Exercises
at this time. ]

TC[1-14]
[1-14]
[1-15]
[1-14]

1/37

Fill in the blanks to nnake true sentences.

1. *6 + -9 = 2. -5 + = ^7 3. *4 + = "8

4. + = *7 5. + "8 = 6. "11 + = "10

7. (*9 + "2) + _= "15 8. ("3 + ) + *9 = '20

Multiple -choice. Draw a loop around the correct answer.


1. What is the sum of a positive number and a positive number?
(A) a positive number (B) (C) a negative number

If the sum of a real number and a real number is a positive


number and one of them is a negative number, what is the
other?
(A) a positive number (B) (C) a negative number
If the sum of a nonpositive real number and a nonpositive real
number is neither a positive number nor a negative number
then each of them must be .

(A) a positive number (B) (C) a negative number

Answers for quiz •

I. 1. *3 2. "11 3. *2 4. *70 5. "4

6. 7. "2 8. "19 9. *1.5 10. *2.8

II. 1.
"3 2. *12 3. "12 *8
5.

6. n 7. "22 8. -26

III. 1. a positive number 2. a positive number

TC[l-15]b
[1-15]
1/36

Answers for Part B.

1.
"8

6. *15 7. 8. "171 10 ^10

11. "31 12. a numeral for any number in the first blank, and
a copy of this nunneral in the second blank.

[One purpose of Exercises 13-20 is to drive home the point that one
obtains a numeral for a real number whenever one prefixes a ' 'or
a*~'to any numeral for a number of arithmetic. For example [Exer-
cise 13], '*(7 + 5)' is a numeral for the real number ^12, and consists
of the numeral *(7 + 5)' for the number 12 of arithmetic together with
a prefixed *'. Note also that '*5' is not a correct answer for Exer-
'

cise 13. [Seepage 1-108.]]


13. 14. 2 15. ~3 16. 2 17. 4
18. 19. 20. 21. "937

Answers for Part C.

[In discussing these exercises, no special attention should be given


to the word 'subset'. If a student asks what a subset is, you may
tell him it is a set made up of things which belong to the other set.
We doubt that any student will raise this question.]
1. The set of nonnegative real numbers is the set which consists of
the positive numbers and zero.
2. The set of nonpositive real numbers is the set which consists of
the negative numbers and zero.
3. Yes, 0. [Note well that is neither positive nor negative, and
that 'positive' is not a synonym for 'nonnegative'!]

Here is a quiz which covers the ideas of addition of real numbers.

I. Simplify.

1- *5 + "2 2. "4 + -7 3. "6 + *8 4. *50 + *20


5. *8 + "12 6. "9 + *9 7. % + "8 8. "20 + *l

9. "5.3 + *6.8 10. *21. 3 + "18. 5

TC[l-15]a
[1.02] [1-15]

B^. Fill in the blanks to make true sentences.

1. *8 + =0 2. "3+ ="3

3. "3+ =^6 4. "3 4 ="6

5. 0+ =0 6. + *2 = M7

7. "3+ = ""3 8. + ^71 = "100

9, + M = ~1 10. + ^3 = M3

11. + *52 = *52 + "31 12. *87 + = + *87

13. ^7 + *5 = *(7 + ) 14. -2+"ll = -{ +11)

15. *8 + = *(8 - 3) 16. "(15 - 2) = "15 + ;;^

17. *4 + "7 = "(7 -


) 18. "(6 - 3) = *3 +

19. "(8 - 4) = + "4 20. "[(5 + 2) - 3] = ("5 + "2) +

21. (^351 + ) + "284 = ^351 + ("937 + "284)

C^. You have learned about three kinds of real numbers- -positive real
numbers, negative real numbers, aund 0. Each real number is

either positive, negative, or 0. And no real number is of two of


these kinds. So, the real numbers can be classified into three
subsets:

the set consisting of the positive real nunnbers,


the set consisting of the negative real numbers, and
the set consisting of the real number 0.

1. Another way of classifying the real numbers is to note that


each real number is either a negative real number or a
nonnegative real number. Describe the set of nonnegative
real numbers.

2. Describe the set of nonpositive real numbers.

3. Does the set of nonnegative real numbers have any numbers


in common with the set of nonpositive real numbers?
[1-16] [1.02]

EXPLORATION EXERCISES
A. Suppose that a pump fills a tank with water at a rate of 3 gallons

per minute. V/hat will be the increase (gallons) in the volume


of water in the taJik

1. 1 minute from now? 2. 4 nninutes fronn now?

3. IOt- minutes from now? 4. minutes from now?

B^. Suppose that the tank is full and the pump empties the tank at a
rate of 4 gallons per minute. V/hat will be the decrease in the
volume of water in the tank

1. 1 minute fronn now? 2. 4 minutes from now?

3, IOt rninutes from now? 4. nninutes from now?

C^. Suppose the pump fills the tank at a rate of 5 gallons per minute.
How nnany fewer gallons of water were there in the tank

1. 1 minute ago ? 2. 4 minutes ago?

3. 10 J minutes ago? 4. minutes ago?

D. Suppose a full tank is emptied by a pump at a rate of 3 gallons

per minute. How many more gallons of water were there in


the tank

1. 1 minute ago? 2. 4 minutes ago 9

3. 10- minutes ago? 4. minutes ago?


[1-17]

These Exploration Exercises prepare the student for a physical


interpretation of multiplication of real numbers. As in the case
of addition of real nunrxbers, students come to understand the opera-
tion of multiplication through a physical interpretation. The inter-
pretation is so designed as to lead to con^puting rules which are in
accord with accepted procedures for multiplication of real numbers.
Since this is basically a question of definition, the physical inter-
pretation must involve a certain elenrient of arbitrariness. NeverthC'
less, we have tried to make the interpretation as reasonable as
possible.
^1,
'c

Answers for Exploration Exercises. [Solutions are numbers of


arithmetic. ]

A. 1. 3 2. 12 3. 4.
^'t
B. 1. 4 2. 16 3. 42 4.

C. 20 4.
1. 5 2. 3.
"i
D. 1. 3 2. 12 3. ^4 4.

[in doing these problems it is interesting to speculate on the capacity


of the tank. The miniinum capacity is 52-=- gallons.]

Recall our convention, mentioned on TC[l-8]b, according to which


the numeral to the right of a X names the multiplier. It is still
'
'

permissible to read ''''4 X ~2' as 'positive four times negative two*,


but you should fairly frequently use the more appropriate phrases,
'the product of positive four by negative two', or 'positive four
multiplied by negative two'. [A glance at TC[1-51] will show you
the importance of establishing, before reaching that point, the
convention of "writing the multiplier on the right". ]

TC[1-16, 17]
[1-16]
[1.03] [1-17]

1 . 03 Multiplication of real numbers. --In telling what we mean by


addition of real numbers, we gave, in terms of trips, interpreta-
tions of numerals such as
-4 + -7', • -8 + *9', "3 + no', and ' '5 + 'IT,

In order to explain multiplication of real numbers, we shall give an


interpretation of numerals such as

'*4X*3', '"8X^7', '2X-5', and 'Mx'Z'.


This interpretation should help us find the product of each pair of
real numbers
We know that real numbers are numbers which can be used
to measure trips. What are the characteristics of trips which
make this possible? A trip involves

(1) a change in position by a certain amount,

and (2) a change in position in one of two opposite directions.

In general, anything which involves an amount and one of two


opposite directions can be measured by a real number. So, in
looking for an interpretation of a numeral such as

' "4 X *3',

we look for something which involves both an eunount and a direction


so that it can be measured by a real number.

A PUMP, A TANK, AND A MOVIE


Think of a pump which can pump water either into or out of a
tank, and a camera which takes a movie of the tank while the pump
is operating. Suppose the pump and camera are turned on together,

Water

Movie Camera Glass Tank Pump


[1-18] [1.03]

and that 4 gallons of water flow through the pipe each minute.
After the pump and camera have run for 3 minutes, they are
stopped. The film is then developed and projected on a screen.
What change in the water -volume do you observe on the screen?
It is easy to predict that the change observed on the screen will
be a change of IZ gallons. But, will it be an increase? In order
to answer this question, you need to know two more things.

One of the things you need to know is whether the flow


of water was into the tank or out of the tank. Suppose that the
flow was into the tank. Will the picture you see on the screen
show an increase in water -volume ? If your answer is 'yes' then

you are probably assuming that the film is being run forward
through the projector. But suppose the filni were run backward
through the projector. [Have you ever watched a comedy film
in which a man seems to dive up out of the water and land on a
diving board, or a film of a race horse running backward on a
muddy track, picking up its footprints as it goes?] If the filna
were run backward, what change in water -volunne would you see
on the screen ?

Now, suppose the water was being pumped out of the tank
while the picture v;as being taken. If the film were run forward
through the projector, what change in water- volunne would you
observe on the screen? If the film were run backward, what
change would you see on the screen?
So, in order to predict what change you will observe on the
screen, you need to know

(a) the amount of water per minute being pumped into or


out of the tank, and

(b) the number of minutes the film is being run forward or


backward.

Each of these things involves an amount and a direction, and


therefore can be measured by real nun^bers. We can use real
numbers to measure the rate at which the water is being pumped,
[1-19]

1/39

When explaining the material on page 1-18, you should be as


dramatic as possible. Ask the students to imagine a pump
^which pumpa water into the tank, and to think of what happens
to the water level as the punap operates. The kids will tell
you that the water level "goes up". .' ^ they say this, you
should move your hands to pantomime this movement upward
from the bottom toward the top of this imaginary tanlt. Next,
ask them to imagine that someone takes a motion picture of
this tank-filling process. Now ask:

(a) If this film is projected in the normal way, what


would you see on the screen?
and: (b) Ifthis film is projected backward, what would you
see on the screen?
As these questions are being answered, you should again
pantomime the observed changes.

The students need an accurate mental picture of these events.


It should not be difficult for them to acquire one, if you take
a bit of time at the beginning to make the notions clear.

TCtl-18, 19]
[1-18]
[1.03] [1-19]

deciding to use

positive numbers when water flows into the tank,

and
negative numbers when water flows out of the tank.

Thus, if 4 gallons of water are being pumped into the tank each

minute, we say that the rate is *4 gallons per minute. Explain


what is meant by saying that the rate is "4 gallons per minute.

Also, we can use real numbers to measure how long the film
is being projected, deciding to use

positive nunnbers when the film is run forward ,

and
ne gative numbers when the film is run backward .

So, if the film is running for ^3 minutes, we know it is being pro-


jected forward (normally) for 3 minutes. Explain what is meant
by saying that the film is running for "3 minutes.
Now, how does all of this help us in interpreting a numeral
such as
'"4 X *3'?

We can think of "4 as measuring the rate at which water is being

pumped. We can think of *3 as telling us how long the film is being


projected. And, finally, we can think of the product

-4 X ^3

as nneasuring the change in water -volume which we see on the screen.


[Let's agree to use positive nunnbers to measure observed increases,
and negative numbers to measure observed decreases.] In this case,
since water is being pumped out of the tank at 4 gallons per minute
and the film is being run forward for 3 minutes, the change in water
volume observed on the screen is a decrease of 12 gallons. So,

the change is "12 gallons. Since we agreed to think of the product


as a measure of the change, we can say that

-4 X +3 = "12.
[1-20] [1.03]

Let's take another case :

*9 X "8 = ?

This number, *9 X ~8, measures the change in water -volunne which

you would observe on a screen. The first nunraber, *9, measures


the rate of pumping [is it filling or is it ennptying?], and the second

number, "8, tells how long the filnn is being projected [is it being
run forward or is it being run backward?]. Does *9 X "0 measure
an observed increase in water -volume or an observed decrease?
Since a backward projection of the movie of a tank being filled shows
a decrease in water -volume, we can say that

*9 X -8 := -72.

EXERCISES
A. The table below contains problenns dealing with the pump-tank-
movie interpretation. From each problem you can learn how to
multiply a pair of real numbers. We have solved the first
problem for you as a sample.
In this problem you are told that a punno is filling the tank

at the rate of 4 gallons per nriinute. Therefore, a ' *4' is written


in the column headed 'Pump'. You learn from the second column
that the nnovie has been run backward for Z minutes. Therefore,
you write a "2' in this column. Now, we ask about the change

in water -volume that would be observed on the screen. Since


the pump is filling the tank (as indicated by the '''4'), and since
the film is run backward (as indicated by the '~2' ), the volume
of water appears to be decreasing. So, we observe un tne screen
a decrease in volunne of 8 gallons. The number "8 measures
this observed change. Finally, we write the corresponding
multiplication statement in the last row.
[1-21]

»=>ge
[1-20]

1/40

Answers for Part A [on pages 1-21 and 1-22].

2. decrease of 8 gallons; "4, *2, ~8; "4 X *2 = "8.

3. increase of 8 gallons; *4. *Z, *8; ^4 X *2 = *8.

4. increase of 8 gallons; "4, "2, *8; "4 X "2 = *8.

5. increase of 24 gallons; *8, ^^3. *24; *8 X *3 = *24.

6. decrease of 24 gallons; "8, "^3, "24; 8 X *3 = "24.

7. increase of 24 gallons; "8, "3, *24; "8 X "3 = ""24.

8. Emptying 5 gallons per minute. Running backward 6 minutes,


increase of 30 gallons; *30; *30,

9. Filling 7 gallons per minute, Running backward 3 minutes,


decrease of 21. gallons; "21; "21.

10. Ennptying 8 gallons per minute. Not running,


no change ;0; 0.

11. Emptying 6y gallons per minute. Running backward 4 minutes,


increase of 26 gallons; *26; *26.

By the time the students have worked the exercises in Part A on


pages 1-21 and 1-22, they should have formulated their own rules
for finding products of real numbers. Remember, if they have not
formulated such rules, they should continue to use the pump-tank-
film interpretation. As in the case of addition of real numbers, no
student wants to continue using the long procedure, when he feels
sure there must be a short cut for it. Again, it is the student's job
to find the short cut.

'I*

As in the case of addition, the physical interpretation which leads to


the rules for multiplying real nunnbers does not prove that the rules
are correct. The rules thennselves are consequences of the defini-
tions of real numbers and the operations, and these definitions are
implicit in the physical interpretation.

-I"

Answers for Part B are on page TC[l-23].

TC[1-21, 22]
[1.03] [1-21]

Complete the table

Observed Change
Pump Movie in Volume

Filling Running backward


4 gal. per minute Z minutes 8 j^jlj^^

u
^

Corresponding multiplication statenaent: ""4 a'oZ ---^

Emptying Running forward


4 gal. per minute 2 minutes

Corresponding multiplication statement:

Filling Running forward


4 gal. per minute 2 minutes

Corresponding multiplication statement:

Emptying Running backward


4 gal. per minute 2 minutes

Corresponding nnultiplication statement:

Filling Running forward


8 gal. per minute 3 minutes

Corresponding multiplication statement:

Emptying Running forward


8 gal. per minute 3 minutes

Corresponding multiplication statement:

Emptying Running backward


8 gal. per minute 3 minutes

Corresponding multiplication statement:


[1-22] [1.03]

[Note : In the rest of the problems you are given real numbers
and you should fill in the corresponding blanks. ]

Observed Change
Pump Movie in Volume

8.

"5 6
Corresponding multiplication statement: "5 X "6 =

9.

*7 "3

Corresponding multiplication statement: *7 X "3 =

10.

"8

Corresponding multiplication statement : "8X0 =

11.

-^\ "4

Corresponding multiplication statement: "6y X "4 =

B^. Simplify. Use the pump-tank-film interpretation as long as you


need to, but try to find a short cut.

1. *5 X ^2 2. % X -^3
3. *8-| X *8

4. ^ X *6 5. % X "2 6. "2 X *6

7. "5 X *7 8. *S X "0 9. "9 X no


10. "12 X "10 11. "7 X "8 12. '15 X "3

13. "1 X "1 14. "8 X "12 15. '7X0


[1-23]
[1-22]

1/42

Here is a quiz which covers the ideas of multiplication and addition


of real numbers.

Simplify, [Be careful not to confuse addition signs with multi-


plication signs, ]
1. *3 X ? 2, *4 X *2 3, -5 X -3 4. "6X0
"
5. *3 + "7 6. '2 X 8 7. -5 + "3 8. + "6

Fill in the blanks to make true sentences.


1. "7 X = "28 2. *4 X = "8 3. X -5 = *5

4. X-9 = '9 5. "3 X = 6. _ + -5 = *5


7. {*3 + '5) X *4 = 8. ("6 + no) X = "12

9. {"7 + *2) X = *30 10. {^3 + ) X-7 = ^42

Multiple -choice. Draw a loop around the correct answer.


1. The product of a positive numb er by a negative number
is

(A) a positive number (B) (C) a negative nunaber


If the product of a real number by a real number is a positive
number and one of them is a negative number, what is the
other?
(A) a positive number (B) (C) a negative number
If the sum of two real numbers is 0, what is their product?
(A) a positive number (B) (C) a negative number
«.!..
'!"•

Answers for quiz.

I. 1. -21 2. *8 3. ns 4.

5. -4 6. ne "8 -6
7. 8.

*4
II. 1. 2. z 3. "1 4. n 5.

6. no 7. "8
8, -3 9. "6 10.

III. 1. a negative number a negative number


3. a negative nunnber

TC[l-23]b
[1-23J

\
[1-22]

1/41

Answers for Part B [on pages 1-22 and 1-23].

1. MO 2. M8 3. *68 4. *56 5. "12

6. "12 7. "35 8. "64 9. "90 10. "120

11. *56 12. *45 13. M 14. *96 15.

16. 17. 18. 19. *4 20. "250


- 3
21. ZZ. "45 23. "84 24. *24 25. "24
2

26. *6 27. "34 2G. "2726 29. "1755 30, * 10575


"
31. *6450 32. 3.3123 33. -72. 182

34. *24 35. *42 36. "60 37. ^36 38. *84

39. *60 40. 41.

Answers for Part C.

1. "14 2. "18 3. "29 4. "3 5. M


6. *2 7. -3 8. "1 9. -22 10. "53

11. M20 12. "64

''I'*

Answers for Part D.

1. "4 2. "6 3. *3 4. *3 5. *12


+
6. "3 7. "16 8. -1 9. 10. -3

11. 12. "2 13. 14.

'I*

TC[l-23]a
[1.03] [1-23]

16. OX" S 17. 0X0 18. "12 X


"1 +5
19. 16 X 20. 100 X J 21. •^'^
4 TO
22. *3 X '15
-
23. -7 X *IZ 24. "3 X "8

25. ^6 X 4 26. -3 X "2 27. "17 X *Z

28. *47 X "58 29. '27 X "65 30. '705 X '15

31. "86 X "75 32. "1.83 X "l .81 33. '9. 65 X -7. 48

34. (^2 X "3) X 4 35. ('2 X '7) X -3

36. {^5 X •3) X M 37. {^6 X -2) X -3

30. M X (
•3 X 7) 39. "6 X (-2 X "5)

40. (^73 X -81) X 41. (^5 X "17 ) X ("3 X 0)

[M(Dre exercises are in Part E, Suppleni 32itary Exercisec. ]

C. Simplify. [Be careful not to confuse addition signs with multi-


plication signs.]

1. {'5 + "3) X "7 2. ('3 X -4) + "6

3. (*S X "3) + "5 4. ('12 + "11) X "3

5. ('1 + '1) + *1 6. ('1 X '1) + -1

7. (
"1 + "1) + "1 8. (
-1 X "1) X "1

9. (
"4 X "2) + ("5 X '6) 10. ("3 X '7) + ('8 x '4)

11. ('71 + "11) X ('6 + "4) 12. ("8 + 0) X ('4 X '2)

D. Fill in the blanks to make true sentences.

1. '5 X = -20 2. "3 X = '18

3. '7 X = '21 4. 7 X = "21

5. + -3 = '9 6. X -3 = '9

= "8
7. *8 + = "8 8. '8 X — —

9. '3 X - '1 0. '3 + -


_

11. '3 X = 2. '3 + = *1

13. '8 X "2 = -(8 X 14. -5 X = '(5 X 2)


[1-24] [1.03]

EXPLORATION EXERCISES
Consider the table of pairs of numbers at

the right. One of the interesting features 89 . .


1

of this table is that you can carry out some 178 . .

computations with the numbers listed in 267 . .


3
one column by doing computations with the 356 . .
4-

corresponding numbers listed in the other 445 . .


5
column. For example, suppose you want 534 . .
6
to find the sum of, say, 267 and 445, two 623 . .
7
numbers listed in the lefthand column. 712 . .
s
To simplify 801 . .
^
'267 + 445' 890 . .
10
nnerely simplify 979 . .
II
'3 + 5'. 1068 . .
/a
3 and 5 correspond with 267 and 445, 1157 . .
15
respectively. '3 + 5' simplifies to '8', 1246 . .
14-

and 8 corresponds with 712. And we 1335 . .


/5
find that 1424 . .
Ih
267 + 445 = 712. 1513 . .
11
Here is another example. 1602 . .
IS
1691 . .
/9
534 6
1780 . .
XO
801 _9_
15

15 corresponds with 1335, and it turns


out that 534 + 801 = 1335.
Use the table and the illustrated procedure to simplify each
of the following. Check your results by carrying out the simpli-
fication directly.

1. 356 + 534 2. 712 + 979 3. 1068 + 267

4. 445 + 1157 5. 1424 - 1068 6. 1157 - 178

7. Is It possible to multiply pairs of numbers listed in the left-


hand column by multiplying the corresponding numbers listed
in the righthand column? Try simplifying '178 X 5 34' that way.
[1-25]

n
1/43

The purpose of these Exploration Exercises is to get the students to


beconne aware of the concept of isomorphism . We >want the student
to have this awareness as he reads pages 1-29 through 1-32, We
think that if he went immediately to this discussion, it would be quite
meaningless for him. In the discussion which starts on page 1-29 we
are trying to get across the point that the system of nunnibers of arith-
metic and the system of nonnegative real nunnbers are different sys-
tems which have the same structure. By 'system of numbers' we
mean a set of things which we call 'numbers' together with one or
more operations. Thus, when we talk about the system of numbers
of arithmetic in Section 1.04, we mean the set of numbers of arith-
metic together with the operations of addition and niultiplication (of
numbers of arithmetic). Also, the systena of nonnegative real nunn-
bers is the set of nonnegative real numbers together with the opera-
tions of addition and multiplication (of nonnegative real nunnbers).
To say that one of these nunnber systems is isomorphic to the other
nneans that there is a one-to-one correspondence between the num-
bers in one system and the numbers in the other such that the sum
(product) of two numbers in one system is the "mate" of the sum
(product) of the mates in the other system.

Let us consider the example in Part A. Suppose we take as one num-


ber system the set of whole numbers 1, 2, 3, ..[only some of these
. .

numbers are listed in the right-hand colunnn] together with the opera-
tion of addition of these numbers. The second number systenn consists
of the set of numbers 89, 178, 267, ,,,, together with the operation of
addition of these numbers. Then, these two nunnber systems are
isomorphic to each other. The numbers in the right-hand column act
like the numibers in the left-hand column when you add them. But,
the fact that these two systems of numbers have the same structure
is hardly sufficient reason to say that the nunnber s in one system are
the same as the numbers in the other system. Similarly, since the
system of the numbers of arithmetic is isomorphic to the system of
nonnegative real numbers, the two systems have the same structure;
but this is no reason for saying that the numbers in one system are
the same as the numbers in the other. The fact that two number
systems can have the sanne structure without anyone wanting to claim
that the numbers are the same is one of the points we regard as
most important in these exercises.

TC[l-24, 25]a
[1-24]
[1-25]

1/44

Exercise 7 of Part A points out that even though the two sets of
numbers act like each other with respect to addition, they do not
act like each other with respect to nnultiplication. If we call the
numbers listed in the right-hand column and all others like them
'set A' and those in the left-hand column 'set B', then we would
say that the system consisting of set A together with addition is
isomorphic to the systena consisting of set B together with addition,
but the system consisting of set A together with nnultiplication is
not isomorphic to the system consisting of set B together with
multiplication.

Ans wars for Part A.

1. 890 2. 1691 3. 1335 1602

5. 356 6. 979 7. No.


vl>

Answers for Part B [on page 1 -25].

.034290 40 4. .041148
1. 2.
-i 3.
1
5. 6. .068580
8>f

We suggest that pages 1-24 through 1-28 be covered in not more than
one class period plus one honnework assignment.

TC[l-24, 26]b
[1-24]
[1.03] [1-25]

B^. Here is another table in which

you can add some pairs of numbers .030861 . . ..36|


listed in one column by adding the .013716 . .
..6i
corresponding numbers of the other .041148 . . . . 49

column. For example : .020574 . .


..^4i

.027432 32-i
.006858 . .
••
4
.041148 49 .024003 . .

8>| .037719 . .

81 -^^ corresponds with .068580. So, .027432 . . ..3Z§


.068580 is the sum of .027432 and .0 34290 . . ..40|
.041148. [Check this by adding. ]
.068580 . . ..3,|

Use the table to simplify each of the following. Check your

results by direct simplification. [Note that Exercise 2 requires

that you work with numbers listed in the lefthand column. ]

1. .013716 + .020574 2. 8^ + l6-j

3. 16-j + 24-| 4. .020574 + .020574

5. 40 1 + 40 I
6. .030861 + .037719
[1-26] [1.03]

Here is a table which can be used


to find products of some pairs of .5 . . 2

numbers listed in one column by .25 . . 4

computing products of the corre- .2 . . 5

sponding nunnbers of the other .125 .. . . 8

column. For example : .1 . . 10

.0625 .. .. 16
.05 . . 20
25 _4
8 .04 . . 25
.03125 . . . . 32
8 corresponds with . 125, and
.025 .. . . 40
. 125 is the product of . 5 and . 25.

Use the table to simplify each of the following, and check


by direct simplification.

1. . 5 X . 2 2. 8 X 5 3. .0625 x . 5

4. 8X4 5. . 125 X .25 6. .2 X . 2

'I"

Can you use this table for addition?

D. Here is another table which can be used for multiplication.

*2 ^5
^2
M
*5 no *20
3 5 9 3 9 27 81

1 1

2 5
3 5 9
4 1
4 2.7 4.05

Use the table to find the products, and check by direct


simplification.

1
-i , *1
^X^2 3. 9x1
i X '10
4. ^5 lix 2.7 6
^-
'2 X
"^
3 27
[1-27]

1/45

Part C dealo with two sets of numbers which act like each other
with respect to multiplication but not with respect to addition.

'i-

Answers for Part C.

1. .1 2. 40 3. .03125 4. 32
5. .03125 6. .04 7. No.

Answers for Part D.


* 2 4
l.l'^ 2.
3
-J
3.1^
3 ^5.
.
4. ^
*20
5. 4.05 6.
81

>\<

Part E emphasizes the fact that an essential itena in establishing


an isomorphism between two systenas is that the numbers in the
systenns be placed in one-to-one correspondence. The "weird"
arrangennent in Part E points out that a two -column table is nothing
nnore than a listing of a set of ordered pairs. The individual pairs
of cells show the pairs, and the difference in shading shows the
order. The table in Part E works for addition.

TC[l-26. 27)
[1-26]

C.
[1.03] [1-27]

E. Here is a table which can be used just as you used the other
tables. Figure out how to use it, and see if it works for multi-
plication or for addition.

+8 "42

-;;x;::^.;::.;;' .•::;:.
•^

'
3
(:+L5 ^
4
<
'i'-'Si^^^i-^

"3

ww^m^:
+ 3
-ill
10

u '21
^ »
[1-2SJ [1.03]

For each table, see if it can be used for finding sums or products
of some pairs of numbers listed in one row by computing sums
or products of the corresponding numbers listed in the other

row.

4 9 16 25 36 64 100 144 225 324 400 576

*2 *3 *4 *5 *6 *8 MO ^2 ns ns *20 *24

^3
2.
"3 '2 "1
4 2 4
n *Z ^3 ^4 ^6 *8

*9 *6

1
*3
"3
4 ^2 4 "3 "6 '9 '12 "18 "24

-1
3. 1 3 4 5 6 7 8 9 12 18 35 54

1 2 3 4 5 6 7 8 9 12 18 35 54

4. 1 2 4 8 16 32 64 128 256 1024 2048 4096

1 2 3 4 5 6 7 8 10 11 12

-1 "2 "3 "4 "5 -6 "7 -8 -10 "12 -13 "20 -42

1 2 3 4 5 6 7 8 10 12 13 20 42

*1 *2 *3 "4 ^5 *6 *7 *8 no ^2 n3 *20 M2

1 2 3 4 5 6 7 8 10 12 13 20 42
[1-29]

'6
1/46

The exercises in Part F have been carefully selected to lead up to


the climax in Exercise 6. Exercise 1 shows that the system
consisting of the "perfect square" nunnbers of arithnnetic together
with the operation of multiplication is isomorphic to the system
consisting of the nonnegative integers together with multiplication.
Exercise 2 shows that the systena consisting of real numbers
together with the operation of addition is isomorphic to the system
consisting of the real multiples of ~3 together with the operation of
addition.

By the time students reach Exercise 3 they are quite eager to find
a table which "works" for two operations. Exercise 3 provides
such an example. But, as naany students will point out, the example
is a trivial one. What the example does show is that every number
system is isomorphic to itself.

Exercise 4 provides an example of an isomorphism between one


number system which consists of the integral powers of 2 together
with the operation of multiplication and a second number systenn
which consists of the whole numbers of arithmetic together with
the operation of addition. The fact that the operations are named
by different words has nothing to do with the fact that the structure
of the systems is the same. You will notice, of course, that this
is the kind of isomorphism which is used in carrying out computa-
tions by means of logarithms.

In Exercise 5 we have an isomorphism between two systenns in


which one systenn consists of the set of nonpositive real numbers
together with addition, and the other system consists of the
nuinbers of arithmetic together with addition. If we check this
table, we find that the isomorphism breaks down if we try to
bring multiplication into the systems. By this time, students
should understand that although the nonpositive real numbers act
like the numbers
of arithmetic with respect to addition, this is
no reason say that the nonpositive numbers are the same as
to
the numbers of arithmetic.

Finally, in Exercise 6 we find the case in which the isomorphic


systems include both multiplication and addition. And, as we have
noted repeatedly, although the structure of the two systems is the
same, the elements of the systems are not the same.

On page 1-30 you may want to tell the students that mathematicians
use the phrase 'are isomorphic to' instead of 'act like'.

TC[l-28]
[1-28]

F.
[1.04] [1-29]

1.04 Numbers of arithmetic and real numbers . --Think carefully


about what you do when you compute the sum of a pair of nonnegative
or a pair of nonpositive real nunabers. For example, what do you
think about when you do the following problems?

*7 + nz = ? and 7 + 'IZ = ?

Most likely, you first simplify:

7 + 12

and get 19,

and then you say that

*7 + *12 = ^9 and "7 + "12 19.

When you do thi£:^ you no doubt imagine a table which links up each
nonpositive nunnber and each nonnegative number with a number of

arithmetic.

. . . . . . . . .

"1 1

2 • • •
2 • • • •
2

1 . . . . 1 . . . . . n
-.i.. ...i.. •4
2 . . . . 2 . . . . .
^2

"3 . . . . 3 . . . . .
^3
r

"4 . . . . 4 . . . . M
5 . . . . 5 . . . . "5

'6 . . . . 6 . . . .
"6
"

[1-30] [1.04]

In this table, each real number corresponds with that number of

arithmetic which gives the distance part of a trip measured by the

real number. Thus, in order to find sums of pairs of nonpositive


or nonnegative real numbers, you should begin by finding the arith-
metic numbers which correspond with the real numbers, and then
add the arithmetic numbers. We can summarize these remarks by
saying that

the nonpositive real numbers and


the nonnegative real numbers act
like the numbers of arithmetic
with respect to addition.

You probably discovered this idea when you were looking for short
cuts in computing sunns.
What about multiplication? Do the nonpositive real numbers
act like the corresponding numbers of arithmetic with respect to
multiplication? A quick check of the table above shows that they
do not.

"2 2
X X
-3 3

6 corresponds with "6 but "2 X "3 / "6.

Do the nonnegative real numbers act like the corresponding


numbers of arithmetic with respect to multiplication? The answer
xs 'yes'. So,

the nonnegative realnumbers act


like the numbers of arithmetic with
respect to both addition and multi-
plication.
[1.04] [1-31]

SHORTER NAMES FOR POSITIVE NUMBERS

In view of the fact that the nonnegative real numbers act like
the numbers of arithmetic with respect to both addition and multi-
plication, it will cause no trouble if we use the names of numbers
of arithmetic as names of the nonnegative real numbers. When we
want to state a fact about real numbers, for example, that

(1) *9 + -3 = %,

we can just write :

(2) 9 + '3 = 6.

Anyone who looks at sentence (2) and believes that it should make
sense must conclude that '9' and '6' are numerals for positive
numbers rather than numerals for nunnbers of arithmetic. [He
would conclude this because it wouldn't nriake sense to add a nega-
tive real number to a number of arithmetic] So, when you look
at (2) you "see" it as (1). And writing (2) instead of (1) saves you
the irrouble of writing the little plus signs.
Consider another example.
(3) 7X4 = 28.

Is this a statement about numbers of arithmetic or is it a state-

ment about real numbers? Unless you kiiow the problem which
led to someone's writing (3), you are free to interpret it either
w^y..

Numerals such as those in sentence (3) which name more than


one number are said to be ambiguous . V/e have already seen an
'0' names number of
ambiguous numeral in the case of '0'. the

arithmetic and it names the real number 0. We are now preparing


to deal withmany more cases of ambiguous numerals.
Ambiguous words or names may cause confusion. For example,
suppose there are two students in your class each having the name
'Ann Brown'. If this message is sent from the principal's office:

Ann Brown is to report to the principal's


office at 3:30 for a conference
[1-32] [1.04]

there is likely to be confusion since 'Ann Brown' refers to either


of these students, and the teacher would not know which student
should get the message. On the other hand, suppose the following

message is sent :

Ann Brown is to report to the principal's office


at 3:30 for a conference with the other fresh-
men representatives to the Student Council.

It is likely that the ambiguity of 'Ann Brown' in this message would


cause no trouble since the rest of the message makes clear which
Ann Brown is intended.
Similarly, although the '9' in each of the sentences:
(4) 9 X 4 = 36

and :

(5) 9 X "7 = -63

is ambiguous, we know from the rest of sentence (5) that it refers


to the real number *9 and not to the number 9 of arithmetic.

EXERCISES
A. Each of the following sentences contains at least one ambiguous
numeral. In which sentences are you unable to tell whether
real numbers or numbers of arithmetic are intended?

1. 2 X -3 - "6 2. 8 + 4 = 3 + 9

3. 5 + -5 = 4. 4 X 7 = 14 X *-2

5. 8X3 = 6X4 6. 10 X 5 = -25 X "2

B. Simplify.

1. 3 X -4 2. 7X5 3. "8 X (6 + 4)

4. 2 X -5 5. 15 X "3
] 6. "2 X -7

7. 3 + -4 8. 7 + '5
- "15 + 12
9.

10. (17 + -8) + 3 II. ("21 + 5) + -5

12. ("53 X 6) X i 13. {-751 X 7) X "i


[1-33]

1/47

There is a very subtle issue which arose during one of our summer
conferences in connection with Exercise 6 of Part A. There is a
possibility that it will be brought up by some of your very perceptive
students. The issue is this. Although the numeral '10 X 5* must
stand for a real number in order that the sentence nnake sense, it
is not nececsarily the case that the component nunnerals '10' and
'5' must stand for real numbers. They could stand for numbers
of arithmetic in which case the nunneral '10 X 5' stands for a number
of arithmetic. Then, by our convention concerning the abbreviation
of numerals for positive real numbers, we could regard such a
numeral ac an abbreviation for '*(10 X 5)'. The ambiguity in this
problem centers around the multiplication sign. As pointed out
earlier, the operation of multiplication of real numbers is different
from the operation of multiplication of numbers of arithmetic. Yet,
the same sign is used to denote both of these operations. If
different signs were used, there would be no ambiguity. For
example, if 'X 'denotes the operation of multiplication of numbers
of arithmetic, and 'X denotes the operation of multiplication of
'

real numbers then in


'
10 X^ 5
'

the numerals '3 0' and '5' stand for numbers of arithmetic, whereas
the num.erals '10' and '5' in
'
10 X„ 5'
K.
stand for real numbers. [Similar remarks apply to Exercise 4.]

Answers for Part A.

In Exercises 2 and 5 you cannot tell whether real numbers or


numbers of arithmetic are intended.

Answers for Part B.

1. "12 2.
'"35 3. "80 4. 10[or:*10]
5. "45 6. 14[or:*14l 7. "1 8. 2 [or : *2]

9. '3 10. 12[or:n2] 11. "21 12. "53

13. 751 [or: *75l]


[Do not make an issue of the use of short cuts in Exercises 10-13.]

TC[l-32]
[1-32]

there>
[1.05] [1-33]

_C. Each of the following sentences contains ambiguous words or


phrases. Be prepared to give two interpretations for each
sentence.
1. There will be little change in ladies' pocketbooks next year.
2. Charles ran after Henry.
3. Please leave the table.
4. One of the horses was scratched.
5. This is a fine day for the race.
6. Look at the scales.

7. Mr. Blattingham gave his address.


8. What does that ring mean?
9. 10.

SLOW SAVE RAGS


Children AND
At Play WASTE PAPER

'

: _,^
ji< kli'fcja3a
^/fill..

1.05 Punctuating numerical expressions --The pictures below show .

the seat arrangeinents of two classrooms in a school building. Under


each picture someone has tried to indicate the number of seats in the

room and, at the same time, show his method of computing that number.

L_

1 I]

1 1


1 1 1
L C
1 u c n 3
_—_
J E
1 u r
1

1^ ?<5' + <2. ^ X 5" -^ ^


[1-34] [1.05]

It is easy to see that the room shown on the left has 32 seats and

that the room shown on the right has 42 seats. If the collection of

marks :

6X5 + 2

is a numeral, it would seem to be the case that this numeral stands


for 32 and for 42 ! V/ithout the pictures, the numeral '6x5 + 2'

is of little help. This is a case of ambiguity which could cause

trouble. V/hat ceui be done to remove the ambiguity?

Here is a case of an English sentence which is ambiguous:


(1) John will play and Bill will sing or Mary will sing.
This sentence could mean that

(2) Either John will play and Bill will sing or Mary will sing.

Or, it could mean that

(3) John will play and either Bill will sing or Mary will sing.

To make an English sentence unambiguous you usually have to

rewrite it and use grouping words like 'either. . . or . . .


'.

In mathematics we can remove aunbiguity from expressions by


punctuating them with parentheses. \Vc could use parentheses to

punctuate sentence (1) to show the first meaning. We would write:


(2') (John will play and Bill will sing) or Mary will sing.
To show the other meaning we would write :

(3') John will play and (Bill will sing or Mary will sing).
Similarly, we can punctuate '6x5 + 2' this way:
(6 X 5) + 2

when we mean 32, and like this :

6 X (5 + 2)

when we mean 42. [To read '(6 X 5) + 2' aloud, say 'the sum of
6x5 and 2' or 'the quantity 6X5, plus 2' or 'parenthesis 6X5
close parentheses, plus 2'. How do you read aloud '6 X (5 + 2)'?]
Here is another expression which is ambiguous:
8x3+2X5.
Give Sonne numbers which it might nanne . Here are some of the
ways in which this expression can be punctuated to make it unam-
biguous .
[1.05] [1-35]

If we want to name 34, we can write :

(8 X 3) + (2 X 5).
To name 130, we can write ;

[(8 X 3) + 2] X 5.

If we mean 200 then we should write :

[8 X (3 + 2)] X 5.

Is there another possibility?


Notice that when we want to show a grouping inside of another
grouping, we use both parentheses and brackets . [Why don't we
just use two pairs of parentheses?] Here is a punctuated expression
in which parentheses, brackets, and braces are used:
{[(7 X 2) + 6] X 5} + 7.

This expression is a name for a number. Often it is helpful to


have a simpler looking name for such a number. [For example,
if someone asked you how much your new cap cost, and you replied

that it cost {[(7 X 2) + 6] X 5} + 7 cents, he would have left before


you had finished speaking.] To find a sinapler looking expression
equivalent to
{[(7 X 2) + 6] X 5} + 7

[that is, to find a simpler expression wtiich names the sanne number],
we must find a simpler expression equivalent to ;

{f(7 X 2) + 6] X 5};

this will be easier to do if we find a simpler one equivalent to:

[(7 X 2) + 6],

and this is done by first tmding a simpler name for the number
named by
(7X2).
Hence, to simplify our original expression we would think through
the following steps to obtain equivalent expressions.

{[(7 X 2) + 6] X 5} + 7

{[ 14 + 6] X 5} + 7

{ 20 X 5} + 7

100 -I- 7

107
Most people would agree that'107'is a simpler looking name for 107

than is '{[(7 X 2) + 6] X 5} + 7'.


[1-36] [1-05]

EXERCISES

A. Simplify. [Be careful not to confuse '


+ 's with ' X 's. ]

1. (344)X2 2. 34(4X2) 3. {7 X 5) + 4

4. (8 + -3)X-2 5. -IX ("6 + 4) 6. ("1 X "6) + 4

7. (9 r 3) + 3 8. 9 ^ (3 + 3) 9. (8 4- 2) X 4

10. 8 V (2 X 4) 11. (6 X 5) V 5 12. 6 X (5 =-


5)

13. (8 + 2) + 5 14. 8 + (2 4 5) 15. (2 X 5) X 3

X "7
16. [(8 4 2) X 3] 4 5 17. [('3 4 -2) 6] 4

18. 5 4 [-1 X (8 4 -2)] 19. (5 4 -1) X (8 4 -2)

20. (8X7)4(2X7) 21. (8 4 2) X 7

22. (3 X -6) 4 (7 X -6) 23. (3 4 7)X-6

24. ("2 X "12) 4 (2 X -12) 25. ("2 4 2) X "12

26. 2 X {[(3 4 2) X 4] 4 [(5 4 9) X 4]}

27. "3 X {["2 X (8 -.


12)] 4 [(9 4 31) X "4]}

28. {[5 X (8 4 -1)] 4 3} X {"2 4 [4 X (3 4 'Z)]}

5 4 21
^^' T w 1 o [Note : When a bar is used to indicate division,
'

-.1
Italso .
acts u
as a grouping symbol. , o
So,
'
5 4 21
means
^ .
^

(5 4 21) 4 (2 X 13). ]

^n ^8 - 3 15 4 4 ^, 17 4 (6 X2)
^"- ^^- ^^-
100-50 (7X5)43 17 - {b X Z)

., [(9 X 3) 4(5 X 2)] 4 3 [(5 X6) - (4 X 2)] 4 [(3 X 4) - 3]


^^-
[8 4 (5 X 5)] - 13 2 4 {5 4 [3 4 2 X (27 - 1)])

jB. 1- Joey and Jane went to the candy store; Joey bought 5 candy
bars at 6 cents each and a 10-cent package of bubble gum.
Jane bought 6 suckers that were 5 cents each and 10 sacks
of peanuts that were also 5 cents each. What single expres-
sion could you write which (if no grouping symbols were used)
might name the cost of either Joey's or Jane's purchases?
1/48

Here is an interesting set of exercises which you nnay want to give


to the class before they work on Part A. They were suggested to us
by similar exercises found in certain European arithmetic textbooks.
The object in each exercise is to insert grouping symbols and opera-
tion signs to make true sentences.

5 4 2 = 22 2. 2 = 30

3 5 4 2 = 18 4. 5 4 2 = 13

5 5 4 2 = 10 6. 5 4 2=0. 625
7 5 4 2 = 2.5 8. 5 4 2 = 40

9 5 4 2 = 11 10. 5 4 2 = "1

11 5 4 2=3 12. 5 4 2 = "3

'1^

Answers for Part A.

1. 14 2. 11 3. 39 4. -10 5. 2

3
6. 10 7. 6 8. 9. 16 10. 1
2

11. 6 12. 6 13. 15 14. 15 15. 30

16. 35 17. "37 18. -1 19. 24 20. 70

21. 70 22. -60 23. -60 24. 25.

26. 152 27. 600 28. -76 29. 1 30.

29
31. 32. 33. 2 34. fSe TC[l-36,
[See 37]b. ]

TC[l-36, 37]a
[1-36]
[1-37]

1/49

Notice that there is an ambiguity in the denominator of the fraction


in Exercise 34. Although this was originally an oversight on our
part, it turned out to be useful in at least two of the demonstration
classes which were taught at institutes during the sunnmer of 1958.
Students should express some puzzlenaent over the exercise itself,
and if they are able to plow through the arithmetic, they should
assert that this fraction stands for either of the two numbers, r-r=-
and •=-.

Answers for Part B [on pages 1-36 and 1-37].

1. 5 X 6 + 10 2. yes, yes

[In some parts of the country, students may object to our use of
the word 'sacks' in Exercise 1, and say that we should have used
the word *bag' [or even 'poke']. In the Middle V/est the words 'sack'
and 'bag' are used interchangeably. One of our teachers (having
helped nnalce gardens in her youth) wrote us that in i.lrs, Plantin's
garden [Exercise 2] we should have had 3 hills of potato plants in
each of 7 rows. She explains that a hill of potatoes contains several
potato plants! Of course, if we said 'hills' of potato plants, our
problem might not be quite as effective. We would have to change
the question to:
Could the expression above which indicates the number of
plants set out by Mrs. Gardner also be used to indicate
the number of green pepper plants and the number of hills
of potatoes in Mrs. Plantin's garden?

and that puts a little different "twist" on it.]

TC[l-36, 37]b
[1-36]
[1.05] [1-37]

2. Mrs. Gardner and Mrs. Plantin were making vegetable


gardens. Mrs. Gardner said she had planted 3 rows of

cabbages with 7 plamts in a row, and 5 rows of tomatoes


with 3 plants in each row. Could the expression *3 X 7 +5'
represent the number of cabbage and tomato plants in

Mrs. Gardner's garden?


Her friend, Mrs. Plantin, explained that she had set
out a row containing 5 green pepper plants; also, she had
planted potatoes so that she would have 3 potato plants in
each of 7 rows. Could the expression above which indicates
the number of plants set out by Mrs. Gardner also be used
to indicate the number of green pepper euid potato plants in
Mrs. Plantin 's garden?

CONVENTIONS FOR OMITTING GROUPING SYMBOLS


You have seen how parentheses and other grouping symbols can
be used to remove ambiguity from numerical expressions. In fact,

there would never be an ambiguous expression if everyone followed


the rule that each operation sign [ +, X, -, -r ] required a pair of
grouping symbols. You would have expressions like these:

(5 + 4), [(6 - 2) -f 9], [3 X(6 X 5)],

{[(4 + I) + 3] + (6 - 5)}, {[(3 X 4) + (6 - 2)] v (12 ^ 3)}.

In the expression '{5 + 4)', the parentheses go with the ' + '.

In the expression *[3 X (6 X 5)]', the parentheses go with the second


*
X ', and the brackets go with the first *
X '.

In the expression '{[(4 + 1) + 3] + (6 - 5)}', the braces go with the

third '
+ ', the brackets go with the second *
+ ', the first pair of
parentheses goes with the first ' + ', and the second pair goes with
the * - '.

Tell which pairs of grouping symbols go with which operation signs


in the other two expressions written above. Make up three expressions
each of which contains at least two operation signs and a pair of
grouping symbols which go with each sign.
If an expression contains nnany operation signs, this rule requires
that the expression contain just as many pairs of grouping symbols.
[1-38] [1.05]

Such expressions are frequently hard to read and look quite compli-
cated. So, people follow certain agreements [or conventions l which
permit them to omit some grouping symbols and still avoid ambiguity.
Under such conventions, an expression like:

9 + 2X4-4^2
is unannbiguous. Our job now is to learn what these conventions are
so that when we come upon an expression such as the one above, there

will be no doubt what number is intended by the person who followed


our conventions in writing it.

One convention [which we have been using throughout this book] is

to omit the outermost grouping synnbols. For example, we would write

'5 + 4' as an abbreviation for '(5 + 4)',

and '(6 - 2) -f 9' as an abbreviation for '[(6 - 2) + 9]'.

This convention was followed in all of the expressions in Part A on


page 1-36. Turn to that page now and, for the first ten expressions,
tell which operation sign goes with the omitted grouping symbols.
When we use such abbreviated expressions to form larger expressions,
we often have to unabbreviate them by replacing the omitted grouping
symbols. For example, we name the product of 3 + 2 and 4 + 5 by
'(3 4 2) X (4 + 5)'. Although *
3 + 2' and '4 + 5' are not ambiguous,
'3 + 2X4 + 5'is annbiguous.

Another convention which we shall adopt is illustrated in these


examples.

'3+5 + 6' is an abbreviation for '(3 + 5) + 6'.

'5X2X7' is an abbreviation for '


(5 X 2) X 7 '.

'4 + 3 + 2+9' is an abbreviation for '[{4 + 3) + 2] + 9'

'
12 2 - 3' is an abbreviation for 3'.
- '
(12 - 2) -

18 V 3 2' is an abbreviation for 3) ^ 2


• -^ '
(18 -=-
'.

'2 + (3 + 5) + 6' is an abbreviation for '[2 + (3 + 5)] + 6'.

'2 X 3 X (5 X 6)' is an abbreviation for '(2 X 3) X (5 X 6)'.

Notice that each of the abbreviated expressions contains only one kind of
operation sign. The expressions which contain no grouping symbols are
unabbreviated by introducing a pair of grouping symbols for each operation
1/50

Here are the unabbreviated expressions which correspond with those


in Exercises 1-10 of Part A on page 1-36. Notice that each operation
sign is linked with a pair of grouping synnbols. [At the blackboard
you can use, say, yellow chalk for one pair of grouping symbols and
the corresponding operation sign, and blue chalk for the other pair
and its corresponding sign.]

1. [(3X4) + 2] 2. [3 + (4 X 2)] 3. [{7 X 5) + 4]

4. [(8 + ~3) X -2] 5. ["1 X {"6 + 4)] [(-1 X -6) + 4]

7. [{9 ^ 3) + 3] 8. [9 T (3 + 3)] 9. [{8v2)X4]

10. [8^(2X4)]

Example (4) on page 1-39 n:iay contradict conventions given in other


texts which tell you "to do all multiplications first, then divisions
next". Please follow the convention given here. The parallelism
between addition-subtraction and multiplication-division is too neat
to want to give up.

'I*

Do you see that the procedure for unabbreviating an expression is


tantamount to a procedure for deciding upon order of operations?
In the example at the bottonn of page 1-39 students should see that
each operation sign [except the last one] has a pair of grouping
symbols.

TC[l-38, 39]
[l-3fll
[1.05] [1-39]

sign, proceeding from left to right. So, we unabbreviate:

4 + 3+2+9
by first writing:

(4 + 3) + 2 + 9,

and then writing:


[(4 + 3) + 2] + 9.

[As before, we don't include the outermost grouping symbols.] As


in the step from '
{4 + 3) + 2 + 9 ' to '
[(4 + 3) + 2] + 9', when some
grouping symbols are already present in an expression to be unabbre-
viated, we bring in grouping symbols for those operation signs which
do not already have them, again proceeding from left to right.

Examples :

(1) 2 + (3 + 5) + 6 (2) 2 X 3 X (5 X 6)

i i

[2 + (3 + 5)] + 6 (2 X 3) X (5 X 6)

This same convention is used for unabbreviating expressions in

which the operation signs which lack grouping symbols are either all

addition and subtraction signs or all multiplication and division signs.

(3) 4 + 3-2+7 (4) 8 X 2 -= 4 X 3

i i
(4 + 3) - 2 +7 (8 X 2) V 4 X 3

i i

[(4 + 3) - 2] +7 [(8 X 2) -f 4] X 3

(5) 9 - (3 X2) + 4 - 5 (6) 10 T 5 ^ (2 + 2) X 2

i I
[9 - (3 X2)] + 4 - 5 (10 -^ 5) V (2 + 2) X 2

i 1

{[9 - (3 X2)] +4} - 5 [(10 -^ 5) ^ (2 + 2)] X 2

Consider this expression:

12 + [3 X (5 + 6) 4- 2] - 5.

To unabbreviate it, we first attack the expression in the brackets:

12 + [{3 X (5 + 6)} = 2] - 5.

And, now we unabbreviate this expression, getting:

{12 +[{3 X(5 + 6)} =-


2]} - 5.
[1-40] [1.05]

Here are some more examples of this.

(7) 4 X (3X2X5X3) (8) 38 + [ 3X4 V 6 X (2 + 4)]

i i

4 X ([3 X 2] X 5 . X 3) 38 +[ (3 X 4) 4- 6 X (2 + 4)]

1 1

4 X({[3 X2] X 5} X 3) 38 + [{(3 X4) V 6} X (2 + 4)]

EXERCISES
A. On a separate sheet of paper, rewrite each of these expressions
in unabbreviated form. [You need not put in outermost grouping
symbols. ]

Sample . 6 -f 4 + 3 + 9

Solution . [(6 + 4) + 3] + 9

1. 2 4 8 + 3 2. 7X5X3
3. 9-5-3 4. 6 + 2 + 3

5. 8 + 3 + 5+4 6. 9X2X3X5
7. *5 + *2 + *3 + ^7 8. *3 X ^2 X ^6 X *1

9. 3+8-2+5 10. 15-7+5-9


11. 5X6+3X4 12. 3X4 + 2 + 3

13. 15-8-3-7 14. 24 + 3 + 2 + 4

15. 5 + 9 + (6 + 8) 16. 7 + (9 + 5) + 8

17. 2 X (5 X 3) X 4 18. 6 X 8 X (3 X 7) X 4

19. 5 + (3 X 2 X 8) 20. 6 X (4 + 9 + 3 - 8)

21. 8 + [3 X{7 + 2) X 5] - 6 + 7 - 5

22. (5 X 3) + (4 X 7) - (8 X 2) + (8 - 2)

23. 6 + [5 X(3 + 7)] + [4 X (8 - 5)]

24. 4 X [5 - 3 - 1] X [7 + (2 X 4)]

B. According to the conventions we have discussed, each of the


expressions in Part A is unambiguous. You should be able to
simplify each expression without rewriting it in unabbreviated
form. Do so.
vl, »l^ ^\,
'f '1^ '1^
1/51

[Change Exercise 13 to '15 - 8 - 3 !'•]

Ans\wers for Part A.

1. 2 + 8) + 3 2. (7 X 5) X 3

3. 9 - 5) - 3 4. (6 -f 2) -f 3

5. (8 + 3) + 5] + 4 6. [(9 X 2) X 3] X 5

7. CS + *Z) + *3] + *7 8. [("3 X *2) X *6] X *7

9. (3 + 8) - 2] + 5 10. (15 - 7) + 5] - 9

11. (5 X 6) ^ 3] X4 12. [(3 X 4) V 2] T 3

13. (15 - 8) - 3] - 1 14. [(24 ^ 3) -^


2] -f 4

15. 5 + 9) +{6 + 8) 16. [7 +(9 + 5)] + 8

17. 2 X (5 X 3)] X 4 18. [(6 X 8) X (3 X 7)1 X

19. . + ([3 X 2] X 8) 20. 6 X ({[4 + 9] + 3} - 8)

21. {(8 + [{3 X (7 + 2)} X 5]) - 6} + 7] - 5

22. [(5 X 3) + (4 X 7)] - (8 X 2)} + (8 - 2)

23. 6 t [5 X(3 + 7)]} + [4 x(8 - 5)]

24. 4 X [(5 - 3) - 1]} X [7 + (2 X4)]

'1*

Answers for Part B •

1. 13 2. 105 3. 1 4. 1 5. 20

6. 270 7. *17 8. *252 9. 14 10. 4

11. 40 12. 2 13. 3 14. 1 15. 28

16. 29 17. 120 18. 4032 19. 53 20. 48

21. 139 22. 33 23. 68 24. 60

TC[l-40]
[1-40]

Here
[1.05] [1-41]

In simplifying abbreviated expressions which contain multi-


plication or division signs and addition or subtraction signs such
as
9+2X4-4-=- 2,

you first do the multiplications and divisions, working from left

to right, and then the additions and subtractions in the same order.
In particular, the expression:

9-^2X4-4^2
is unabbreviated in two steps:

9 -F (2 X 4) - {4 V 2),

and then:
[9 -^ (2 X4)] - (4 ^ 2).

So, the given expression is, in view of our conventions, completely


unanibiguous. It st5inds for 15.

Example 9 - Simplify by first un abbreviating:

5X9 + 3 -8-=-2X3.

Solution . Unabbreviate the expression.

5X9+3- 8 + 2X3
= (5 X 9) + 3 - [(8 + 2) X 3]

= [(5 X9) + 3] - [(8 + 2) X 3].

Now simplify it.

[(5 X9) + 3] -
[(8 + 2) X 3]

= [ 45 + 3] - [ 4 X 3]

48 - 12

= 36.

Example 10 . Simplify by first unabbreviating:

3 + (6 + 2 X 5) - (17 - 4 X 3).

Solution . Unabbreviate first.

3 + (6 + 2 X 5 ) - (17 - 4 X 3 )

= 3 + (6 + [2 X 5]) - (17 - [4 X 3])

= {3 + (6 +[2 X5])} - (17 - [4 X3])

(continued on next page)


[1-42] [1.05]

Simplify next.

{3 4(6 +[2 X5])} - (17 - [4 X 3])

= {3 + (6 + 10 )} - (17 - 12 )

= {3 4 16 } - 5

19 - 5
= 14.
^1, v', vU
'I"- 'l^ '!"•

C^. Rewrite each of the following expressions in unabbreviated form,


and then simplify it.

1. 3 + 5x10 2. 9X2+4 3. 8^2+5


4. 7-=-2X5 5.4X3-1 6. 2+5X6
7. 4X5+3X7+2X3 8. 6v2+15-^3+20-fl0
9. 3X7-2X5 + 4 + 8 10. 10x5 + 2X4+100
11. *5X"'4+"3X"2 12. 4 + "3 X 7 + "2 X "4

13. 8 + "2 + "3 X 6 14. 8 + "2 X "5 + 10 X 3

15. 7 X (8 + 3) X (6 + 2) 16. 12 X (4 + "2) + (20 + "3)

17. 3 + (16 + 2 + 4) + (5 X 2 - 3 X 3)

18. 5 X 3 - 6 + 2 + [(4 + 5) X 3 + 2 + 4 X 2]

19. (5 + 3) X (7 + 1) - 8 X 2 X (3 - 1) + 7 X 5 + 1

20. 6 + 3 + 8 X (6 - 3 X 2 + 5 + 4 + 2) + (4 - 3)

D^. Simplify.

1. 3X6-4X2 2. 5X4-3X2
3. 7X4 + 2 + 3X7 4. 6X5 + 2+4X9-^2
5. 18-7+2-8 + 2 6. 12 - (5 + 3) + 2 + 6

7. 18 - (7 + 2) - 8 + 2 8. 12 - (5 + 3) + (2 + 6)

9. 84 - {5 X [5 X 2 - 3 + (6 X 4 - 5) - 7] - (6 + 2 + 8)}

10. 16 + 3 X[(3 + 5) X2 - (2 + 1) X4] + 6 X(8 - 3)

[More exercises are in Part F, Supplementary Exercises .


]
1/52

The rewriting for Part C and [if any is necessary] for Part D
should be done on a separate sheet of paper.
vt>

Answers for Part C.

1. 3 + (5 X 10); 53 2. (9X2)+ 4; 22

3. (8 ^ 2) + 5; 9 4. (7 ^ 2) X 5j 17.5

5. (4X3) - 1; 11 6. 2 + (5 X 6); 32

7. [(4 X5) + (3 X 7)] + (2 X 3); 47

8. [(6 4- 2) + (15 T 3)] + (20 -=-


10); 10

9. [(3X7) - (2 X5)] + (4 T 8); 111

10. {[(10 X 5) + 2] X 4} 4 100; 1

11. (*5 X-4) + ("3 X -2); "14 12. [4 + ("3 X 7)] + ("2 X "4); "9

13. (8 + ~2) + ("3 X 6); "12 14. [8 + ("2 X "5)] + ( 10 X 3); 48

15. [7 X (8 + 3)] X(6 ^ 2); 231 16. [12 X (4 + "2)] + (20 + "3); 41

17. {3 + ([16 V 2] V 4)} + ([5 X 2] - [3 X 3]); 6

18. [(5 X 3) - (6 -f 2)] + [({(4 + 5) X 3} + 2) + (4 X 2)]; 49

19. [{[(5 + 3) X (7 + 1)] - [(8 X 2) X (3 - 1)]} + (7 X 5)] + 1; 68

20. [(6 + 3) + {8 X ([{6 - [3 X 2]} + 5] + [4 r 2]) }] + (4 - 3); 66

Answers for Part D.

1. 10 2. 14 3. 35 4. 33 5. 9

6. 14 7. 5 8. 11 9, 10. 58

TC[l-42]a
[1-43]

Here is a quiz which covers some of the ideas related to the conven-
tions for omitting grouping symbols.

I. Use the conventions for omitting grouping symbols and abbreviate


these expressions as nnuch as possible.
Sample 1^. (5 X 3) + 7

Answer . 5X3+7
Sample 2^. (4 + 5) + (6 + 3)

Answer. 4+5 + (6 + 3)

1. (8 X2) +(3X9) 2. (8 X2) +(3 +9)


3. [(3 X 8) X 7] X 6 4. [{3 X8) X7] + 6

5. (3 X 8) X{7 + 6) 6. [(3 X8) + 7] X 6

7. [2 X(9 + 1)] X 3 8. 2 X[(9 + 1) + 3;

9. [(3X5)-r2]X8 10. (3X5) + (2 X8)

II. Simplify.
1. 3 + 7X5 2. 3X7 + 5
3. 5 X "2 + 5 X "3 4. 5 X ("2 + 5) X -3

5. 2 X 3 + 4 + "4 6. 5 + 3 X "10 + *80

•J,

Answers for quiz.

I. 1. 8X2 + 3X9 2. 8 X2 + (3 + 9)
3. 3X8X7X6 4. 3X8X7+6
5. 3 X 8 X (7 + 6) 6. [3 X8 + 7] X 6

7. 2 X(9 + 1) X 3 8. 2 X [9 + 1 + 3]
9. 3 X 5 -r 2 X8 10. 3 X5 -^ (2 X8)

n. 1. 38 2. 26 3. "25

4. "45 5. 6 6. *55

TC[l-42]b
[1-42]

i
[1-43]

•n
1/55

'4 + 5 + 6' is an abbreviation for "(4 + 5) + 6', *4 X 5 X 6' is an


abbreviation for '(4 X 5) X 6'. Later, when we have learned about
the associative principles for addition and multiplication, we shall
see statements such as:
4+5+6=4+(5+6).
It case that '4 + 5 + 6' is an abbreviation for '4 + (5 + 6)'.
is not the
The sentence displayed above is true because it is an instance of the
associative principle for addition. On the other hand, the fact that
the sentence :

4 + 5 + 6 = (4 + 5) + 6
is true is a consequence of our convention for onniitting grouping
symbols. [See T C[l -48]a. ]

Answers for Part B.

1. 397 2. 7552 3. 683 4. 1027

5. 370 6. 9 800 7. 487000 8. 687100

9. 182 10. 6426 11. 1551 12. "7275

13. 360 14. 8400 15. 770 16. 1624000

17. 98 18. "9624 19. 634 20. 97

21. "79 22. 5627

TC[l-43]b
[1-43]

'n
1/54

These exercises should be worked without your pointing out short


cuts. That is, each exercise in these two parts can be worked the
long way. The student who doesn't discover the short cuts will be
able to do the problems anyway. If he tries to do them mentally, he
will be compelled to search for short cuts.

In Part A, we think the students will see that in Exercise 1, for


example, the given nutnber is divisible by 7 and by 8. Some nnay
also suggest that 2, 4, and 14 are also divisors. The answers below
indicate the obvious divisors [which we hope the students will dis-
cover], and also [for your convenience] suggest others by giving a
simpler name for the nuinber being considered. Avoid, if possible,
emphasizing the other factors.

2. 9, 10, other whole number factors of 90.


3. 13, 20, other whole number factors of 260.
4. 31, 7, other whole number factors of 217.
5. 6, 100, other whole number factors of 600.
6. 593, 5, other whole number factors of 2965.

7. 19, 15, other whole number factors of 285.

8. 7, 13, other whole nunnber factors of 91.


9. 41, 138, other whole number factors of 5658.
10. 87, 107, other whole number factors of 9309.

11. 547, 2728, other whole number factors of 1492216.


12. 3163, 7680, other whole number factors of 24291840.

'I"

Note carefully that in expressions in Part B like:


4 + 5 + 6, and: 4X5x6,
grouping symbols have been omitted by convention. Thus,

TC[l-43]a
[1.05] [1-43]

EXPLORATION EXERCISES
A. For each number listed below, give a whole number (other than
itself and 1) which divides it. ['divides it' means the same thing
as 'divides it exactly'. For example, 2, 3, 4, and 6 each divides
12, but 5 does not divide 12. ]

Sample . 3X5+7X5
Solution . This simplifies to '50'. So, 5 is a number which
divides 3X5+7X5. [Other numbers are 2, 10,

and 25. ]

1. 3X7+5X7 2. 8X9 + 2X9


3. 2X13 + 18X13 4. 5 X 31 + 2 X 31

5. 93 X 6 + 7 X 6 6. 3 X 593 + 2 X 593

For each number listed below, give two numbers which divide it.

7. 3X19 + 12X19 8. 6X7+7X7


9. 67X41+71X41 10. 51X87+56X87
11. 1319X547+1409X547 12. 3163X3833+3163X3847

Simplify mentally.

1. (387 + 9) + 1 2. (7452 + 75) + 25

3. 583 +92+8 4. 927 + 152 + "52

5. (37 X 5) X 2 6. (98 X 25) X 4

7. 487 X 25 X 40 8. 6871 X 20 X 5

9. (82 + 47) + 53 10. 13 + (987 + 5426)

11. 894 + 751 + "94 12. "6341 + "275 + "659

13. (12 X 15) X2 14. (5 X84) X 20

15. 55 X 7 X 2 16. 40 X 812 X 50

17. 98 + 76 + "76 18.' "583 + "9624 + 583


1
19. 634 X 5 X f 20. 97 X yy X 17 .

21.
48
X "79 X "48 22. "384 X 5627 X ^
384
[1-44] [1.06]

1.06 Principles for the numbers of arithmetic --The numbers of


.

arithmetic have certain properties which you make use of time and
again as you do problems with these numbers.

(I) Here is a start on a multiplication table. Your job is to

fill in the empty spaces.

2 3
X 1200
4
87 21
3

63
21 14 1827 441
4

87 58 104,400 7569

2 4 1
3 9 2

3 9 261
900
4 16 4

1200 800 1,440, 000 25, 200


[1-45]

^he

There may be some students who are not acquainted with this type of
multiplication table. Point out to them that the numbers to be multi-
plied are listed in the top row and in the left-hand column, and that
the products are listed in the cells. Do not point out to them that
they are supposed to look for a short cut in this exercise. The very
fact that there is a short cut is something which must be discovered
by the student.

Similarly, the sorting exercise at the top of page 1-45 calls for a
discovery on the part of the student that there is a short cut. The
purpose of Exannples I and II is to build an awareness of the com-
mutative principle for multiplication. It is the case, however, that
students have been aware of this principle for a long time. What we
are trying to do in these exercises is to shock them into thinking
about the principle. Another way of getting them to think about the
commutative principle is to have them fill in the blanks of the follow-
ing sentences to nnake true ones out of them:

(a) 9 X = 63 (b) X 12 = 3 X 20

(c) 9X = 4X9 (d) 5X = 32 X 5

(e) X 95 = 95 X 142 (f) 583 X 684 = 684 X

Notice that the first two exercises in this list involve multiplication
and division computations whereas the other exercises require only
an awareness of the commutative principle for multiplication. It
may be that a few students will be trapped into carrying out laborious
computations. For these students, the recognition that the commuta-
tive principle elinninates the need for such computations comes as a
genuine surprise.

Once students have become aware of the commutative principle of


multiplication, they will be ready to tell you that there is also such
a principle for addition.

Even though the title of this section is 'Principles for the numbers of
arithmetic', you inay have to remind the students every once in a
while that we are now talking about numbers of arithnnetic and not real
numbers. A later section deals with analogous principles for real
numbers.
'1^

TC[l-44, 45]a
[1-44]

l.Of
[1-45]

^he

Here is the sorting for (II) on page 1-45,


3
21 X 17 X96 1984 X 657 X 891 27 X 31
I
17 X 21 96 X I X 19 84 891 X 657 93 X9

Verbalizations of the principles [in terms of variables, or "pronunn-


erals"] are given in Unit 2. As far as Unit 1 is concerned, all we
want is that the student know the names of the principles and that he
be able to give instances. As the student practices giving and identi-
fying instances, he becomes aware of patterns. It is this awareness
that we count on in Unit 2 when we come to state the principles. With
the statement of the principle at hand, the giving of instances is a
mechanical task. We want students to do some thinking before they
become mechanically proficient. Here is a way to develop a feeling
for the principles without actually stating them.

Ask each of students A and B to think of a number of arithnietic.


Then have B tell his nunnber to A and ask A to multiply his num-
ber by B's, and to record the product. Then have A tell his
number to B and ask B to multiply his number by A's, again re-
cording the product. The class should then be able to predict
that both students will get the sanne product. This prediction is
based on the student's acceptance of the commutative principle
for Multiplication. You can obtain the relevant instance of the
commutative principle for multiplication in this case as follows.
Ask A for his number. Suppose it is 7. When he tells it to you,
write the following on the board:
7 X X 7.

Then ask B for his number. Suppose it is 4. Then fill in the


empty spaces to get:
7X4 = 4X7.
[Using three colors of chalk is helpful- -white for the times signs
and equality sign, blue for the '7's, and yellow for the *4*s.]
The completed sentence is an instance of the commutative prin-
ciple for multiplication.

Toillustrate the generality of the associative principle for addition,


select three students. A, B, and C. Ask each to think of a number.
Then B tells his number to A, and A adds B's number to his. Then
TC[l-44, 45]b
[1-44]

1.01
[1-45]

^he

1/58

C tells his number to A, and A adds C's nunnber to the sum he just
obtained, recording the total. Now, C tells his number to B, B
adds C's number to his, B tells the sum to A, A adds this sum to
his number, and records the total. The class should be able to
predict that the totals are the same. To produce the relevant
instance, first write this fornn on the board:

( + ) + - +( + ).

remarking that in each case two additions are to be carried out.


Then ask for A's number,
(7+ ) + = 7 +{ + ).

Then for B's number.


(7 + 4) + = 7 + (4 + ).

Then for C's nunaber.

(7 + 4) + 9 = 7 + (4 + 9).

TC[l-44, 45]c
[1-44]

l.Of-
[1.06] [1-45]

(II) Sort the expressions below into pairs of numerals for the
same number.
21X17 657X891 1984 X |- j X 96

4x1984
4
17X21 ^X243
59
891X657

96 X I 27X31 243 X ^^ 93X9


How many products did you need to compute to make your list of

pairs ? How nnany products did you need to compute to fill the

table in the first problem?


If you had to do more than two computations for problem (II)

and any for problem (I), you failed to recognize places to make
use of an important property of the numbers of arithmetic. This
is that if you multiply a pair of numbers, you get the same answer
no matter what order you use in multiplying. This is the commu -
tative principle for multiplication . Instances of this principle are:

5X7 = 7X5,
3 3
21 X X = T X 21.
4 4

Notice that you don't have to simplify the numerals connected by


the equality signs in these two sentences in order to know that the

sentences are true. If you believe that the commutative principle


for multiplication is true then you believe that each instance of it

is true, also.
Is there a corresponding property of the numbers of arithmetic
for addition? If Mr. Brown has 12 Black Angus cows and 8 Holstein

cows on his farm, how many cows of each kind must he buy to have
twice as many cows, and the same number of Black Angus as
Holstein?
To solve this problem quickly you need to recognize that

12 + 8 = 8 + 12,

and this is an instance of the commutative principle for addition .

Other instances are:


986 + 724 = 724 + 986,

16. 357 + 5. 009 = 5. 009 + 16. 357.


[1-46] [1-06]

Notice, again, that you don't feel any ur^c to compute to see if the

last two sentences are true. As long as you believe that the connmu-
tative principle for addition is true, you also believe that each instaJice
of it is true.

There are still other properties of the numbers of arithmetic


which are useful in solving problems, especially in finding short

cuts.

For example, suppose you were trying to find the total number
of raffle tickets sold on three consecutive days.
First day. . . . 47, Second day. . . . 75, Third day. . . . 25.

One way of doing this is to find the total for the first and second
days, and then add to this total the number sold on the third day.
So,

(47 + 75) + 25 = 122 + 25 = 147.

But a much easier way, which most likely has already occurred to
you, is to do the problem this way ;

47 + (75 + 25) = 47 + 100 = 147.

There was probably no doubt in your mind that you would get the
samae sum in doing the problem the second way as you would in

doing it the first way. You feel sure about this because you feel
sure about another property of the numbers of arithmetic, a property
which is expressed by the associative principle for addition . Other
instances of this principle are :

(8 + 5) + 19 = 8 + (5 + 19)

23 + 91 + 9 = 23 + (91 + 9),

15 + (85 + 38) = 15 + 85 + 38.


And, as you have probably guessed by now, there is also the
associative principle for multiplication . Notice how it gives you
short cuts.
(27 X 5) X 2 = ?
(27 X 5) X 2 = 27 X (5 X 2). So, (27 X 5) X 2 = 27 X 10 = 270.
(897 X 4) X 25 = ?

(897 X 4) X 25 = 897 X (4 x 25). So, (897 X 4) x 25 = 89700.


50 X (2 X 68) = ?

50 X (2 X 68) = (50 X 2) X 68. So, 50 X (2 X 68) = 6800.


[1.06] [1-47]

Sometimes you use short cuts which depend upon more than just

one of these principles. For example, suppose you want to find the

total nunnber of points you made in a test which had three parts:

Part I 27 Part II 39 Part III 23

The straight -forward procedure would be:

(27 + 39) +23 = 66 + 23 = 89.

But a short cut might involve thinking through these steps:

(27 + 39) +23 = 27 + (39 + 23),

because of the associative principle for addition, and

27 + (39 + 23) = 27 + (23 + 39),

because the commutative principle for addition tells us that

39 + 23 = 23 + 39.

Finally,
27 + (23 + 39) = (27 + 23) + 39,

because of the associative principle for addition. So, we know that

(27 + 39) + 23 = (27 + 23) + 39. Since 27 + 23 is 50, and 50 + 39 is 89,

we know that

(27 + 39) + 23 = 89.

[Did you, without realizing it, use the associative and connmutative
principles for addition in finding that 27 + 23 is 50?]
Suppose you were asked to do long "column addition" as in these

examples. Do them.

5 2 5 14
3 9 7 25
5 1 2 32
7 8 3 51
6 9 3 98

Did you skip around to find the easy combinations ? Do you believe
that you can get a correct total this way? That you do get a correct
total even though you skip around is a consequence of the associative
and commutative principles for addition.
[1-48] [1-06]

EXERCISES

A. Each of the following sentences is an instance of one of the four

principles you have just learned. Tell which principle.

1. 9+7=7+9 Z. 3X5=5X3
3. 61 + 17 = 17+61 4. 97X816 = 816X97
5. 81 + (9 + 13) = (81 + 9) + 13 6. (93 X 5) X 100 = 93 X (5 X 100)

7. 71 X (51 + 47) = (51 + 47) X 71

8. 523 + 43 + 79 = 523 + (43 + 79)

9. 657 X 982 X 539 = 657 X (982 X 539)

10. (841 + 56) + (75 + 37) = (75 + 37) + (841 + 56)

11. [(72 + 45) + 63] + 85 = (72 + 45) + (63 + 85)

12. 72 + [(45 + 63) + 85] = [72 + (45 + 63)] + 85

13. 72 + 45 + (63 + 85) = 72 + [45 + (63 + 85)]

14. 72 + (45 + 63) + 85 = 85 + [72 + (45 + 63)]

15. (81 + 37) + (92 + 54) = (92 + 54) + (81 + 37)

16. 7 + 3|- = (7 + 3) + [' 3|-' IS an abbreviation for '(3 + ^)'.]


^
17. 85f X (25 + 48) = (25 + 48) X 85^

18. 72 + (45 + 63) + 85 = 72 + [45 + 63 + 85]

19. (72 + 45 + 63) + 85 = (72 + 45) + (63 + 85)

20. (3X7X2X5)X(8+4+3+2) = (8+4+3+2)X(3X7X2X5)


B^, None of the following sentences is an instance of any of the four
principles you have just learned. However, some of the sentences
are consequences of the principles. Tell which sentences are
consequences of which principles.

Sample 1_. 5 + (7 X Z) = 5 + (2 X 7).

Solution . Since '7X2 = 2X7' is an instance of the commu-


tative principle for multiplication, the given
sentence is a consequence of that principle.
[1-49]

1/59

Answers for Part A.

1. cpa 2. cpm 3. cpa 4. cpm


5. apa 6. apm 7. cpm 8. apa
9. apm 10. cpa 11. apa 12. apa
13. apa 14. cpa 15. cpa 16. apa
17. cpm 18. apa 19. apa 20. cpm
[Notice, in Exercise 16, the use of the ap'a in justifying simplifica-
tion of expressions containing "mixed numbers".]

Note on the associative principles --Recall [page 1-38] that, for


.

example,
'72 + 45 + 63' is an abbreviation for '(72 + 45) + 63'.

Consequently,
'72 + 45 + 63 + 85' is an abbreviation for '(72 + 45) + 63 + 85',

and, similarly ,

'(72 t 45) + 63 + 85' is an abbreviation for '[(72 + 45) + 63] + 85'.


So, for exanaple:

72 + 45 + 63 + 85 = [(72 + 45) + 63] + 85

is naerely an abbreviation of the trivial statement:


[(72 f 45) + 63] + 85 = [(72 + 45) f 63] + 85.
On the other hand:
72 + 45 T 63 + 85 = (72 i- 45) + (63 + 85)

is an abbreviation of:

[(72 + 45) + 63] + 85 = (72 + 45) + (63 + 85),


which is an instance of the associative principle for addition, and,
so, is by no means trivial. Other exannples of "trivial" sentences:
72 + (45 + 63 + 85) = 72 + [(45 + 63) + 85]
72 + 45 + (63 + 85) = (72 + 45) + (63 + 85)
72 + (45 + 63) + 85 = [72 + (45 + 63)] + 85
(72 + 45) + 63 + 85 = [(72 + 45) + 63] + 85
(72 + 45 + 63) + 85 = [(72 + 45) + 63] + 85
(72 + 45) + 63 + 85 = (72 + 45 + 63) + 85

TC[l-48]a
[1-48]

a
[1-49]

1/60

Here are sentences vi/hich require, for their justification, the associ-
ative principle for addition:

72 + (45 T 63 -i- 85) = 72 + [45 + (63 + 85)]


72 + (45 + 63 + 85) = [72 + (45 + 63)] + 85

72 + 45 + (63 + 85) = 72 + [45 + (63 + 85)]


72 T 45 + (63 + 85) = [(72 + 45) + 63] + 85
72 + (45 + 63) -t- 85 = [(72 + 45) + 63] + 85
72 + (45 + 63) + 85 = 72 + [(45 + 63) + 85]
(72 + 45) + 63 + 85 = [72 + (45 + 63)] + 85
(72 + 45 + 63) + 85 = (72 + 45) + (63 + 85)
Each of the following sentences is an instance of the associative
principle for addition.
(72 + 45 + 63) + (85 + 22) = (72 + 45) + [63 + (85 + 22)]
(72 + 45 + 63) + (85 + 22) = (72 + 45 + 63 + 85) + 22
Similar remarks can be made concerning the associative principle
for multiplication. [In fact, you can reread the preceding para-
graph, substituting 'X' for '
+ and 'multiplication' for 'addition'.]
'

In claiming that a sentence like:

75 + 45 + 63 = 75 + (45 + 63)
is true, a student naight [correctly] say that it is an instance of the
associative principle for addition, or he might even say that he used
the associative principle and grouped the '45' with the '63' to get the
expression on the right. On the other hand, in claiming that a sentence
like:
75 + 45 + 63 = (75 + 45) + 63
is true, a student should not mention the associative principle or even
say that he associated the '45' with the '75'. Instead, he should say
that the sentence is true by convention, or that he grouped the '45*
with the '75' by convention.

Be sure to distinguish between instances of principles and, more


generally, sentences which are consequences of such instances. For
example, 3 + 2 + 5 = 3 + (2 + 5)' is an instance of the apa.
' The sen-
tence •6X(3 + 2+5) = 6x[3+(2 + 5)]' is not an instance of the apa,

TC[l-48]b
[1-49]

1/61

but is a consequence of the instance just mentioned. [See discussion


of this nnatter in TC[l-56].]

Here is a quiz which tests the ability to recognize instances of the two
comnnutative and the two associative principles; it also tests the ability
to distinguish between such instances and sentences which are conse-
quences of the conventions for omitting grouping symbols.
Each of the following sentences is true. It is either an instance of
the commutative principle for addition [cpa],
the connmutative principle for multiplication [cpm],
the associative principle for addition [apa],
the associative principle for multiplication [apm],
or it is a consequence of our conventions for omitting grouping symbols.
Tell which by writing either an abbreviated name of the principle or the
word 'convention'.
Sample U 6x9 = 9x6
Answer . 6x9 = 9x6 cpm
Sample 2. (3 X 9) X 5 = 3 X 9 X 5

Answer . (3X9)X5 = 3X9X5 convention

1. 8 X 12 X 5 = 8 X (12 X 5)

2. 6 + 4+3 = (6 + 4) +3
3. 6 + (4 + 3) = 6+4 + 3

4. 7X9X3 = 7X(9X3)
5. 3 + 8 + 9 = 9 + (3 + 8)
6. (120 + 6+2)X3 = 3X (120 +6+2)
7. 8X5+7 = 7+8X5
8. 9 X (12 + 15) = (12 + 15) X 9

9. [(6 + 1) X (7 + 4)] + (7 + 6) = (6 + 1) X (7 + 4) + (7 X 6)

10. (8 X 3 X 2) + [(6 X 5) + (8 X 3 X 2)] = (8 X 3 X 2) + (6 X 5) + (8 X 3 X 2)

Answers for quiz.

1. apm 2. convention 3. apa 4. apm 5. cpa


6. cpm 7. cpa 8. cpm 9. convention 10. apa

TC[l-48]c
[1-48]

A.
[1-49]

N
1/62

Answers for Part B [which begins on page 1-48],


[In addition to theanswers we list the instances of principles from
which the given sentences follow. None of the given sentences is
itself an instance of cpa, cpnn, apa, or apm.]
1. cpm; '6X5 = 5X6' 2. cpa; '4+7 = 7 + 4'

3. cpa; •7 + 3=3 + 7' 4. cpa; *9 + 3 = 3 + 9'

5. convention, or: abbreviation 6. convention


7. cpa; '6 +
I =
I + 6' 8. convention
9. cpm and cpa; '(6 + 5) X 3 = 3 X (6 + 5)' and '6 + 5 = 5 + 6'
[or: '6 + 5 = 5 + 6' and '(5 + 6) X 3 = 3 X (5 + 6)'.]

10. apa and cpa; '8 + 5 + 3 = (5 + 3)', '5 + 8++ 5', 3=3 and
'8 + (3 + 5) = 3 + 5 + 8' [or: '8 + 5 + + (8 + 5)', 3=3
•8 + 5 = 5 + 8', and '3 + (5 + 8) = 3 + 5 + 8'.]

11. cpa; '4 + 7 = 7 + 4', '3 + 8 = 8 + 3', and


*(7 + 4) + (8 + 3) = (8 + 3) + (7 + 4)' [or: ... .]

12. cpm; '10 X 3 = 3 X 10' and '15 X 3 = 3 X 15'.

'I*

Answers for Part C.


1. 72 59
a numeral for any number of arithmetic in the first blank and a
copy of this numeral in the second blamk.
4. Same as for Ex. 3 5. 10 6. 9
7. Same as for Ex. 3 8. Same as for Ex. 3 9. 7
10. 7 11. 10 12. 84
13. 12 14. 13 15. 1/2
16. 8 17. 859 18. 9

Answers for Part D.


1, No. 7-5 = 2. but '5 - 7' is nonsense; 64-2=3 but 2-r6= 1/3.
2. No. (8 - 5) - 2/ 8 - (5 - 2); {12 + 4) + 2/ 12 v(4 4- 2).

TC[l-49]
[1.06] [1-49]

Sample 2. 72 + (45 + 63) + 85 = [72 + (45 + 63)] + 85

Solution . The expression on the left of the' equality sign is

merely an abbreviation for the expression on the

right. So, none of the four principles is required


in showing that the given sentence is true.

1. 6X5X3 = 5X6X3 2. 4+7+3 = 7+4+3


3. 4 + (7 + 3) = 4 + (3 + 7) 4. {9 + 3) X 5 = (3 + 9) X 5

5. 7+4X3 = 7+ (4X3) 6. 8+2+9 = (8 +2) +9

7. 9^ + 6| = 9|- + (I + 6) 8. 14 X2;^ = 14 X (2 + ^)

9. (6 + 5) X 3 = 3 X (5 + 6) 10. 8+5 + 3 = 3+5+8


11. (4 + 7) + (3 + 8) = (8 + 3) + (7 + 4)

12. (10 X 3) + (15 X 3) = (3 X 10) + (3 X 15)

C. Fill the blanks to make true sentences.

1. 19 +_ =72+19 2. 31 X = 59 X 31

3. 6 X X 9 = 6 X ( X 9) 4. 85 + 97 X = 85 + X 97

5. 10+7 + 3=3+ +7 6. (8 + 5)X2=8+ X2


7. 9 + + 4 = 9 + ( +4) 8. ( +5) +7 = 7+5 +

3X7 3X7
9.
7X5 ~ 5 X
10. 1 X(7 X i) = ^ X

11. 3X5X3y= X5 12. 61 + ( + 39) = 184

13. 6 + 3 = + 6 14. 9 - 5 = - 9

15. 24 + 4 + 2 = 24 + (4 •
__^ _)
-
16. 15 - 6 - 1 = 15 -
(_ 1)

17. 107 X + 372 X 76 = 76 X 372 + 859 X 107

18. (8 +7) X (9 + 16) = (16 + ) x( 7+8)

D. 1. Are subtraction and division commutative operations? Give


examples to justify your answers.

2. Are subtraction and division associative? Give examples.


[1-50] [1.06]

ANOTHER PRINCIPLE
Perhaps you have found short cuts for some problems which involve
both multiplication and addition.

7X11+3X11=?
Do you see a short way of solving this problem? If you don't, you may
see it after you have filled in the blanks in the following sentences.

4X15+ 6 X 15 = 60 + 90 150 10 X 15

8 X 29 + 2 X 29 = + X 29

13 X 21 + 17 X 21 = + _
= = X 21

5X9 + 6X9 = + X 9

21X8 +19X8 = + X 8

2 3
I-X7 +I-X7 = + X 7

Can you do these problems by a short cut?

8X7 + 3X7=?
6 X 582 + 4 X 582 = ?

The same short cut can be used in the following problem.

Suppose you have two vacation jobs, one of which pays


85 cents an hour, and the other $1. 15 an hour. How
much have you earned if you worked 35 hours on each
job? Some people may solve this problem the hard
way by first multiplying to tell how much was earned
on each job, and then adding the results.

. 85 X 35 + 1. 15 X 35 = 29. 75 + 40. 25 = 70. 00.

Do you see an easy way? First, add the rates of pay,


and then multiply.

(. 85 + 1. 15) X 35 = 2. 00 X 35 = 70. 00
[1-51]

~tal

1/63

The blanks should be filled in like this:

= ^3^ + 56 = ^90 = \0 X 29

= ^73 + 357 = 6:>hO = ^0 X 21

= 46 + 54 = 99 = (/ X 9

= /^8 + 16^ = 3^0 =4-0x8


/4
+ "5" = "^ = /
X 7

On TC[l-44, 45]b and c, we describe a classroonni technique for


developing a feeling for the connmutative and associative principles.
Here is a similar technique for helping students feel "comfortable**
with the distributive principle. This principle is the one which is
most surprising to students, probably because it "mixes" both addi-
tion and multiplication. Again select students A, B, and C. B tells
his nunnber to A, and A adds B's nunnber to his. Then A tells this
sum to C who multiplies it by his nunnber, recording the product.
Now, A tells his number to C, and C multiplies it by his nunnber.
Similarly, B tells his number to C, and C again multiplies it by his
number, adds this product to the one just obtained, and records the
sum. The class should predict that the recorded product is the same
as the recorded sum. The relevant instance is constructed in these
steps
{ + )X =(X) + (X)
[A sum and a product in one case, two products and a sum in the
other. ]

{7 + ) X = {7 X ) + ( X )

(7 + 4) X = (7 X ) + (4 X )

{7 + 4) X 9 = (7 X 9) + (4 X 9).

By convention, we can omit the grouping symbols on the right side.

(7 + 4)X9 = 7X9 + 4X9


Whenever questions arise concerning the proper identification of a
sentence as an instance of a certain principle, the proper "pattern"
sentence should be written [that is, the one with operation signs and
grouping symbols only], and the empty spaces filled, one at a tinae.

TC[l-50]
[1-51]

^tal
1/64

Here, in identifying the distributive principle for nnultiplication over


addition, we^ depart fronn the usual convention. Most other texts
would say that, for example, 3 X (5 + 8) = 3 X 5 + 3 X 8* is an instance
'

of the dpma and that (5 + 8) X 3 = 5 X 3 + 8 X 3* is an instance of the


'

right {-hand) distributive principle for multiplication over addition.


But, to be consistent with the convention according to which we write
a nanne for the multiplier to the right of the nnultiplication sign, one
should, as we do, say that (5+8)X3 = 5X3 + 8X3 because multipli-
cation distributes over addition and, so, say that '(5 + 8) X 3 =
5X3+8X3' is an instance of the dpma. On page 1 -52 we say that
'
3 X (5 + 8) = 3 X 5 + 3 X 8' is an instance of the left dpma. When cor-
rectly filled in, the sentences in Exercises 9, 10, and 1 1 of Part A
are instances of the idpma.

Answers for Part A.


1. 3 2. 8 3. 9 4. 8 5. 3 6. 3

7. 2 8. 8 9. 2 10. 5 [for both blanks] 11. 9

Sentences 1, 2, 3, 4, and 6 are instances of the dpma. [The sentence


in Exercise 10 can also be completed to a true sentence by writing a
' 1' in the first blank and a 7' in the second.
'
If something like this
occurs to your students, tell them that it's clever of them to have
thought of it but that it will be helpful later if they establish the habit
of writing copies of the same numeral in similarly shaped blanks. ]

You may want to ask your class of what principles the completed
sentences in Exercises 5 and 8 are instances. [Exercise 5: apa;
Exercise 8: cpm] Also, with Exercise 7, ask if it would be possible
to show that the completed sentence is a consequence of the dpnna.
[It is, by noting that 10 = 2X5. The completed sentence is, then, a
consequence of the dpma and the fact that 2X5 = 10. ]

TC[1-51]
[1.06] [1-51]

The fact that you are sure that this easy way will give you the total
earned is a consequence of your belief in a principle called the
distributive principle for multiplication over addition . The instance
of it which we just used is:

. 85 X 35 + 1. 15 X 35 = {. 85 + 1. 15) X 35.

Here are other examples of how this principle makes conaputations


easier.
Example 1. 8 7- X 5 = ?

Solution . (8+7-)X5 = 8X5 + rX5


= 40 +
1
= 40|.

Example 2 . 91x61+9x61 = ?

Solution . 91 X 61 + 9 X 61 = (91 + 9) X 61

= 100 X 61

= 6100.

EXERCISES
A. Fill in the blanks to make true sentences, and then tell which are
instances of the distributive principle for nnultiplication over
addition.

1. 5X7+3X7 = (5+ )X7


2. 8X5+2X5 = ( +2)X5
3. 7X9 + 3X9 = (7 + 3) X
4. (2+8)X5 = 2X5+ X5
5. (6 + 3) + 2 = 6 + ( +2)
6. 2X7+ X7 = (2+3)X7
7. 10 + 3X5= X5+3X5
8. (6 + 5) X = 8 X (6 + 5)

9. 7X(3+2) = 7X3+7X
10. 5X(4+8)= X4+ X
11. 9X7+9X3= X{7+3)
UMIVLK^ITY OF ILLINOIS
LIBRARY
[1-52] [1-06]

v'x v^ vl^
'i- '1^ 't-

If you wrote a ' 9 ' in the blank in Exercise 3 of Part A, you


obtained the true sentence:

7X9+3X9 = (7+ 3) X 9,

and this sentence is an instance of the distributive principle for


multiplication over addition. An instance of this principle tells you

a way of connputing the sum of two products which have the same
nnultiplier.

If you wrote a ' 9 ' in the blank in Exercise 1 1 of Part A, you


obtained the true sentence:

9X7+9X3 = 9X{7+3).
This sentence tells you a way of computing the sum of two products
which have different multipliers. So, this sentence is not an instance
of the distributive principle for multiplication over addition. How-
ever, it is an instance of another principle which is called the left
distributive principle for multiplication over addition . Here are
some other instances of this new principle.

12 X 3 + 12 X 97 = 12 X (3 + 97)

20 X (4 + ^) = 20 X 4 + 20 X ^ .

Write three instances of the left distributive principle for multipli-


cation over addition, and three instances of the distributive principle
for multiplication over addition.

B^. Fill in the blanks to make true sentences, and then tell what
principles they are instances of.

1. 4X9+7X9 = {4+ ) X 9

2. 9X4 +9X7 = 9 X( + 7)

3. 6X8j = 6X8+ X
J
4. 4X7X3 = 4X(7X )

5. (8 + 2) X (9 + 3) = (9 + 3) X (8 + )

6. (6 + 5) X 9 + (7 + 3) X 9 = [(6 + 5) + (
+ 3)] X 9
7. (8 + 5) X (6 + 2) = (8 + 5) X + (8 + 5) X 2

8. (8 + 5) X (6 + 2) = 8 X (6 + ) + 5 X (6 + 2)
[1-53]

1/65

Three instances of the left distributive principle for multiplication


over addition:
(1) 5X(3+Z) = 5X3f5X2
(2) J X (4 + 5) = 3i X 4 + 3j X 5
3

(3) 7X8 + 7X9 = 7X{8 + 9)


Three instances of the distributive principle for multiplication
over addition:
(1) {3 + 2)X5 = 3X5 + 2X5
(2) (4 + 5) X 3-^ = 4 X 3-| + 5 X 3|-

(3) 8 X 7 + 9 X 7 = (8 + 9) X 7

Answers for Part B.


1. 7, dpma 2. 4, idpma 3, 6, idpma
4. 3, apm 5. 2, cpm 6. 7, dpma
7. 6, idpma 8, 2, dpma

TC[l-52]
[1-52]

i
t

'I
\
[1-53]
[1-52]

If

obf 1/66

Of course, other principles besides the pml are needed to justify the
conclusion stated at the bottom of page 1-53. For example, one
needs to know that 7/7 = 1, that 5/5 = 1, and how to multiply rational
nunnbers. However, you should at present overlook this point unless
your students insist on it. [In Unit 2, students will learn to derive
theorems about real numbers from basic principles and one of these
'2 3 2* 5 + 3» 7'
theorems will have as an instance the sentence -= + -^ ~ = p
7* 5
7 5
inwhich the numerals name real numbers, rather than, as on page
1-53, numbers of arithmetic]

For your information, principles for real numbers, analagous to the


principles for nunnbers of arithinetic which are illustrated on page
1-54, are illustrated on page 1-61.

TC[l-53, 54]
[1.06] [1-53]

MORE PRINCIPLES
V/e have mentioned and illustrated six principles which express
certain properties of operations with the numbers of arithmetic.

There are also certain numbers of arithmetic which have interesting


properties. For example, there is a number of arithmetic which
when added to any number of arithmetic gives you back that number.
In each of the blanks below, write a numeral which makes the sent-
ence true.

4 + =4

18 ~ 18

Is there a number of arithmetic which does a similar thing for multi-


plication? What must you write in each blank below to make the

sentence true ?

4 X = 4

23 _ 23
18 18
These properties of and 1 are expressed by the principle for adding
and the principle for multiplying by 1_. [Can you guess what the principle
for multiplying by is? Give sonne instances of it. ]

The principle for multiplying by lis used in adding fractional


2 3
numbers. For exanaple, suppose you want to add =- and — . In

order to simplify the expression:

2 3
we need to find names for =- and — which have the same denominator.
The procedure usually followed
to
3
multiply ^ by
7
=- to get
21
-^j^- .
is to multiply
Then we write:
10
2 5
— by — to get
10
-^
-io
, and

1_0 2J_ _ 3j_


35 35 ~ 35
So,

2 3 _ 3J_ •

7 5 35
[1-54] [1-06]

Although we know that jr is the sum of j^ and j^ , how do we know


2 3
it is the sunn of =- and — ? The principle for multiplying by 1 assures

us that since
5.,^
— is 1,
^
the product of
,2— and,5.2
g- is — .
c
bo,
10.
^ is — .

Similarly, the principle for multiplying by 1 assures us that -^ is

7-
5
. So, the sum
summary
of -rr
So
and -r^ is the
ib
sum of
to
— and —

we have discussed
.

Here is a of the principles for the

numbers of arithmetic.

The Comnnutative Principle for Addition [cpa]

7 + 29 =:: 29 + 7

9. 34 + 5 = 5 + 9. 34

The Associative Principle for Addition [apa]

(9 -;-
5) + 15 = 9 + (5 + 15)

6|+{7| +8|-) = (6| +7|) +8i-


87 + 9 + 91 = 87 + (9 + 91)

The Commutative Principle for Multiplication [cpm]

590 X 2 = 2 X 590

81. 3 X 17. 7 = 17.7 X 81. 3

4^
2
X 6x
4
= 6x
4
X 4^
2

The Associative Principle for Multiplication [apm]

(84 X 5) X 20 = 84 X (5 X 20)

16| X(6 X 59) = {16| X 6) X59


97 X 25 X 4 = 97 X (25 X 4)

The Distributive Principle for Multiplication over Addition [dpma]

5 X 98 + 95 X 98 = (5 + 95) X 98

(|- + X 12 = |- X 12 + |- X 12
I")

X59 + X 59 = (|- + X59


I" I I")
[1-55]

*:ion [idpma]
1/67

Answers for Part A.

1, paO pml cpa 4. cpm


5. pmO cptn pml 8. dpma
9. apm 10. Mpma 11. dpma 12 Mpma
13. dpma 14. cpm 15. cpm 16 dpma
[Students may need to be reminded, in connection with Exercise 9,
that *3.59 X 8.61 X 7. 32' is an abbreviation for *(3.59 X 8.61) X 7. 32'.]

We suggest that you assign Part G of the Supplementary Exercises at


this time. Have students do the problems orally. [See the discussion
inTC[l-44, 45] and on TC[l-50] on identifying instances.]

Here is a quiz which tests the ability to apply the principles listed on
pages 1-54 and 1-55, and the ability to identify the principles used.
Fill in the blanks to make true sentences, and write the name of the
principle of which the true sentence is a consequence.
85 + = 85 [ ]

X 1 = 798 [

846 + _= 982 + 846 [ ]

4. 68 + (73 + 15) = 68 + +15 [ 1

5. 91 X (84 + 12) = 91 X (12 + ) [ ]

6. 105 + (31 X 76) = (31 X ) + 105 [


]

7. (35 + 64) X 75 = 35 X 75 + X 75 [
]

8. 61 X (7 + 9) = (7 + _)X61 [ .]

9. 1 X (62 + 37) = 1 X 62 + x 37 [
_]
10. [3 + (7 + 9)] X 5 = [(7 + 9) + 3] X
>.*,
"r
Answers for quiz.

1. 0. paO 2. 798, pml 3. 982, cpa 4, 73, apa


5. 84, cpa 6. 76, cpa 7. 64, dpma 8. 9, cpm
9. 1, idpma 10, 5, cpa

TC[l-55]
[1.06] [1-55]

The Left Distributive Principle for Multiplication over Addition [idpma]

5 X 8 + 5 X 12 = 5 X (8 + 12)

7X3y = 7X3+7x|-
The Principle for Adding [pa 0]

6+0 = 6

87
J + = 87^
The Principle for Multiplying by 1 [pm 1]

19 X 1 = 19

86. 73 = 86.73 X 1

The Principle for Multiplying by 0_ [pm 0]

7X0 =

= 318 X

EXERCISES
A. Each of the following sentences is an instance of one of the prin-

ciples for the numbers of arithmetic. Tell which principle.

1. 7+0 = 7 2. 3X1 = 3 3. 4 + 1 = 1+4


4. 5X8 = 8X5 5. = 9X0 6. 17 = 17X1
7. 4 X (5 + 8) = (5 + 8) X 4 8. {5 + 8) X 4 = (5 X 4) + (8 X 4)

9. 3. 59 X 8. 61 X 7. 32 = 3. 59 X (8. 61 X 7. 32)

10. 758 X (321 + 684) = 758 X 321 + 758 X 684

11. 67 X 531 + 33 X 531 = (67 + 33) X 531

12. (85 + 3) X (17 + 12) = (85 + 3) X 17 + (85 + 3) X 12

13. 8 X (17 X 9) + 12 X (17 X 9) = (8 + 12) X (17 X 9)

X X
14.

15.
(97 + 35)

94
II
35
X 1 = 1 X
(9 + 2) = (9 + 2)

94
-tI-
35
(97 + 35)

16. (
2
Y
3
12
+ t)
2
X 6 = T
3
X 6 +
1
T
2
X 6

[More exercises are in Part G, Supplementary Exercises .


]
[1-56] [1.06]
vU
'1"*
^1^ vl^
'I" 'I-

As we have said earlier, the principles for the numbers of

arithmetic are very useful in computing because they give you


different ways of reaching the same result. This means that you
have different ways of carrying out a given computational task.
AVith practice you will learn how to pick the easiest way. Here
are some exannples.

Example 1_. 493 + 39 + 7 = ?

The uninspired way of simplifying '493 + 39 +7' consists of two steps,

First, add 39 to 493; second, add 7 to this sum.

Hard way
493 + 39 + 7 = 532 + 7

= 539.

A more sensible approach is to observe that it is easier to add 7 to

493 than it is to add 39 to 493, and that once you've done this, it is

easy to add 39 to the sum.

Easy way

493 + 7 + 39 = 500 + 39

= 539,

But, wait a minute ! How were you able to predict that you would get
the same answer the easy way? You could have based your predic-
tion on the principles. For example:

493+ 39+7 =493 +(39 +7) [apa]

493 + (39 + 7) = 493 + (7 + 39) [cpa]

493 T (7 + 39) = 493 + 7+39 [apa].

Or, you could have used the principles like this:

493 + 39 + 7 = 7 + (493 + 39) [cpa]

7 + (493 + 39) = 7 + 493 + 39 [apa]

7 + 493 + 39 = 493 + 7 + 39 [cpa].


[1-57]

1/68

The argument suggested by lines 7b, 6b, and 5b on page 1-56


deserves some discussion here. How much of the following discus-
sion you wish to bring to your students is up to you.

Lines 7b and 5b are instances of the apa and, so, are consequences
of this principle. Line 6b, on the other hand, is a consequence of
the cpa although not an instance of it. Our first job is to see just why
line 6b is a consequence of the cpa. Now, the sentence '39 + 7 = 7 + 39'
is an instance of the cpa. So, what we need to show is why line 6b is a
consequence of this sentence. Due to our interpretation of = ', this '

instance of the cpa asserts that '39 + 7' and '7 + 39' are names for a
single number. Hence, given any sentence S, if we can obtain a sen-
tence S* by replacing a '39 + 7' in S by a '7 + 39', then S* is a conse-
quence of the two sentences '39 + 7 = 7 + 39' and S. Now, line 6b can
be obtained from the sentence '493 + (39 + 7) = 493 + (39 + 7)' by
replacing the second '39 + 7' in this sentence by a '7 + 39'. So, line
6b is a consequence of 39 + 7 = 7 + 39' and '493 + (39 + 7) =
'

493 + (39 + 7)'. But, again due to our interpretation of = ', the '

latter sentence is true on logical grounds alone ['*A thing is itself."].


So, we say that line 6b is a consequence merely of '39 + 7 = 7 + 39'.

The substitution procedure described above can be formalized by the


following scheme. [You may, if you like, read the as ' '

'therefore'. ]

[cpa] [A thing is itself.]

39 + 7 = 7 + 39 49 3 + ( 39 + 7) = 49 3 + ( 39 + 7)
493 + (39 + 7) = 493 + (7 + 39)

[You may read this aloud as: 39 + 7 = 7 + 39, 493 + (39 + 7) =


493 + (39 +7), therefore [by the substitution rule] 493 + (39 + 7) =
493 + (7 + 39).]

Lines 7b, 6b, and 5b on page 1-56 are the building blocks for an
argument to show that the sentence '49 3 + 39 + 7 = 493 + 7 + 39' is a
consequence of the apa and the cpa. The mortar is supplied by the
substitution procedure illustrated above. The formal argument nmight
be set up like this.

493 + (39 + 7) = 493 +(7 +39) 493 + 39 + 7 = 493 + (39 + 7)

493 + (7 + 39) =493 + 7 + 39 493 + 39 ^ 7 = 493 + (7 + 39)

49 3+ 39 + 7 =493 + 7 + 39
TC[l-56]a
aritr
[1-57]

1/69

In abbreviated form, the formal argument looks like this,

B = C A = B
C = D A = C

This argument contciins two applications of the substitution rule.


(1) B = C, A = B, therefore A = C.

(2) C = D, A = C, therefore A = D.

Since lines 7b and 5b are consequences of the apa, and line 6b is a


consequence of the cpa, this argument shows that '493 + 39 + 7 =
493 + 7 + 39' is a consequence of these two principles.

The argument displayed above may be interpreted as consisting of


two applications of a logical principle called the transitivity of equality
["If A = B and B = C then A = C. "] Notice that this principle is justi-
fied by the substitution procedure which we have been discussing [and
the latter is valid by virtue of our interpretation of = ']. Another '

logical principle called the symmetr y of equality [*'If A = B then


B = A- "] is also justified by the sanie substitution procedure, taken
together with the reflexivity of equality ["A = A. '*
or "A thing is
itself."] This is how :

A = B A = A
B = A

TC[l-56]b
[1-56]

As.
aritt
[1.06] [1-57]

or even like this:

493 + 39 + 7 = 39 + 493 + 7 [cpa]

39 + 493 + 7 = 39 + (493 + 7) [apa]

39 + (493 + 7) = 493 +7+39 [cpa].

In any case the principles assure you that

493 + 39+7 = 493 +7 +39.

Example 2. 987 X 593 + 593 X 13 = ?

Hard way

987 X 593 + 593 X 13 = 585291 + 7709

= 593000.

Easy way

593 X (987 + 13) = 593 X 1000

= 593000,

The principles justify the easy way by enabling us to show that

987 X 593 + 593 X 13 = 593 X (987 + 13).

Here is one way of showing this:

987 X 593 + 593 X 13 = 987 X 593 + 13 X 593 [cpm]

987 X 593 + 13 X 593 = (987 + 13) X 593 [dpma]

(987 + 13) X 593 = 593 X (987 + 13) [cpm].

And, here is another way:

987 X 593 + 593 X 13 - 593 X 987 + 593 X 13 [cpm]

593 X 987 + 593 X 13 = 593 X (987 + 13) [Mpma.].


[1-58] [1.06]

These examples and the exercises which follow help you to learn how
to use the principles to check short cuts. With practice, the checking
procedure becomes almost automatic, and even suggests short cuts.

vU ^1^ v'^
'1^ '1^ 'p

B^. Each sentence below suggests a short cut in carrying out a conn-
putation. Your job is to justify the short cut by showing that the
sentence is a consequence of the principles. In connection with
Example 1 above, we have given three samples of how to show
that the sentence:

493 + 39+7 = 493+7+39


is a consequence of the principles. In Example 2, we gave two
samples of how to derive the sentence:

987 X 593 + 593 X 13 = 593 X (987 + 13)

fronn the principles. For each of the following sentences, it is

enough if you give just one derivation of the sentence from the
principles.

1. 5x(9x|-)= 1x5X9
2. 43 X 31 + 31 X 57 = 31 X (43 + 57)

2 3 2 3
3. 8|- + ^ = 8 +(y + ^)

4. 799 + (58 + 1) = 799 + 1+58


5. ^ X (85 +85) = |- X (1 + 1) X 85

6. 9 X 75 + 75 = 75 X (9 -: 1)

7. 5|- + 3| M5 + 3) +(i + |)

8. 9 X 38 + 70 X 38 + 38 X 2 1 = 38 X (2 1 + 9 + 70)

9. 27+13 = (2 + 1) X 10 + (7 + 3)

[Recall that ' 13' is an abbreviation for *(! X 10 + 3)'. ]

10. 25 + 3 X (7 + 25) = 25 X (1 + 3) + 3 X 7

11. 27 X 13 = (2 X 1) X (10 X 10) + (2 X 3 + 7 X 1) X 10 + 7 X 3


[1-59]

"ng as

of

1/70

Answers
derivations

1. 5 x(9 X |)
for Part B.
.
]

= (5
[Your students

X9) X
may suggest other correct

[apm]
N
I
(5 X9) X x{5 X9) [cpm]

|- X{5 X9) =|-X5X9 [apm]

2. 43 X 31 + 31 X 57 = 31 X 43 + 31 X 57 [cpm]
31 X 43 + 31 X 57 = 31 X (43 + 57) [idpma]

2 3 2 3
8| + ^ = 8+{| + |) [apa]

2 2
['Sy' is an abbreviation for '(8 + y)'.]

4. 799 + (58 + 1) = 799 + (1 + 58) [cpa]


799 + (1 + 58) = 799 + 1+58 [apa]

X (85 + 85) = X (85 X 1 + 85 X 1) [pml]


Y "I-

| X(85 X 1 + 85 X 1) =
-I
X [85 X(i + 1)] [idpma]

i- X [85 X 1 + 1)] = X 1 + 1) X 85] [cpm]


(
J [(

y X [(1 + 1) X 85] = y X(l + 1) X 85 [apm]

9 X 75 + 75 = 9 X 75 + 75 X 1 [pml]
9 X 75 + 75 X 1 = 75 X 9 + 75 X 1 [cpm]
75 X 9 + 75 X 1 = 75 X (9 + 1) [Mpma]

TC[l-58]a
[1-58]
[1-59]

ng as
of

1/71

7. si + 3| = (si + 3) + [apa]
I
(5i + 3) + = [5 + (|- + 3)] + [apa]
I I
[5+(i-l-3)]+
I =[5 +(3+ 1)]+ I
[cpa]

[5 +(3+ ^)]+ I = [(5 + 3) + i] + [apa]


I
[(5 + 3) + i] + = (5 + 3) + (i + |) [apa]
I

8. 9 X 38 + 70 X 38 + 38 X 21 = 38 X 9 + 38 X 70 + 38 X 21 [cpm]
38 X 9 + 38 X 70 + 38 X 21 = 38 X (9 + 70) + 38 X 21 [idpma]
38 X (9 + 70) H 38 X 21 = 38 X [(9 + 70) + 21] [Mpma]
38 X [(9 + 70) + 21] = 38 X [21 + (9 + 70)] [cpa]

38 X [21 + (9 + 70)] = 38 X (21 + 9 + 70) [apa]

9. 27 + 13 = [(2 X 10 + 7) + 1 X 10] + 3 [apa]

[(2 X 10 + 7) + 1 X 10] + 3 = [2 X 10 + (7 + 1 X 10)] + 3 [apa]

[2 X 10 + (7 + 1 X 10)] + 3 = [2 X 10 + (1 X 10 + 7)] + 3 [cpa]

[2 X 10 + (1 X 10 + 7)] + 3 = [(2 X 10 + 1 X 10) + 7] + 3 [apa]

[(2 X 10 + 1 X 10) + 7] + 3 = (2 X 10 + 1 X 10) + (7 f 3) [apa]

(2 X 10 + 1 X 10) + (7 + 3) = (2 + 1) X 10 + (7 + 3) [dpma]

10. 25 + 3 X{7 + 25) = 25 + (3 X 7 + 3 X 25) [idpma]


25 + ( 3 X 7 + 3 X 25) = 25 + ( 3 X 25 + 3 X 7) [cpa]
25 + (3 X 25 + 3 X 7) = (25 + 3 X 25) + 3 X 7 [-pa]

(25 + 3 X 25) + 3 X 7 = (25 X 1 + 3 X 25) + 3 X 7 [pml]


(25 X 1 + 3 X 25) + 3 X 7 = (25 X 1 + 25 X 3) + 3 X 7 [cpm]
(25Xl+25X3) + 3X7 = 25X(l + 3) + 3X7 [Mpma]

TC[l-58]b
[1-59]

ng as
of

1/72

11. [27 X 13 = (2 X 10 + 7) X(l X 10 + 3) [abbreviation]


(2 X 10 + 7) X (1 X 10 + 3)

= 2 X 10 X(l X 10 + 3) + 7 X(l X 10 + 3) [dpma]

2X10X(1 X10 + 3) + 7X(1 X10 + 3)


= [2 X 10 X (1 X 10) + 2 X 10 X 3] + [7 X (1 X 10) + 7 X 3] [idpma]

[2 X 10 X (1 X 10) + 2 X 10 X 3] + [7 X (1 X 10) + 7 X 3]

= 2X10X(lX10) + 2X10X3 + 7X(lX10)f7X3] [apa]

2X10X(1X10) + 2X10X3 + 7X(1 XI 0) + 7X3


= 2X10X(1X10) + [2X10X3 + 7X(1X 10)] + 7x3 [apa]

2 X 10 X (1 X 10) + [2 X 10 X 3 + 7 X (1 X lO)] + 7X3


= 2 X 10 X 1 X 10 + [2 X 10 X
( 3) + 7 X 1 X 10] + 7 X 3 [apa]

2 X 10 X 1 X 10 + [2 X (10 X 3) + 7 X 1 X 10] + 7 X 3

= 2 X (10 X 1) X 10 + [2 x( 10 X 3) + 7 X 1 X 10] + 7 X 3 [apm]

2 X (10 X 1) X 10 + [2 X (10 X 3) + 7 X 1 X 10] + 7 X 3

= 2 X (1 X 10) X 10 + [2 X (3 X 10) + 7 X 1 X 10] + 7 X 3 [cpm]

2 X (1 X 10) X 10 + [2 X (3 X 10) + 7 X 1 X 10] + 7 X 3

= 2X1X10X10 + [2X3X10 + 7X1X10] + 7X3 [apm]

2X1X10X10 + [2X3X10 + 7X1X10] + 7X3


= 2 X 1 X (10 X 10) + [2 X 3 X 10 + 7 X 1 X 10] + 7 X 3 [apm]

2 X 1 X (10 X 10) + [2 X 3 X 10 + 7 X 1 X 10] + 7 X 3

= (2 X 1) X( 10 X 10) + (2 X 3 + 7 X 1) X 10 + 7 X 3 [dpma]

TC[l-58]c
[1-58]

These
to U'"

P
[1-59]

ng as
of
1/73

Answers for Part C.

1. 32 2. 16 3. 47 4. 118 5. 2100

6. 1925 7. 250 8. 170 9. 1000 10. 750

II. 34000 12. 42000 13. 2500 14. 15. 30.6


16. .02 17. 9 18. 1/27 19. 1 20. 1

After discussing Part D, ask: Could someone \who didn't know the
dpma get along by knowing the Mpma [and the cpm] ?

Answer for Part D.


39 X 83 + 39 X 17 = 83 X 39 + 17 X 39 [cpm]
83 X 39 + 17 X 39 = (83 + 17) X 39 [dpma]
(83 + 17) X 39 = 39 X (83 + 17) [cpm]

Exercise 15 nnay cause some questioning on the part of the students as


to how one interprets a symbol such as '102%'. We interpret *102%' as
a name for an operation, and *102% of 30* as a nunneral, analogous say,
to *the square of 4', 'the (common) logarithm of 2', 'half of 8', and
'the square root of 9'. In each case we are naming the nunnber which
results when a certain operation is applied to a certain number. The
result of applying the square rooting operation to the nunnber 9 is the
square root of 9 [another name for this number is '3']; the result of
halving 8 is half of 8 [another name is '4']; the value of the logarithm
function for the argument 2 is the logarithm of 2 [a name for an approxi-
mation to this number is '0. 3010']; the result of squaring 4 is the square
of 4; and the result of applying the operation 102% to the number 30 is
102% of 30. In describing an operation on numbers one must tell how
to find the result of applying it to a number. For example, one finds
the square of a number by multiplying the number by itself; and one
finds the half of a number by dividing the number by 2. In particular,
one finds 102% of a number by multiplying the number by 102/100.
Note that 'of is not just an alternative to 'X'. Rather, it refers to
applying an operation to a number. For excimple, to find the square
of 4 does not mean to multiply square by 4; to find the logarithm of
2 does not mean to multiply logarithm by 2. Similarly, to find 102%
of 30 does not mean to multiply 102% by 30. [It does mean to mul-
tiply 30 by 1.02.]

TC[l-59]
[1.07] [1-59]

C. Simplify, using as many short cuts and doing as little writing as


possible. [Be prepared to justify your short cuts on the basis of
the principles,

1. 7 X(8 X |-) 2. 9 X(|- X8)

3. 16 + (27 + 4) 4. 75 + (18 + 25)

5. 88 X 21 + 21 X 12 6. 19t X 12 + 19t X 88
4 4
7. 15 X (10 + 1) + 85 8. 65 + 7 X (10 + 5)

9. 30 X 31 + 70 10. 29X25+ 25

11. 68 X (^ X 1000) 12. 84 X 500

13. 67 X 25 + 33 X 25 14. (18 X 75) X (93 X 0)

102% of 30 77 X 01 + 23 X
15. 16. 0. 0. 1.
j^
17.
3X9+8X9 18.
7
11 7X15 + 12X7
19.
5X7+4X7 20.
6X5+7X5
9X3+4X9 5X8+ 25
Part H, Supple'mentary Exercises. ]

D. Suppose you want to solve the problem:

39 X 83 + 39 X 17 = ?

You know a short cut which is based on the left distributive principle

for multiplication over addition. Remem.ber Stan and Al ? Suppose


Stan hasn't told Al about the left distributive principle. Could Al
justify the short cut on the basis of the other principles? In other

words, could Al derive the sentence:

39 X 83 + 39 X 17 = 39 X (83 + 17) ^-

from just the principles mentioned on page 1-54? How would he do it?

1.07 Principles for the real numbers . --The principles you have learned
about in the last section refer to the system of numbers of arithmetic.
It is natural to ask if there are similar principles for the system of real
numbers. we asked this question about
If just the nonnegative real num-
bers, the answer would be 'yes' [Why?]. But, the system of real num-
bers includes the negative numbers as well; so, for example, the question
whether multiplying by a real number distributes over adding requires
some investigation.
'

[1-60] [1-07]

(I) For each sentence below, simplify both sides and label the
sentence 'True* or 'False'.

(a) ^3 + "2 = "2 + '3

(b) (*6 + -4) + "7 = ^6 + ("4 + *7)

(*7 + -12) X "3 = ^7 X "3 + '12 X "3


(c)

(d) ("6 X -4) X "7 = ^6 X (-4 X "7)

(e) "3 X "2 = "2 X "3

(f) (-3 X -5) + M = (-3 4 M) X (-5 + M)


(g) -5 + "6 = "6 + "5

(h) -3 + ("12 + -5) = ("3 + -12) + -5

(i) -6 X M + -8 X -4 = ("6 + "8) X M


(^9 + -3) X -5 = *9 + (-3 X -5)
(j)

(k) -3 X -12 X -5 = "3 X {'12 X "5)

-5 X -6 = -6 X -5
(1)

(m) -2 + M + "6 = -2 + (*4 + "6)

(n) -2 X C4 X -6) = {-2 X M) X 'b

(II) Classify as many as possible of the true sentences in (I) as


instances of these five principles for real numbers .

Connmutative principle for addition

Commutative principle for multiplication

Associative principle for addition

A.ssociative principle for multiplication

Distributive principle for multiplication over addition

(III) Make up one more instance of each principle mentioned in (II) for

real numbers. How many of your five new sentences are true?
[1-61]

1/74

It important that the students actually simplify both sides of the


is
sentences in (I), They are here investigating whether the principles
apply to operations on real numbers. Of course, the principles do
apply, and the reason that they do can be found in the nature of the
physical interpretations we gave for the operations. The interpre-
tations were so selected as to lead to definitions of addition and
multiplication consistent with the principles.

«».'-'

^t^

Here are answers for (I).

(a) l = *1 True (b) "•2


+ *7 = '6 + *3
*9 =*9 True

(c) -5 X-3 = "21 + *36 (d) "24 X *7 = *6 X "28


"15 = "15 True "168 = "168 True

(e) "6 = "6 True (0 *i5 + *4 = n X-1


*19 = "1 False

"11 = "11 True (h) "3 + "17 = "15 + "5


(g)
"20 = "20 True

(i) "24 + "32 = "14 X -"4


(J)
"•6 X -5 = *9 + *1

"56 = "56 True "30 = *24 False

(k) *36 X "5 = "3 X *60 (1) *30 = ""SO True


"180 = "180 True

(m) *2 + "6 = "2 + "2 (n) "2 X "24 = "8 X "6


"4 = "4 True *48 = *48 True
;U

TC[1 -60, 61]a


[1-60]

(I)
[1-61]

•ed

1/75

Answers for (II) [page 1-60].

Commutative principle for addition: (a), (g)

Connmutative principle for multiplication: (e), (1)

Associative principle for addition: (b), (h), (m)


Associative principle for multiplication: (d), (k), (n)

Distributive principle for multiplication over addition: (c), (i)

Another approach to developing rules for miultiplication is the


following.

One can show that if one accepts the usual rule for the addition of
real numbers, and wishes to define inultiplication of real numbers
in such a way that the Mpma holds then the pmO naust hold. More-
over, if one also wishes the pm*ltohold then it must be the case
that ~1 X ~1 = *1 To prove the first, note that since by the usual
.

rule for addition,


0+0 =

it follows that, for ajiy real number a,

aX(0+0) = aXO
and, if the Mpn\a is to hold, that

aXO+aXO = aXO.
But, with the usual rule for addition, this can be the case only if

a X = 0.

To prove the second, note that, since, by the rule for addition,
*1 + "1 = 0. it follows that

"1 X(*l + "1) = "1 X


= 0,

and, if the idpma is to hold, that


"1 X ""1
+ "1 X "1 = 0.
Hence*, if the pm*l is to hold,
"1 + "1 X "1 = 0.

But, with the usual rule for addition, this can be the case only if
"1 X "1 = *1

TC[l-60, 6l]b
[1-61]

red

1/76

[In the same way one can establish any such equations as:
"3 X "1 = *3, ""Z X "1 = ~2, etc.

So, one wishes to define nnultiplication of real numbers in such


if
a way it will be associative as well as satisfying the Mpma and
that
the pm*l, it nnust, for example, be the case that
*3 X "2 = 'S X (*2 X "1)

= (*3 X +2) X "1

= [*3 X (+1 + -^1)] X "1

= X "1
C3 + '•3)

= -^6 X "1

= "6.

And if, further, nnultiplication is to be commutative, it nnust be the


case that "2 X *3 = ~6.]

The answer to both questions in Exercise 3 of Part A on page 1-61 is:


Yes. The reason for the second answer is that, just as is suggested
in Part D on page 1-59 in the case of numbers of arithmetic, so here,
in dealing with real numbers, each instance of the Mpma for real
numbers is a consequence of an instance of the dpnaa and three
instances of the cpm. Students should illustrate this in class by
justifying, say, '"3 X (*7 X "12) = ~3 X *7 + "3 X "12* by application of
the cpm, and dpma.

TC[l-60, 6l]c
[1-60]

(I) F

1
[1.07] [1-61]

Correct lists for Exercise II, page 1-60, might have appeared
as follows.

The Commutative Principles for Addition and Multiplication

^ 3 + "2 = "2 + ^
3 ^ 3 X "2 = "2 X * 3

"5 + "6 = "6 + "5 "5 X "6 = "6 X '5

The Associative Principles for Addition and Multiplication

("6 -!- -4) + *7 = *6 + ("4 + "7) ("6 X "4) X "7 = ^6 X ("4 X *7)

"3 + ("12 + "5) = ("3 + "12) + "5 "3 X '12 X "5 = "3 X ("12 X "5)

"2 + "4 + "6 = "2 + (M + "6) "2 X (M X "6) = ("2 X *


4) X "6

The Distributive Principle for Multiplication over Addition

("^7 + "12) X "3 = *7 X '3 + "12 X "3

"6 X ^4 + "8 X M = ("6 + "8) X *4

EXERCISES
A. 1. Make up two more instances of each of the five principles

and check each of your ten sentences.

2. The fact that the real numbers have these same five prop-
erties as the numbers of arithmetic is important because
they give us short cuts in working with the real numbers.
The other properties of the numbers of arithmetic -
-those
expressed by the principle for adding and the principles
for multiplying by 1 and by 0--also hold for the real num-
bers. That is, there is a principle for adding the real
number 0^, a principle for nnultiplying by the real number *
1,

and a principle for multiplying by the real number 0. Make


up three instances of each of these three real number prin-
ciples and check each of the nine sentences.

3. Is there a left distributive principle for multiplication over


addition for the system of real numbers? What must be
your answer if you accept the five principles mentioned at

the top of this page?


[1-62] [1.07]

B. For each of the following sentences fill in the blank to nnake it

true, and tell what principle the true sentence illustrates.


"
1. *5 X = "12 X *5 2. -3 X "7 = X

3. ("3 + '7) X *5 = "3 X "5 + X "5

4. "6 X (-8 X ) ^ ("6 X *8) X *15

5. *9 + '3 + *5 - *9 + ( + *5)

6. "2 X "7 4 '2 X = "2 X ("7 + ""17)

7. *9X =0 8."3X "3

9. "5 + = '5 10. "5 X -7 X =

11. = X 12. -!- =

13. 4X(7+ )=4X7+4X


14. "25 = +0 15. "25 = X

C. Use the principles numbers to help you simplify the


for the real

following numerical expressions. Do as little writing as you can.

1. ("18-^ + 85) + 3.i 2. (212 + "473) + 473

3. 7X8 + 8X-9 4. -7X8 + 2X-7


5. ("185 X i) X 3 6. ( ~ X 5. 792) x "5

7. ("793 X |) X 8. '12 4 876 + 512


f
9. "18X57 + 57X68 10. "3X15789-6+15789.6x13
11. "16^ + 86 + "3-^ 12. '972. 75 X -37 + "27. 25 X "37

13. (892 X i) X ("37 X 0) X 18 14. 27 X "


1 1 7 + 17 X 27

15. "8 + (*3 + ^8) 16. (-3 + "7) + (*3 + *7)

17. "453 + (*624 + '453) 18. ("587 + "426) + ('587 + M26)

19. ^ X 45. 678 X 16 + "45.678

20. 891. 23 X 386.9 -^


(j x "891. 23) x (7 x *386.9)
[1-63]

1/77

Ans^wers for Part B.

1. "12, cpm 2. "7, cpm 3. *7, dpma 4. *"15, apm


5. "3, apa 6. *17, Mpma 7. 0, pmO 8. *1, pm'^1

9. 0, paO 10. 0, pmO


11. A numeral for any number may be written in the blank, pmO
12. 0, paO
13. A numeral for any number may be written in the blanks, Mpma
14. "25, paO 15. It is not possible to make the sentence true.

•J,
'J-

Answers for Part C.


1. 70 2. 212 3. "16 4. "70

5. "185 6. 5.792 7. "793 8. 1376


9. 2850 10. 157,896 11. 66 12. 37000
13. 14. "2700 15. *3 16.

17. *624 18. 19. 20.

TC[l-62]
[1-63]

\
[1-62]

B.

1/78

The purpose of the Exploration Exercises on pages 1-63 through 1-66


is to prepare the student for the work on inverse operations in Section
1.08. Students should go through these exercises rapidly. Note that
the Exploration Exercises refer to operations with numbers of arith -
metic and not to operations with real numbers. We are trying to
develop the general notion of inverse operations. It will be applied to
real numbers in Section 1.09.

Answers for Part A.

1. 3 2. 9 3. 7 4. 40 5. 8
6. 4 7. 10 8. 4 9. 20 10. 45
11. 283 12. 13. 253 185.9
"i 14.

Answers for Part B [on pages 1-63 and 1-64],

1. 7 2. 20 3. 12 4. 900 5. 9
6. 8423 7. 7 8. 19 9. 29 10. 876
11. 82 12. 2431 13. 8 14. 49 15. 24
16. 68 17. 7 18. 15 19. 3 20. 72
In each of the exercises from 21 through 28 a numeral for any number
can be used in the first blank, and a copy of this numeral must be
written in the second blank of the exercise.
29. 7 30. subtracting 192 31. 15
32. 97 33. subtracting 54 34. subtracting 71

Answers for Part C [on page 1-64).

1. 3 2. 4 3. 2 4. 86 5. 12
6. 42 7. 40 8. 99

TC[l-63, 64]
[1.07] [1-63]

EXPLORATION EXERCISES
A. Guess the number.

1. Multiply it by 8, and you get 24.

2. Double it, and you get 18.

3. Add 10 to it, and you get 17.

4. Add 23 to it, subtract 3 from the sum, and you get 60.

5. Add 12 to it, subtract 12 from the sum, and you get 8.

6. Multiply it by 5, multiply the product by 2, and you get 40.

7. Multiply it by 7, multiply the product by -= , and you get 10.

8. Alultiply it by 9, multiply the product by 5-, and you get 4.

9. Multiply it by -f- , multiply the product by 5, and you get 20.

10. Add 98 to it, subtract 98 from the sum, and you get 45.

11. Add 974 to it, subtract 974 from the suin, and you get 283.
2 2 1
12. Add 81-^ to it, subtract 81 rr- from the sum, and you get 57-^.

13. Multiply it by -=^ , multiply the product by 78, and you


get 253.

14. Add 2. 7 to it, subtract 2. 7 from the sum, and you get 185.9.

B. Fill in the blanks to make true sentences.

1. {
+ 4) - 4 = 7 2. {
+ 9) - 9 = 20

3. (
+ 8) - 8 = 12 4. (
+ 5) - 5 = 900

5. {
+ 51) - 51 = 9 6. (
+ 79) - 79 = 8423

7. (7 + 5) - 5 = 8. (19 + 3) - 3 ^

9. (29 + 2) - 2 = 10. (876 + 15) - 15 =

11. (82 + 769) - 769 = 12. (2431 + 1893) - 1893 =

13. (9 + 8)- =9 14. (73 + 49)- =73

(continued on next page)


'

[1-6.1] n.o7]

15. (57 + 2,4) - = 57 16. (68 + 25) - = 25

17. (6 + _) - 7 = 6 18. (93 + ) - 15 = 93

19. (19 + _ )
- 3 = 19 20. (72 + ) - 72 = 72

21. (9 + ) - =9 22. (34 + ) - =34

23. (53 + _ ) - - 53 24. (117 + ) - =117

25. ( + 5) - 5 = 26. ( + 71) - 71 =

27. ( + 48) - 48 = 28. (


+5i)-5i =

29. If you add 7 to a number, you can get back the number by
subtracting fronm the sum.
Example 1 . 91 + 7 = 98 and 98 - = 91.

E-vcainple 2. (43 + 7) - = 43.

30. If you add 192 to a number, you can get back the number
by from the sum.

Example . (4375 + 192) - = 4375.

31 . If you want to undo the result of adding 15, subtract


from the sum.

Example . (869 + 15) - = 869.

32. Subtracting undoes what adding 97 did.

33. undoes what adding 54 did.

34. undoes what adding 71 did.

Fill in the blanks to make true sentences.

1- ( X 5) X i ^ 3 2. ( X 7) X i = 4

3. (
X 9) X i = 2 4. ( X 35) X J^ = 86

5. (
xi)X4=12 6. ( X i) X 7 = 42

1- (
xi)x8 = 40 8. ( xi^)xil=99
[1-65]
1/79

Answers for Part C [on pages 1-65 and 1-66],

9. 6 10. 7 11. 8 12. 173 13. 15


14. 36 15. 8 16. 92 17. 2 18. 8
1 1 1
19. 7 20. 13 21. 22. 23.
9 7 11
1 1 1
24. 25 • 26. [none] 27.
77 17 3
1
28. 29 . 8 30. 19
19
In Exercises 31 and 32 a numeral for any number can be used in the
first blank, and a numeral for the reciprocal of that number must be
written in the second blank. [In connection with Exercises 31 and 32,
the teacher might ask whether there is a nunnber such that a numeral
for it could be written in both blanks of the exercise and make the
sentence true.

In each of the Exercises 33 and 34, a numeral for any number may be
written in the first blank, and a copy of this nunneral should be written
in the second blank. [In each of these Exercises (33 and 34), a student
could get true sentences by writing different nunaerals for the same
number in the two blanks; however, we want to prepare the students
for the convention in Unit 2 that blanks of the same shape must be
filled by copies of the same numeral.]

35. 36.
Y
In the bracketed paragraph, answers to successive questions are:
Q-; V; ~yft 2' ^ ' y^s; has no reciprocal; there is no number
such that tim.es that number is 1.

[In connection with the bracketed paragraph, you nnay find it desir-
able to ask other similar questions. For example, 'What is the
reciprocal of 0.25?' and 'What is the reciprocal of 2983/2983?*
[Answers: 4, 1.] Since you are here discussing numbers of arith-
metic, you will, of course, not ask 'What is the reciprocal of ~2?'.]

37. 53 38, 39. 17 40. 17 41.


12 9
42. 43. 44. ^ 45. reciprocal, 7
46. a numeral for any number can be used in the first blank, and
a copy of this numeral should be written in the second blank.

TC[l-65, 66]
[1.07] [1-65]

9. 6 X 3) X |- = 10. (7 X 2) X ^ =

_
11. 8 X 5) X |- 12. (173 X84) X =
g^
13. 15 X j) X 3 = _ 14. (36 X |-) X9 = .

15. 8 X 1") X9 16. (92 X :^) X79 =

17. 6 X 0. 5) X .= ^ 18. (72 X 0. 125) X = 72

19. 63 X y) X = 63 20. (56 X pr) X = 56

21. 51 X9) X_ = 51 22. (38 X 7) X = 38

23. 85 X 11) X .
= 85 24. (77 X 6) X = 6

25. 29 X 17) X = 29 26. (583 X 0) X _ ~ 583

27. 81 X ) X 3 - 81 28. (751 X ) X 19 = 751


1
29. 52 X )
xi = 52 30. (847 X ) X
19
= 847

31. 48 X ) X = 32. (31 X ) X _= 31

33. X 7) X y = 34. ( X g-)X8 =

35. f you multiply a number by 7, you can get back the number
by multiplying the product by .

Example 1. 5 X 7 = 35 and 35 X = 5.

Example 2. (31 X 7) X = 31.

36. If you multiply a number by — , you can get back the number
by multiplying the product by .

Ex amp le 1. 12 X 7- = 2 and 2 X 12.


D

Example 2_. (54 X 7-) X 54.

[ Note : Since 2 X 0. 5 = 1, 0. 5 is called the reciprocal of Z_. Since


0.5X2=1, 2 is called the reciprocal of 0. 5 . In general,

pairs of numbers whose product is 1 are called reciprocals,


and each is the reciprocal of the other. What is the recip-
rocal of 9? Of |- ? Of 0.37? Of ? Of 1? Is there a
I
number of arithmetic which does not have a reciprocal?
What is the reciprocal ofO? 0X? = 1.]

(continued on next page)


[1-66] [108]

37. If you multiply a number by ,


you can get back the
number by multiplying the product by the reciprocal of

53.

38. If you multiply a number by IZ, you can get back the
number by multiplying the product by .

39. If you want to undo the result of naultiplying by ,

multiply the product by the reciprocal of 17.

Example . (93 X ) X jy ::: 93.

40. Multiplying by undoes what multiplying by the


reciprocal of 17 did.

Example . (42 X jj) X = 42.

41. Multiplying by undoes what multi-


plying by 9 did.

42. Multiplying by undoes what multiplying by the


reciprocal of 9 did.

43. Dividing by undoes what multiplying by 7 did.

44. Multiplying by undoes what multiplying by 7 did.

45. Multiplying by the of undoes what


multiplying by 7 did.

46. Dividing by does what multiplying by the recip-


rocal of does

1.08 Inverse operations . --Here is a grade school subtraction


problem :

13 - 4 = ?

Everyone knows that the result is 9. But, let's look a bit more
deeply into how we know that this is the correct answer.
In Part B of the preceding Exploration Exercises you learned
that

subtracting 4 undoes what adding 4 did.


[1-67]
1/81

A classroom device for introducing the notion of an operation as a


set ofordered pairs is the following. Ask students for examples of
adding 4. Then record the examples in a column like this:

3 + 4 = 7

2 + 4 = 6

6 + 4 = 10

12 + 4 = 16
0+4 = 4
8 + 4 = 12.

Next, erase the '+ 4 =' from each sentence, leaving a list of pairs.
Thus, the pair (3, 7) corresponds with an exannple of adding 4. If
we think of the operation adding 4 as somehow made up of all its
"examples", it is easy to think of the operation as the set of pairs
corresponding with these exanaples.

Here is a classroom device which helps develop the idea of the


inverse of an operation. Tell students you are going to list the pairs
in multiplying by 5. Start the list as follows.

(3, 15), (7, 35), (4, 20)

Then, list the first number of a pair and ask the students to give the
second. Also, list the second number and ask thenn for the first.

TC[l-67, 68]b
[1-67]

•^ing
1/80

Here we introduce a device which should be quite helpful in getting


students to understand the idea of operations and their inverses.
In particular, it should make very clear what happens when they
conae to subtraction of real numbers in Section 1.09.

We regard an operation such as adding 4 as a set of ordered pairs in


which the second component in each ordered pair is the sum of the
first component and 4. Thus, the expression 'adding 4' is a noun
whose referent is an operation. Notice that the student must begin
developing the concept of an ordered pair, and this is a painless way
to do it. For example, he knows that the pair (3, 7) belongs to the
operation adding 4, but that the pair (7, 3) does not. So, it must be
the case that (3, 7) is not equal to (7, 3). The operation subtracting
4 [some of the pairs in this operation are listed on page 1-68] has a
very close connection to the operation adding 4. In fact, if you were
to interchange the components of the ordered pairs in one operation,
you would get the ordered pairs in the other. We say that the inverse
of adding 4 is^ subtracting 4. This use of the word 'inverse* is pre-
cisely the use we make of the word in connection with functions and
their inverses. [See Unit 5.] [Also, see pages 1-103 through 1-109.]

Ifthe subsection "Operations "is read aloud then, for example, the
sentence which begins on line 3 of this subsection should be read as:
it says that the phrase subtracting-four and the phrase
the-inverse-of-adding-four are names of the sanne thing.
[See TC[l-L]a.]

Answers for Part A [on page 1-68],

Pairs that belong to the inverse: (ll|,3|-) (17|-,9|-) (9^, r)

(19^.11^^) (15|1, 7i) (109||. lOli) (44.3,35.9)


2 7
The operations are: adding 8^, and subtracting 8|^.

TC[l-67, 68]a
[1.08] [1-67]

This statement tells us what \we mean by 'subtracting 4'. Subtracting


4 is just the operation you carry out to undo what adding 4 did. So,

if you wish to subtract 4 from 13, you must try to find the number
to which 4 was added to get 13. This number is 9, since 9 + 4 = 13.

So, we say that 13-4 = 9.

Let us rewrite the subtraction statement:


13-4=9
as :

(9 + 4) -4 = 9.

Do you see from this last statement that subtracting 4 undoes what
adding 4 did? A shorter way of expressing this idea is to say that

subtracting 4 is the inverse of adding 4.

Similarly, we can say that


(3582 + 649) - 649 = 3582
because
subtracting 649 is the inverse of adding 649.

OPERATIONS
Let us examine a statement such as :

subtracting 4 is the inverse of adding 4


and see what it tells us. It says that 'subtracting 4' and 'the inverse
of adding 4' are names of the same thing. What is this thing?
'subtracting 4' names an operation, and so does 'adding 4'. Let's
look at these operations.
When you add 4, you start with a number and get to a number.
For example, you may start with 3 and end with 7, start with and
end with 4, start with 15 and end with 19, start with 937 and end

with 937 + 4. You can think of the operation adding 4 as the whole
set of such pairs of numbers. Here is a list of just some of these
pairs. Would it be possible to list all of thenn ? Could the pair (9, 13)

(3,7) (0,4) (15, 19) (937, 937 + 4)

(19,23) (81.85) (52,56) (45,49)

(l|, 5|) (7.5.11.5) (9.03,9.03 + 4) (7.11)


{i-&8] [1. 08]

be --ncaiided in feis list? Ho-a- about (4-D, 45)? How aboxtt (11, 7)?

Notice 1±iat (11, 7) ccnald not be incliaded in this list because


11 -J- 4 jt 7. Buz, tiiere is a related operation to wMcb (11, 7) does
belodjg. T^^ai is it?
Here is a list of some ai the pairs 'sc-Mcb belong to the operation

=-jibtr acting 4. J As before, it -woxild not be possible to list all the


paxTS -sdaich belong to the operation. ]

<il. 71 (7, 3) (4, m U9, 15) (937 ^ 4. 937) \

(23, 19) 485, 81;) (56, 52) (49, 45)

(5|. l|) (li.5, 7.5) (9. 03 ^4, 9.03)

±jEai33ine the two lists -we have given. Sxippose someone put
another j«ir into the first list, l^'onld that suggest a pair 'which
yoa conld ^fst into the second list? Give three more pairs -which
ycm conld put into the list for adding 4. Give three -which you
could put into the list for subtracting 4.

:^o

A. Here is a list oi some of the pairs of numbers which belong to


a c-eTtazj2. operation. Alongside it, make a list of some of the
pairs which belong to the inverse sf that operation.

/ <4' ^4* *4' ^4^


\
4' ^h

m^, i9i) (7i isfi)

(10l|, li>9||-) (35.9, 44.3) /

Can yzfu. guess what these two operations are called?


[1-^
be inch

1/82

Adding 7: (5. 12) (3, 10) (15, 22) (35, 42) (96, 103) (10^,, 17^)
(31.2, 38.2) ... .

Subtracting 7: {12, 5) (10, 3) (22, 15) (42, 35) (103, 96) (17|-, lo|)
(38.2, 31.2) .. . .

Answers for Part C.

1. Multiplying by 5.

2. (35, 7) (10, 2) (5, 1) (11.5, 2. 3) (0, 0) (15, 3) (25, 5) (45, 9)

(5000, 1000) (20, 4) (30, 6)

3. They are inverse operations.


4. Same list as for Exercise 2.

5. The operations dividing by 5 and nnultiplying by the reciprocal


of 5 produce the same result; hence 'dividing by 5' and
'multiplying by the reciprocal of 5' must be names for the sanrie
operation.

TC[l-69]
[1.08] [1-69]

]B. Make two lists of paii-s of numbers which illustrate that

subtracting 7 is the inverse of adding 7.

C_. 1. Here is a list of eleven pairs which belong to a certain


operation. Can you guess what operation this is?

(2, 10) (1, 5)

(0, 0) (3, 15)

(9, 45) (1000, 5000) (4, 20)

(6, 30)

2. Make a list of eleven pairs which belong to the operation


dividing by 5.

3. V/hat do your answers to Exercises 1 and 2 suggest about


dividing by 5 and the operation referred to in Exercise 1 ?

Make a list of eleven pairs which belong to the operation


multiplying by the reciprocal of 5.

What do your answers to Exercises 2 and 4 suggest about


the operations dividing by 5 and multiplying by the recip-
rocal of 5 ?
[1-70] [1.08]

Here are two lists of pairs which belong to operations. Note


that the second operation is the inverse of the first.

adding 5 $ ^ subtracting 5
^
i 1

(0, 5) (15, 20) (5, 0) (20. 15)

(31, 36) <4' 4^ (36, 31)

(7, 12) (91, 96) (12, 7) (96, 91)

(40, 45) (27, 32) (45, 40) (32, 27)

(22, 27) (10, 15) (27, 22) (15, 10)

(2.41, 7.41) (7.41, 2.41)

(36, 41) (96, 101) (41, 36) (101, 96)

(1004, 1009) (1009, 1004)

(2, 7) (45, 50) (7, 2) (50, 45)

How could we use them to do a problem? Consider the problem:

91 + 5 = ?

This is a problem in adding 5. Go to the adding 5 list, and look


for the pair whose first number is 91. What is the second num-
ber in this pair? Is it the result of adding 5 to 91 ?

Now, try a subtraction problem:

45 - 5 = ?

This is a problem in subtracting 5. Go to the subtracting 5 list,

and look for the pair whose first number is 45. V/hat is the
second number in the pair? Is it what you get when you subtract
5 from 45 ?
[1-71]

hough
1/85

Answers for quiz.

I. 1. (7, 9) 2. (0. 0) 3. adding 10 4. (8, 1.6)

5. {70, 64) 6. (0. 0) 7. adding 10

II. 1. (2. 4) 2. (1, 4) 3. {0, 0) <4' ''b

For recreation you can introduce more "complicated" operations.


For example, you can ask for pairs which belong to multiplying by
2 and then adding 3. Such pairs are (1, 5), (7, 17), (0, 3), and
(100, 203). Or, if you wish, you can give some pairs and ask for
a name of cin operation which contains such pairs.

Answers for Part A of the Exploration Exercises [on page 1-72].

1. *4 2. % 3. "8 4. *4 5. "1

6. "6 7. "9 8. 9. "173 10. *286

TC[1-71, 72]c
[1-71]

hough
1/84

Here quiz which covers the idea of an operation as a set of pairs


is a
and the idea of the inverse of an operation.

I. Multiple -choice . Draw a loop around the correct answer.


1. The jeration adding Z contains .

(A) (3, 6) (B) (5. 3) (C) (7. 9) (D) (l/2, 1/4)

The operation multiplying by 2 contains .

(A) (8, 10) (B) (30, 15) (C) (0, 0) (D) (5, 3)

3. (5, 15) belongs to


(A) adding 5 (B) dividing by 3
(C) multiplying by l/3 (D) adding 10

4. Dividing by 5 contains .

(A) (8. 1.6) (B) (10, 5) (C) (12, 2.2) (D) (0, 5)

The inverse of adding 6 contains .

(A) (6, 1) (B) (7. 13) (C) (50, 46) (D) (70. 64)

6. The inverse of multiplying by 7 contains .

(A) (14, 21) (B) (0, 0) (C) (7, 0) (D) (5, 35)

The inverse of the inverse of adding 10 is .

(A) multiplying by l/lO (B) subtracting 10


(C) adding 10 (D) adding l/lO

II. 1. What pair belongs both to adding 2 and to multiplying by 2 ?


2. What pair belongs both to adding 3 and to multiplying by 4 ?
3. What pair belongs both to multiplying by 7 and to the inverse
of multiplying by 7 ?

4. What pair belongs both to multiplying by 3 and to adding 5?

TC[1-71, 72]b
[1-71]

'hough

k.
1/83

Answers for exercises at the top of page 1-71.

20 (2) 7.41 (3) 91 (4) 22 (5) 12 (6) 31


(1)

Answers for Part D.


1. 24 2. 162 8.4

Answers for Part E [on pages 1-71 and 1-72J.

1. 5843.5 2. 2238.25 3. 6824.37 4. 3205.91

Answers for Part F [on page 1-72].

1. 743 2. 156 3. 1 59

Part G on page 1-72 presents an interesting consequence of regard-


ing operations as sets of pairs. Make this a class exercise and
handle it as follows. Ask students to name the operation some of
whose pairs are
(1, 2), (3, 6), (15, 30). (70, 140)
[Describe the operation further by stating that each second number
is the double of the first.] Students will respond that this operation
is called 'multiplying by 2'. Then remark that this is not the namie
you had in mind. [Some student might suggest 'doubling', which is
an appropriate name also.] Try to elicit the name 'dividing by 1/2'.
Then point out that the names 'miultiplying by 2' and 'dividing by 1/2'
are synonyrris. They stand for the same thing. Now, proceed with
Part G. The operation described therein has several names --
'multiplying by 1', 'dividing by 1*, 'adding 0', 'subtracting 0'. [A
name which we shall introduce later [page 1-88] is 'sanneing'. A
technical name for this operation is 'the identity operation'.] The
point of this exploratory work is to develop the idea that an operation
can have many names. This is exceedingly important in connection
with subtracting real numbers, for the major point there will be that,
for example, the operation which we call 'subtracting *4' is also
called 'adding the opposite of *4', Since we know how to add the
opposite of *4 [that is, since we know the pairs which belong to add-
ing the opposite of *4], we must also know how to subtract "^4.

TC[1-71, 72]a
[1.08] [1-71]

Use these lists to solve the following problems [even though


you can do the problems without the lists]:

(1) 15 -f 5 = ?

U) 2. 41 + 5 = ?

(3) 96 - 5 = ?

(4) 27 - 5 = ?

(5) 7+5 = ?

(6) 36 - 5 = ?

'1- 'r

D. pairs which b el(ang to the

(3, 22) (5, 24) (7, 26) {2, 21) (0, 19) (26. 45)

(21,40) (6.2, 25.2) (8.4, 27.4) (Zj, 21 |-) (19,38)

(181, 200) (25.2, 44.2) (162, 181) (27.4, 46.4)

• • •

Use just this list to solve the following problems.

1. 5 + 19 = ? 2. 181 - 19 = ?

3. 26 - 19 = ? 4. 27. 4 - 19 = ?

E. Here is a list of pairs which belong to the operation adding


9734. 62.

(15578.14, 25312.76) (3205.91, 12940.53)


/
/ (5843.52, 15578.14) (12940.53, 22675.15) (6824.37, 16558.99)

1
(11972.87, 21707.49) (2238.25, 11972.87)

(16558.99, 26293.61)
[1-72] [1.08] ;

Use just this list to solve the following problems.

1. 15578.14-9734.62=? 2. 11972.87-9734.62=?

3. 16558. 99 - 9734. 62 = ? 4. 12V40. 53 - 9734. 62 = ?

F. Here is a list of pairs which belong to the operation multiplying


by 789. \

(59, 46551) (0, 0) (743, 586227) (46551, 36728739)

(586227, 462533103) (156, 123084) (1, 789)

(123084, 97113276) (789, 622521)

Use this list to solve the following problems.


^
1. 586227 V 789 = ? 2. 123084 X
789
3. 789-^789 = ? 4. 46551 X the reciprocal of 789

G. Think of the set of all pairs of iiumbers of arithmetic in which


the first number is the same as the second nunaber. V/hat
operation is this?

EXPLORATION EXERCISES
A. Guess the number.

1. Add * 3 to it, and you get ^^7.

2. Add "4 to it, and you get "^2.

3. Add *5 to it, and you get ~3.

4. Add "7 to it, add "7 to the sum, and you get ""4.

5. Add "9 to it, add *9 to the sum, and you get "1.

6. Add '5 to it, add "5 to the suin, and you get "6.

7. Add "2 to it, add '' Z to the sum, and you get '9.

8. Add * 3 to it, add "3 to the suin, and you get 0.

9. Add "10 to it, add "'lO to the sum, and you get "173.

10. Add to it, add to the sum, and you get "286.
\
[1-72]

Us.

1/86

Answers for Part B [on pages 1- 73 and 1 -74] •

1. *5 2. "1 3.
"8 4. *4 5. "73

6. *824 7. *7 8. n2 9. "3 10. "7

11. *9 . 3 12. "5 8 13. *35 14. "21 15. "65

16. "72

In Exercises 17 and 1 8, a nunaeral for any number may be written in


the first blank, and a nunneral for the opposite of this number must
be written in the second blank. In each of the exercises from 19
through 22, a numeral for any nunnber can be used in the first blank,
cind a copy of this numeral must be written in the second blank of the
exercise.

23. '8' in each blank 24. '*5' in each blank

[In the bracketed paragraph, the answer to the question is: 0.]

25. *3 26. "3 27. "9 28. *9

29. *"7' in each blank 30. '*5' in each blank


31. "5 32. m 33, "17 34. *7

35. the opposite of "2 36. '*7* in each blank


37. '"24' in each blank 38. its opposite

[In answering Exercise 36, an alternative is to write 'the opposite of


"7' in the first blank, and '*7' in the second blank. Similarly, for
Exercise 37.]
'I-

Section 1, 09 [page 1-75] on subtraction of real numbers provides the


payoff for regarding operations as sets of ordered pairs. If we know
what the operation adding '5 is [that is, if we know the ordered pairs
which belong to it], then we know what the inverse of this operation
is. It is simply the set of ordered pairs which are obtained by inter-
changing the components of each ordered pair in adding ~5. We use
the expression 'subtracting "5 as a name for the operation which is
'

the inverse of adding "5. But, if we examine the pairs in the operation
subtracting "5 we find that this operation is the operation adding *5.
So, subtracting "5 is precisely the same operation as adding *5.

TC[l-73, 74, 75]


[1.08] [i-73]

B. Fill in the blanks to inake true sentences.

1. ( + *2) + '2 = ^5 2. (_
+ ^3) + "3 = '1

3. ( + "4) + *4 = "8 4. (_
+ -8) + *8 = ^4

5. ( + *6) + "6 = "73 6. (_


+ "51) + ^51 = ^824

7. (""7 + "3) + ^3 = 8. r 12 + ^5) + "5 =

9. ("3 + ^8) + "8 = 10. ("7 + "15) + "15

11. (*9. 3 + "2. 1) + "2. 1 = 12. ("73 + "58) + ^73 =

13. (^14 + "35) + = ^4 14. (-17 + "21)+ ="17

15. ("31+^65)+
^65) + ="31
"31 16. (^72 + nOO) + =^100

17. (*19 + )++


)
=^19
"19 18. ("35 + )+ _='35

19. ( + "72) + "72 = 20. (


+-18) + "18

21. (
+'57)+"57=
+ '57) + "57 = 22. ( + 0) + = _
23. If you add "8 to a number, you can get back the number by
adding to the sum.

Example 1. "2 + "8 = "6 and "6 + = '2.

Example 2. ("384 + "8) + = "334.

24. If you add "5 to a number, you can get back the number by
adding to the sum.

Example l^. "11 + "5=:"16 and "16 + ="11.

E.xample 2. ("384 + "5) + = "384.

[Note: Since "73 + "73 = 0, "7 3 is called the opposite of "73 . Since
"73 + "73 = 0, "73 is called t he o pposite of "7 3 . In general,

pairs of real nnmbors whose sum is are called opposites ,

and each is the opposite of the other. How do we know that


"10 is the opposite of "10? Because our rule for addition

tells us that "10 + "10 = 0. Each real number has an oppo-


site. What is the opposite of 0?]
(continued on next page)
[1-74] [1.08]

25. If you add to a number, you can get back the number
by adding the opposite of *3 to the sum.

26. If you add to a number, you can get back the number
by adding the opposite £f ^ to the sum.

27. If you add "9 to a number, you can get back the number
by adding the opposite of to the sum.

28. If you add *9 to a number, you can get back the number
by adding the opposite of to the sunn.

29. If you want to undo the result of adding ,


add the
opposite of "7 to the sum.

Example 1 . (*36 + ) + ^7 = *2>b.

Exainple Z . (*36 + )
+ the opposite of '1 - *36.

30. Adding undoes what adding 5 did.

Example . (*42 + '5) + = ''42.

31. Adding the opposite of undoes what adding "5 did.

32. A-dding the opposite of undoes what adding *11 did.

33. Adding the opposite of is the inverse of adding '17.

34. Adding the opposite of is the inverse of adding *1

35. Adding -
is the inverse of adding "2.

36. The inverse of adding "7 is adding .

Example . ("382 + "7) + = "382.

37. The inverse of adding *24 is adding .

Example . (""15 + *24) + = *15.

38. The inverse of adding a real number is adding


[1.09] [1-75]

1 . 09 Subtraction of real numbers .


- -As with nunnber s of arithmetic,
we shall use the word 'subtracting' in naming the operation which is
the inverse of adding a given real number. So, for example, we shall
say that

subtracting 5 is the inverse of adding 5

and that
subtracting '^4
is the inverse of adding *4.

Now, let us solve a subtraction problem:

^9 - "5 = ?

This is a problem in subtracting ~5. V/e caii solve this problem by


thinking in terms of the operation adding "5 and its inverse.

y/, '//.', / / / // / ,_
/'/ ?
adding "5 W .1 subtracting 5 /
t'-l

We go to the list for subtracting "5, and look for the pair whose first

number is *9. Its second number is *14. So,

*9 - "5 = M4.

Check these subtraction problems.

(a) ^22 - "5 = *27 (b) "7 - "5 = "2

(c) -3 - "5 = *2 (d) ^4 - "5 = *9


[1-76] [1.09]

EXERCISES
A. 1. Make a list of ten pairs which belong to the operation
adding "3.

2. Use these ten pairs to make a list of pairs which belong to


the inverse of adding ~3, that is, to the operation subtract-

ing '3.

3. Make a list of ten pairs which belong to the operation


adding * 3.

4. Examine the lists you get in Exercises 2 and 3. V/hat do

they suggest about the operations of subtracting ~3 and


adding *3? Could you have predicted this without making
the lists ?

B^. 1. Here is a list of pairs which belong to the operation adding *7.
Use it to construct a list of pairs which belong to the opera-
tion adding "7.

Wadding *7
\; /j adding "7 \'\

(^, ^9)X
("3, '4)

("10 ,
"3)

("7. 0)

(17 ,
"10)

(^7. *14)
/

/
/
/

Solve each problem by using the list for adding *1 . Then,


solve each problem again by using the list for adding "7.

(a) M - '7 = ? (b) - "7 = ?

(c) "1 - *7 = ? (d) "7 - ^7 = ?


[1-77]

1/87

Answers for Part A.

1. (no, *7) (^3, nO) (-8,-11) (*28, *25) ("2,-5) (*2, "1) etc.

2. (*7, nO) (*10, n3) ("11, "8) (*25, *28) (-5.-2) ("l.^Z) etc.
3. Same list as for Exercise 2 may be used.
4. They suggest that these two operations are the same.

Answers for Part B [on pages 1-76 and 1-77].

1. Here is a list of pairs that belong to the operation adding "7:


(*16, *9), (*9, ^Z), (*8, n), (*4, -3), (-1, -8), ("3, "10), (^7, 0),

(0, -7), Cn. ^4). (-10, -17), (*6. -1). (*M, "7). (*3. "4).

2. [In Exercise 2 of Part B students are to find the answer to each


question in two ways; first using the list for adding *7, then using
the list for adding "7. Sample: To find *4 - *1 since subtracting ,

*? is the inverse of adding *7, we look for an ordered pair belongs


ing to adding "^7 whose second component is *4. The only such
pair is ("3, *4). So, "^4 -"^7 = -3. Alternatively, since subtracting
'''7
is adding "7, we look for an ordered pair in adding "7 whose
first component is '4, The only such pair is (*4, "3). So,
^T^*7 = -3.]

(a) -3 (b) -7 (c) -8 (d)

(a) *5 (b) *8 (c) *93 (d) "14

(e) "28 (f) *36 -107 (h) "22


(g)
I

TC[l-76, 77]
[1-76]

A.
[1.09] [1-77]

3. Now solve these probleins all of which involve subtracting *1_.

(a) nz - ^-7 = ? (b) M5 - ^7 = ?

(c) *100 - -^7


^ ? (d) "7 - •'7 = ?

(e) '21 - -^7 = ? (f) '43 - n = ?

(g) "100 - •'7


= ? (h) "15 - '7 = ?

vl- O^ v'^
'1^ '1^ '1^

The problem :

"9 - "8 = ?

involves subtracting "^8. Subtracting *8 is the inverse of adding '^8.

And, as you have seen, the inverse of adding '^S is adding "8, that
is,
"^8
the inverse of adding is adding the opposite of '^8.

So,

subtracting "^8
is the same as adding the opposite of *8.

Hence, the subtraction problem ;

"9 - ""S = ?

can be converted into the addition problem :

"9 + "8 = ?

Therefore,
"9 - "8 =: -9 + "8 = "17.

o, o^ o^
'r '\- 'C

Solve these subtraction problems.

Sample. *8 - "3 = ?

Solution. Since subtracting 3 and adding '^3


are both the
inverse of adding "3, subtracting "
3 is the

same as adding *3. So,


^8 - "3 = "8 + *3 = m.
(continued on next page)
[1-78] [1.09]


1. nz - "3 = ? 2. "5 - n = ? 3. "11 - ^2 = ?

4. "6 - "9 = ? 5. "8 - no = ? 6. 8 - 10 = ?

7. 15 - 3 = ? 8. 15 - "3 = ? 9. "15 - 3 = ?

D. Simplify as quickly as possible.

1. "3 - ^2 2. n2 - ^3 3. M - ^7
4. ^8 - ^8 5. "3 - ^8 6. 4 - 17

7. *5 - -2 8. ^2 - -10 9. "12 - -12

10. -3 - M 11. -3 - 4 12. -5 - ^7

13. -7 - "4 14. -9 - -11 15. "12 - -12

16. "14 - "7 17. - 14 - 6 18. 6 - -14

19. 8 - 2 20. 8 - 1 21. 8 -

8 - -1 8 - "2 24. 8 - -4
22. 23.

25. "5 - "3 26. -5 - -2 27. -5 - -1

28. "5 - 29. "5 - 1 30. -5 - 2

31. 8 - -3 32. 5 - "4 33. 12 - "8

34. 7 35. 36. 13 - "5


- 9 11 - 7

37. "17 - "Z 38. 7 - -3 39. 9 - 18

40. "5 - 6 41. "5 - "7 42. -7 - -5

[More exercises are in P art


Part I, Supplementary
Supr ry Exercises
Exercises. .
]

±i- 1. Suppose '9' and '12' are numerals for numbers of arith-
metic. Which of the following is a numeral?

(a) 12 - 9 (b) 9 - 12

2. Suppose '9' and '12' are numerals for real numbers. V/^hich

of the following is a numeral?


(a) 12-9 (b) 9 - 12
[1-79]

<l as

1/88

Ans\wer8 for Part C [which begins on page 1-77],

1. *12 - "3 = *12 + *3 -^ *15 2. "5 - *4 = "5 + "4 = "9


- "6 - "9 = "6 + *9 = *3
3. "11 - *12 = "11 + "12 _
4.

5. *8 - *10 = *8 + "10 = "2 6. 8 - 10 = 8 + "10 = "2


"
7. 15 - 3 = 15 + 3 = 12 8. 15 - "3 = 15 + *3 = 18
-
9. "15 - 3 = "15 + "3 = 18
nI^
'1^

Ans wers for Part D »

i. n 2. "1 3. "13 4. 5. "5

6. "13 7. *7 8. *22 9. '24 10. "7

11. "7 12. "12 13. "3 14. *Z 15.

16. "7 17. "20 18. 20 19. 6 20. 7

21. 8 22. 9 23. 10 24. 12 25. "2

26. "3 27. "4 28. "5 29. "6 30. "7

31. *11 32. 9 33. 20 34. "2 35. 4


36. 18 37. "15 38. 10 39. "9 40. "1

41. *2 42. "2

vt>
••»'•

Ans wers for Part E >

1. (a) 2. ( a). (b)

»'

TC[l-78]a
[1-78]
[1-79]

e; as

1/89

Here is a quiz which tests the ability to add and subtract real numbers.

I. Simplify.
1. *3 - -2 Z. *1 - *8 3. "5 + *8 4. -3 -

5. 6 - "8 6. 12 - no 7. "2 - *8 8. +7 - -7

I. Fill the blanks to nnake true sentences.

I. *^ - = *7 2. "3 + = *3 3. - *1 = ^
4. "6 - = "6 5. *3 + = -2 6. "7 - = *1

III. Multiple -choice . Draw a loop around the correct answer.

1. If you subtract a positive number from a negative number,


the result is
(A) a positive number (B) (C) a negative number

I add a real number to a positive nunnber and get as the


sum. If I subtract the real number from the positive number,
the result would be .

(A) a positive number (B) (C) a negative number

Mary and John each pick a real nunnber. The difference of


Mary's number from John's is a negative number. What is
the difference of John's number from Mary's?
(A) a positive number (B) (C) a negative number
'1^

Answers for quiz.

I. 1. *5 2. "1 3. '3 4. "3

5. 14 6. 2 7. "10 8. n4
II. 1. *2 2. % 3. n4 4.

5. "5 6. n4
III. 1. a negative number 2. a positive number
3. a positive number

TC[l-78]b
[1-78]
[1-79]

^g as
[1-78]

1/90

Indiscussing the second part of the Solution in Part F you [or one of
your students] should fill in the missing steps:

5 + *3 + "8 + 7 + "2
I
V apa
= (5 + 3) + (~8 + 7) + "2

= (5 + 3) + (7 + -8) + "2 r*

\
apa
= (5 + 3 + 7) + "8 + "2 '

[
apa
= (5 + 3 + 7) + ("8 + -2)
j

I
apa
= 5 + (3 + 7) + ("8 + "2). '

The second step is an application of the commutative principle for


addition and the others a^e applications of the associative principle
for addition. In doing the exercises of Part F, students should not
be required to go through a detailed justification like the above.

»»-

Answers for Part F [on pages 1-79 and 1-80].


1. "3 2. "22 3. "4 4. 15 5. "32
6. "27 7. 15 8. "1 9. "8 10. 2

In Exercises 11-16, students will not know that, for exanaple,


-(~r + *5) = *2 + "5. [We hope they will know this after they have
finished Exercise 1 3 on age 1-86.] So, students will solve Exercise
j"

J 3, for example, as follows.

"2 - ("8 + *Z) - (-3 + -2) - ("5 + "8) + (? - "9)

= "2 - "6 - "5 - "13 + *2


= "2 + *6 + *5 + *13 + *2

etc.

11. "12 12. "5 13. "7 14. "1 15. 24

TC[l-79, 80]
[1.09] [1-79]

F - Since subtracting a real nunnber is precisely the same thing as


adding its opposite [ the principle for subtraction] , every sub-
traction problem can be converted into an addition problem.

Sample . 5 - '3 + "8 + 7 - "2

Solution . 5 - "3 + "8 + 7 - ""2

= 5 + ^3 -f
'8 + 7 + "2,

At this point you can simplify either by working fronn


left to right:

5 + '3 + ~8 + 7 + ~2

8 + '8 + 7 + '2

+ 7 + "2

7 + "2

5,

or by using the commutative and associative principles


for addition of real numbers to change the order:

5 + ""3
+ "8 + 7 + "2

= (5 + 3 + 7) + ('8 + "2)

15 + "10

5.

Simplify .

1. -3 + 8 - "2 - "7 + -3 2. -5 + "3 - *7 + 9 + "16

3. '2 + "4 + "3 4 "5 - "6 4. 8 - "5 + "3 - "6 - "7 -


""8

5. 3 - 15 + "2 - 17 + "1 6. "1 + "5 - 6 - 3 - 10 + "2

7. "2 -f
"3 - "5 - "5 + 10 8. "4 - "6 + "3 - 8 - '9 + "1

9. 11-15+7-9-3 + 1 10. 1 - "2 - 6 - -9 - "3 - 7

(continued on next page)


[1-80] [1.10]

U. -9 - ("2 + "5) 12. -3 - (-4 - -6)

13. (-1 + -3) - (-5 - -8) 14. (6 - 12) - (4 -


9)

15. "2 - CS + "2) - (-3 -i -2) - {-5 + "8) + (-7 - -9)

[More exercises are in Part J, Supplementary Exercises .


]

1.10 Opposites --In studying the system of numbers of arithmetic


.

and the system of real numbers, we have noticed certain similarities.


For example, we have seen that the nonnegative numbers act like the
numbers of arithmetic with respect to addition. V/hat are some other
similarities ?

There is, also, an important difference between the two systems.


It concerns the operation of subtraction. Can you tell what this dif-
ference is ?

If you pick two numbers of arithmetic, a first number and a

second number, can you subtract the first number from the second
number? Can you subtract 9 from 11? Can you subtract 6 from 2?
Notice that there are cases in which you cannot subtract the first
selected number from the second. So, we say that subtraction is
'

not always possible in the system of numbers of arithmetic.

Is subtraction always possible in the system of real numbers?


Can you subtract ""9
from "^11? ""6 from ""2? "5 from ~8 ? To subtract
a real number is to add its opposite. Since addition is always pos-
sible, and since each real nunaber has an opposite, subtraction is

always possible in the real number system.


i\n important fact about the real nunibers is that, for each real

number, there is a real number [the opposite of the first] which


when added to the first gives the sum 0. This fact is expressed by
the principle of opposites - -a nun^ber plus its opposite is 0. You
will see in Unit 2 that it follows irom. the principle of opposites
together with other principles that if the sum of a first number and
a second number is 0, the second number is the opposite of the
first.

V/hat real nunriber is the opposite of ~4? It is *4, because


"4 + M = 0. V/hat is the opposite of *4? [V/hat principle tells you
that if a first number is the opposite of a second then the second
is the opposite of the first?]
[1-81]

1/91

S.
The last two paragraphs on page 1-80 foreshadow the ''algebraic"
point of view of Unit 2. The following discussion is intended to
show you the direction in which we are going, and so enable you to
correct any misconceptions on the part of your students.

At this point students may have the idea that the opposite of a
positive number is the corresponding negative number, the opposite
of a negative number is the corresponding positive number, the
opposite of is 0, and that, "consequently" each real number has
one, and only one, opposite. This is, of course, correct. But, the
point of view which they need to acquire is subtly different fronn this,
and can be expressed by saying [see note at bottom of page 1-73] that
for each real nunnber there is one, and only one, real number
which when added to the first gives the sum 0,
and that this second real number is called 'the opposite' of the first.

Part of this point of view is embodied in the principle of opposite


which> in the language of Unit 2 can be stated as :

(1) for each real number x, x + -x = 0.

The remainder can be stated as:

(2) for each real nunnber x, (and) for each real number y,
if X + y = then y = — x.
These two statements are made, son:iewhat less formally, in the
first of the two paragraphs under discussion. As remarked there,
(2) is a consequence of (1) together with the cpa, the apa, and the
paO. [Here is a sketchy proof: If x + y = then x + y + -x = + — x,
so y + X + -X = -X + 0, y + (x + -x) = — x + 0, y + = -x + 0, and
y = -X. Don't give this to your class. They will get a more adequate
treatment in Unit 2 after they have learned to appreciate it. Any
attempt to foreshadow this now will be a source of confusion.] It is
innportant that students grasp the content of (2) as preparation for
exercises like those in Part C on page 1-84. [But, of course, they
are not prepared to see (2) as written above. ]

The answers to the questions in the last paragraph are:


(1) "What is the opposite of *4?"--"We have just mentioned
that "4 + *4 = 0. So, by the cpa, we know that *4 + "4 = 0. Hence,
~4 is the opposite of *4.

(2) "What principle . . . first ? "--Basically, the cpa is the


principle we need.

TC[l-80]
[1-81]
[1-80]

11

1/92

The operation oppositing is a singulary operation; the operations add-


ing and nnultiplying are binary operations. A binary operation is one
which is applied to an ordered pair of elements in a set; a singulary
operation is one which is applied to a single element in a set. Other
singulary operations are finding the square root of, finding the recip-
rocal of, finding the logarithnn of, adding 2, multiplying by 7, etc.

The three examples near the bottom of page 1-81 suggest that the
operation oppositing could also be called 'subtracting from 0'. We
don't call it this because in our development of the real number system
oppositing is more primitive than subtracting. In fact, subtracting is
defined in terms of oppositing [as expressed by the principle for sub-
traction].

TC[1-81]
[1.10] [1-81]

THE OPERATION OPPOSITING


Finding the opposite of a real number is an operation just aj

adding ^^4 or multiplying by 2 are operations. Here are some


pairs of real numbers which belong to the operation oppositing .

f^ '-.^
x'nWVnxxVxxn^
:, oppositing

rs, -5) ('4, M) (0,0) (7. -7)

(100 - 2, 2 - 100) ("5,^5) ('4. -4)

('3 - -2, "2 - ^3)

List some pairs which belong to the inverse of oppositing . What


can you say about oppositing and its inverse?
Just as we have signs for other operations [x, +, -, -f], we
should like to have a sign for oppositing. We might use a *-j!r*.

Then '-iV
*4' would mean the opposite of *4. So, v.^e would have
statements like :

T!:r*4 = "4,

ii "11 = m,
M - "3 = '4 + t!r "3,

- *5 :- 1^*5,

- "12 = -ii-
"12,

- (*3 - '8) = iV (^3 - '8).

The last three exajnples suggest the notation that nnost people

use for oppositing. write a minus sign. So, following


It is just to

this practice, we shall write '-*4' [read as 'the opposite of posi-

tive four'], to mean the opposite of "^4, and '- "11' when we mean
the opposite of "11. Similarly, the expression:

- (*3 - -8)
[1-82] [1-10]

which is a name for the opposite of (*3 - "8) can be simplified to:

-ni)
which is finally simplified to:

"11.

EXERCISES
A. Use this new notation to write a name for the opposite of each

given number.

1. -^8 - '7 [Answer: -(^8 - "7)]

2. "3 + "5 3. ^5 - "6 4. "2 X "3 + "5

5. 6 - 2 6. 9 - 15 7. 3 - 5 + 6 X (5 + 3)

B^. Read aloud first, and then simplify.

Sample . -{12 - 9) + (9 - 12)

Solution . Read as 'the opposite of the difference of 9

from 12, plus the difference of 12 from 9*.

-(12 - 9) + {9 - 12)
= -^3 + 3

= "3 + "3

= "6.

1. -*8 2. -"15 3. -(^5 - -7)

4. -(^2 X -3) 5. -(55 - 30) 6. -55 + 30

7. -("3 X-7) 8. -(10 - 12) 9. -no -i 12

10. -no - 12 11. --"7 12. -(5 - 7) + (5 - 7)

13. -3 14. --7 + -*7 15. --(-3 - -^3)

16. -{93 - 97) 17. --{84 - 89) 18. --(100 - 101)

19. -{^9 4 -7) 20. -{"9 + ^7) 21. -("3 + -6)

22. -{983 -f 729 - 604) -i (983 + 729 - 604)

23. (3572 - 4871) +(4871 - 3572)


[1-83]

1/93

Answers for Part A,

2. -("3 + *5) 3. -("5 - -6) 4. -("2 X "3 +-5)


5. -(6 - 2) 6. -{9 - 15) 7. -[3 - 5 X (5 + 3)]

'IT-

As a general rule, the "scope" of a symbol for a singulary operation


is taken to be as small as makes sense. Thus,
* -~2 + *3' means the same as '(-"2) + *3'
' -~2 X *3' means the same as '(-"2) X *3'

'-CZ + "3) X ^5* means the same as •[-(*2 + "3)] X -^5'


-"lO - -12' means the same as '(-^lO) - "12'

etc.

Another example, not yet familiar to your students, is the squaring


operation denoted by .

• ""3
X "22* means the same as ' *3 X ("2^)'

'"3 + '"2^'
means the same as * "3 + {*Z^)\

When, in applying two singulary operations, the operator for one is


a prefix and the operator for the other is a suffix as in -3^', one '

must either use grouping symbols or adopt a convention. Thus, by


convention,
' -3^' means the sanne as ' -(3^)'

[rather than '(-3)2'].

-I"

Answers for Part B.

1. the opposite of positive 8; "8

2. the opposite of negative 15; *15

3. the opposite of the difference of negative 7 from positive 5; "12

4. the opposite of the product of positive 2 by negative 3; *6

5. the opposite of the difference of 30 from 55; "25

TC[l-82]a
[1-82]

which
[1-83]

1/94

6. the sum of the opposite of 55 and 30 [or: the opposite of 55,


plus 30]; ~25

7. the opposite of the product of negative 3 by negative 7; "21


[A good exercise to follow Exercise 7 of Part B would be:
-"3 X-7.

Students should see that this is equivalent to the expression


given in Exercise 7, and they should contrast this situation
with the one in Exercises 5 and 6. ]
8. the opposite of the difference of 12 fronn 10; *Z

9. the sunn of the opposite of positive 10 and 12; 2

10. the difference of 12 from the opposite of positive 10; "22

11. the opposite of the opposite of positive 7; *7

12. the sumof the opposite of the difference of 7 from 5, and the
difference of 7 from 5;
[You may have difficulty getting your students to read the
expression as we have suggested here [particularly in view of
the Solution of the Sample]. At first attempt a student will
probably say 'the opposite of the difference of 7 from 5, plus
the difference of 7 from 5*. Accept this, but then ask, 'Is
this principally a problenn of addition? Or, is it a problem of
subtraction?' [In an expression like the one given in Exercise
12, the '+' is considered the principal operator.] The students
will probably agree that it is primarily a problem of adding two
differences. Then ask, 'What do we call the answer to an
addition problem?' After someone replies, 'the sum', you may
ask whether someone could read the given expression in such a
way as to indicate to the listener that it is^ a problem in finding
a sum. You may need to copy -(5 - 7) + (5 - 7)' on the board;
'

then help the class read it by pointing to the '+' and saying 'the
sum of. Next point to the -' in -(5 - 7)* and say 'the oppo-
' '

site of, etc. [This same technique nnay be necessary for


obtaining a careful reading of the expressions given in Exer-
cises 22 and 23. ]]
13. the opposite of the opposite of the opposite of negative 3; *3

14. the sum of the opposite of negative 7 and the opposite of


positive 7;

TC[l-82]b
[1-82]

which 1
1/95

15. the opposite of the opposite of the difference of the opposite of


positive 3 from negative 3;
16. the opposite of the difference of 97 from 93; 4
17. the opposite of the opposite of the difference of 89 from 84; ~5

18. the opposite of the opposite of the difference of 101 from 100;
19. the opposite of the sum of positive 9 and negative 7; "2

ZO. the opposite of the sum of negative 9 and positive 7; *Z

21. the opposite of the sum of negative 3 and negative 6; ^^9

22. the sum of the opposite of the difference of 604 from the sum
of 983 and 729, and the difference of 604 from the sum of 983
and 729;
23. the sum of the difference of 4871 from 3572 and the difference
of 3572 from 4871;

It may take a while for you and your students to get used to the read-
ings given in the answers above. However, these readings have the
advantage of spelling out the meanings of the expressions. The
difference between, say, the conventional reading for Exercise 3:
minus the quantity positive 5 minus negative 7

and the one given in the answer on TC[l-82]a is the difference between
"reading the symbols" and reading aloud what the expression says
in written form. [A similar distinction, regarding the reading aloud
of single-quoted expressions, was called to your attention on TC[l-j]a
and, again, on TC[l-L]a. ] We have violated our own convictions here
by the way in which we stated the Solution of the Sample. Actually,
to read aloud what the expression [in the Sample] says in written form,
one should say 'the sum of the opposite of the difference of 9 from 12,
and the difference of 12 from 9'.

Although we believe that it is worthwhile developing in class the


answers given for Part B, it is not necessary to insist on the point
made here throughout the course. On this, see the last paragraph
on TC[l-88]c.

TC[l-82]c
[1-83]
[1-82]

which 1

1/96

The examples on page 1-83 illustrate the use of the principles dis-
cussed on TC[l-80]a, b, and serve as samples for Part C on page 1-84.
Answers for representative exercises in Part C [on page 1-84].
(8 + 5) + ("8 + "5) "5 cpa]
1. = [(8 + "8) + 5] + [apa,

[(8 + "8) + 5] + "5 = [0 + 5] + '5 [principle of opposites; "8 = -8]

[0 + 5] + "5 = + (5 + "5) [apa]


+ (5 .+ ~5) = + [principle of opposites; ~5 = -5]
+ = [paO]
So, since (8 + 5) + ("8 + ~5) = 0, it follows that "8 + "5 = -(8 + 5).
Hence, -(8 + 5) = "8 + "5.

3. (38 - 16) + ("38 + 16) = (38 + -16) + ('38 + 16) [principle for
subtraction]
(38 + -16) + ("38 + 16) = [(38 + "38) + -16] + 16 [apa, cpa]
[(38 + "38) + -16] + 16 = [0 + -16] + 16 [principle of opposites;
"38 = -38]

[0 + -16] + 16 = + (-16 + 16) [apa]


+ (-16 + 16) = + (16 + -16) [cpa]
+ (16 + -16) = + [principle of opposites]
+ = [paO]
So, since (38 - 16) + ("38 + 16) = 0, it follows that -(38 - 16) =
"38 + 16.

4. ("57 - "9) + (57 - 9) = ("57 + -"9) + (57 + -9) [principle for


subtraction]
("57 + -"9) + (57 + -9) = [("57 + 57) + -"9] + -9 [apa, cpa]
[("57 + 57) + -"9] + -9 = [0 + -"9] + -9 [principle of opposites;
57 = -"57]
[0 + -"9] + -9 = + (-"9 + -9) [apa]
+ (-"9 + -9) = + (9 + -9) [-"9 = 9]
+ (9 + -9) = 0+0 [principle of opposites]
+ = [paO]
So, since ("57 - "9) + (57 - 9) = 0, it follows that -("57 - "9) = 57-9.

TC[l-83, 84]
[1.10] [1-83]

v<- vt-
"!'•
o^
"l^ -t-

Consider the numbers

"8 - ""4
and "-8 - "4.

Is the second number, "^8 - ~4, the opposite of the first, "8 - *4 ?

One way to find the answer to this question is to add the second num-
ber to the first. If the sum is 0, the answer is 'yes'. If the sum is

not 0, the answer is 'no'. Let's try it.

("8 - *4) + (""8 - "4) = ("8 + ~4) + ("8 + ''4) [principle for sub-
traction; ~4 = —*4; *4 = —'4]

("8 + -4) + {^8 + M) = [("8 + ^8) + -4] + ^4 [apa, cpa]

[("8 + '8) + '4] + *4 = [0 + "4] + '4 [principle of opposites; ''8 - -'8]

[0 + "4] + M = + ("4 + ""4) [apa]

+ (-4 + ^4) = 0+0 [principle of opposites; *4 =


— ~4]

0+0 [principle for adding 0]

So, since

("8 - M) + (^8 - "4) = 0,

it follows that ''8 - ~4 is the opposite of "8 - *4. That is, that

^8 - -4 = -('8 - ^4).

[Is "8 - "^4


the opposite of ""8 - "4?]

Consider a second example. Is it the case that

-{98 -! 35) = 98 + "35?

To find the answer to this question, we can proceed as before and find

out whether ("98 + 35) + (98 + "35) = 0.

("98 + 35) + (98 + "35) = ("98 + 98) + (35 + "35) [Why?]

("98 + 98) + (35 + "35) = [Why?]

So, (98 + "35) = -("98 + 35).

^1, O- vl^
"f '4^ 'i*
[1-84] [1.10]

C. For each exercise, use the "adding method" and the principle
of opposites to check each sentence.

1. -(8 + 5) = "8 + '5 [ Hint : Show that {8 + 5) + ("8 + '5) = 0. ]

2. -(95 + '27) = "95 + 27

3. -(38 - 16) = "38 + 16

4. -("57 - "9) = 57 - 9

5. -(24 - 30) = 30-24


6. -("17 - 15) = 15 - "17

7. -(25 + 20) = -25 +-20

8. -(57 - 12) = -57 + 12

9. -(38 - 10) =10-38


10. -(53 X "28) = "53 X "28

11. -74 = 74 X "1

*12. -(38 + "57 - "76) = —38 - "57 + "76

^13. -[(725 - 631) - (497 - 985)] = (631 - 725) - (985 - 497)

D. Multiple -choice . [There may be more than one right answer. ]

1. 12 - 3 = ?

(a) 3 +"12 (b) "3 4 12 (c) -(12 - 3) (d) -(3 - 12)

2. 5 + 4 - 10 = ?

(a) 5+6 (b) 9-10 (c) 5 - (10 - 4) (d) 5 + (4 - 10)

3. 5 - 4 - 10 = ?

(a) 5 - (4 - 10) (b) 5 - (4 + 10)

(c) 5 - (10 - 4) (d) -(10 +4-5)

4. 3 X "2 is the opposite of ?

(a) "3X2 (b) -(3 X "2) (c) -("3X2) (d) -(3X2)


1/97

7. (25 + 20) + (-25 + -20) = [(25 + -25) + 20] + -20 [apa, cpa]
[(25 + -25) + 20] + -20 = [0 + 20] + -20 [principle of opposites]

[0 + 20] + -20 = + (20 + -20) [apa]


+ (20 + -20) = 0+0 [principle of opposites]
+ = [paO]
So, since (25 + 20) + (-25 + -20) = 0, it follows that -(25 + 20) =
-25 + -20.

10. 53 X "28 + "53 X "28 = (53 + "53) X "28 [dpma]


(53 + "53) X "28 = OX "28 [principle of opposites; "53 = -53]
X "28 = "28 X [cpm]
"28 X = [pmO]
So, since 53 X "28 + "53 X "28 = 0, it follows that -(53 X "28) =
"53 X "28.

11. 74 + 74 X "1 = 74 X *1 + 74 X "1 [pml]


74 X *1 + 74 X "1 = 74 X (*1 + "1) [idpma]
74 X (*1 + "1) = 74 X [principle of opposites; "1 = -*1]
74 X = [pmO]
So, since 74 + 74 X "1 = 0, it follows that -74 = 74 X "1.

^12. (38 + "57 - "76) + (-38 - "57 + "76)


= (38 + "57 + -"76) + (-38 + -"57 + "76) [prin. for subtraction]

(38 + "57 + -"76) + (-38 + -"57 + "76)


= (38 + -38) + ("57 + -"57) + ("76 + -"76) [apa, cpa]

(38 + -38) + ("57 + -~57) + ("76 + -"76)


= [0 + 0] + [principle of opposites]

[0 + 0] + = 0+0 [paO]

0+0 = [paO]

So, since (38 + "57 - "76) + (-38 - "57 + "76) = 0, it follows that
-(38 + "57 - "76) = -38 - "57 + "76.

TC[l-84, 85, 86]a


[1-84]

C. Fo^
[1-85]

1/98

^13. [(725 - 631) - (497 - 985)] + [(631 - 725) - (985 - 497)]


= [(725 + -631) + -(497 + -985)] + [(631 + -725) + -(985 + -497)]
[(725 + -631) + -(497 + -985)] + [(631 + -725) + -(985 + -497)]
= [(725 + -631) + (631 + -725)] + [-(497 + -985) + -(985 + -497)]
But, -(497 + -985) = 985 + -497, because
(497 + -985) + (985 + -497) = (497 + -497) + (985 + -985)
= 0+0 =

Hence,
[(725 + -631) + (631 + -725)] + [-(497 + -985) + -(985 + -497)]
= [(725 =1- -725) + (631 + -631)] + [(985 + -497) + -(985 + -497)]
= [0+0] +0 = + = 0.

So, since [(725 - 631) - (497 - 985)] + [(631 - 725) - (985 - 497)] = 0,
it follows that -[(725 - 631) - (497 - 985)] = (631 - 725) - (985 - 497).

Answers for Part D [on pages 1-84, 1-85, and 1-86].

1. b, d 2. b, c, d 3. b, d

4. b, c 5. a, b, c 6. a, d

7. b, d 8. a, b, c 9. a, b, c, d

10. a, b, c, d 11. a, b, c, d 12. a, c, d

13. a, d, e

TC[l-84, 85, 86]b


1/99

Here a quiz which tests the ability to combine the operation oppo-
is
sition with other operations on real numbers.

I. True or false?
1. --3 = --^3

2. -(3 - 2) = 2 - 3

3. -(*5 + -7) = -5 - ^
4. -("6 + -3 + *7) = -"6 + --3 + -*1

5. 16 - (7 - 82) = 16 - 7 + 82

6. -(857 X -359) = "857 X *359


7. -{*35 X "16 X n5) = -35 X n6 X "15

8. -(6 - 2) X (5 - 9) X (8 - 3) X (4 - 2)
= (2 - 6) X (9 - 5) X (3 - 8) X(2 - 4)

II. Fill the blanks to make true sentences.


1. *6 + - = *6 - *83

2. "5 X X *4 = *5 X "38

3. -(28 - 259) = "28 -

4. -(28 + 259) = "28 -

5. --(6 + -5) = "6 +

Answers for quiz.

I. 1. T 2. T 3. F 4. T
5. T 6. F 7. T 8. F

II. 1. ^83 2. *38 259 *259 5. *1

TC[l-84, 85, 86]c


[1-84]

C. Fqj
[1.10] [1-85]

5. 978 X 357 = ?

(a) "978 X "357 (b) -{978 X "357) (c) -("978X357)

6. —(596 - 984) is the opposite of ?

(a) "984 + 596 (b) -596 + 984 (c) "596 + "984 (d) 596 - 984

7. -(19 + 11) = ?

(a) 19-11 (b) -19 +-11 (c) 11+19 (d)-19+-ll

8. -("19 + "11) = ?

(a) -"19 +-"11 (b) "19 + ni (c) "19 - "11 (d) "11 - "19

9. -("3 + "5) = ?

(a) "3 + "5 (b) -"3 +-"5

(c) "3X-1+"5X-1 (d) ("3 + "5)X-l

10. -("3 + "5) = ?

(a) "3 + "5 (b) -"3 +-"5

"3 X "1 + "5 X "1 X "1


(c) (d) ("3 + "5)

11. -("3 - "5) = ?

(a) "3 + "5 (b) -"3 - -"5

"3 X "1 - "5 X "1 (d) ("3--5)X-l


(c)

12. A first number is the opposite of a second number

(a) if the sum of the numbers is 0.

(b) if the opposite of the first number is the opposite of


the second number.

(c) if the first nunnber is the product of the second number


by "1.

(d) if the second number is the product of the first number


by "1.

(continued on next page)


[1-86] [110]

13. The opposite of the sum of a first number and a second

number is

(a) the sum of the opposites of the nunnbers.

(b) the first number minus the second number.

(c) the second number minus the first number.

(d) the opposite of the first number, minus the second


number.

(e) the product of the sum by ~1.

NEW NAMES FOR NEGATIVE NUMBERS


Earlier in this unit we agreed that we could shorten such names
as '
*9 ' and '"304' by leaving out the raised plus signs. So, we could
write '9' and '304' when we intended the real numbers *9 and "304.
Now, to each positive real number there corresponds a real number
which is its opposite. And we get a name for this opposite by writing

an oppositing sign to the left of a numeral for the positive number.


Hence, —9 is the opposite of the real number 9. But the opposite of
the positivenumber 9 is a negative number. Hence, a name for this
negative number is '-9'. And this is the number which we have been
calling "9*.
For nnost of our purposes, we shall use such names as:

9, 35, 6^, 15.83

for positive nunnbers, and such names as:

-7, -58, -2^, -49.7

for negative numbers. In other cases where we want to stress the


notion of oppositing, or the fact that the nonnegative real numbers
are different from the numbers of arithinetic, we shall return to our
use of names like:

*9, "7, *35, "58, "6


J, "3^, "15.83, "49.7.
[1-87]

•ign

1/100
S.
The section on new names for negative numbers is included in order
to introduce students to the standard notation for negative numbers.
Unfortunately, the standard notation is dangerous in that it leads the
student to confuse the opposite of a number with a negative number.
We hope that, with the careful build-up we have given to these ideas,
the student will be aware of the distinction at all times.

Since -9 = "9, it is customary to pronounce '-9' as 'negative 9*. It


would be better to avoid this pronxinciation at present and stick to
'the opposite of 9'. This will pay off when, in Unit 2, students meet
'-x'.

The answer to the problem posed in the bracketed section at the top
of page 1-87 is: -7 = "7.

Ifthe raised minus sign were not available, it would not be possible
to express the fact that the opposite of 7 is negative 7, without using
the words 'negative' and 'opposite'.

If we are serious in trying to reach the goal of getting students to


understand what they are doing then we should regard the discussion
of the three uses of the minus sign as exceedingly important. [Of
course, a student who can distinguish between the three uses will not
necessarily get higher scores on a skill test than a student who doesn't
understand the distinction. That this is the case simply means that
the mathematical notation is ambiguous enough to permit people who
do not understand what they are doing to get the right answers. ] Since
we wish students to develop understanding as well as skill, we should
not treat this discussion in a superficial manner. A very effective
way for getting to the heart of the matter is to invent three signs which
differ much more from each other than the three now in use, and to
let students use these ad hoc signs for 10 to 15 minutes. [See
TC[l-88]a. ]

'C

TC[l-86, 87, 88]a


[1-86]

13.
[1-87]

ign

1/101

The three operations denoted by the minus sign are:

(1) the operation which, when applied to a nonzero number of arith-


metic, yields the "corresponding" negative number [see page
1-108].

(2) the operation which, when applied to a real number, yields the
opposite of that number, and

(3) the operation which, when applied to an ordered pair of real


numbers, yields the difference of the second from the first.

Operations (1) and (2) are singulary operations, and, up to now, (1)
has been denoted by ~ and (2) by - '. Since we know that, of the
' * '

two real numbers which correspond with a given nonzero nunnber of


arithmetic, the negative one is the opposite of the positive one --that
is, since, for example, "7 = -*1 , we can, for the most part, dispense
with a name for the first of the three operations. Operation (3) is a
binary operation, so no confusion should arise from using similar
operators — and -' for both the singulary operation (2) and the
' ' '

binary operation (3).

TC[l-86, 87, 88]b


[1-86]

13.
[1-10] [1-87]

[Here is a case in which you would want to use a raised minus sign
in naoning a negative number: Without using the words 'opposite'
and 'negative', write a sentence which states that the opposite of

seven is negative seven. ]

Using the oppositing sign name negative numbers, instead


to

of the raised minus sign, means that we are now going to use the
same sign in three ways,

(1) when naming a negative number,

(2) when naming the opposite of a real number,

and (3) when indicating a subtraction problena.

You will seldom make a mistake because of this ambiguity for you
will always be able to tell when meaning (3) i» intended, and since
the opposite of a positive number is a negative nuinber, confusing
(1) with (2) will make no difference. Here is an expression in which
the minus sign is used with different nneanings :

-7 8.

You can think of this as subtracting the opposite of the opposite of

8 from the opposite of 7


-*7 -8,

in which case the expression simplifies to '"15*. Or, you can think
of it as subtracting the opposite of "8 from "7

-7 ---8,

in which case, the expression again simplifies to '"15'. Can you


give two more ways of thinking of this expression?
Notice how the expression is to be read.

-7 - - -8

t t t t

t^ CO '-t^ 00
P o
C (1)
00

> -1-1
o > •-<

t^ do S ^ o|o
to &. O 00 g^
0) g^ UJ (i
O (U (- o
3

[1-88] [1.10]

MORE NAMES FOR POSITIVE NUMBERS


In many books [this one, too] you will see nunnerals like ' +7',

'+3r-', and '+(4 - 3)', in which the plus sign is used in the same
position as the oppositing sign. One interpretation of such a numeral
is that the writer wants to emphasize that he is talking about positive
numbers. [We have done this when we have written, say, '*^7' instead
of the shorter '7'. ] Another interpretation is that just as the minus
sign in ' — 7' nnay refer to the operation oppositing, the plus sign in
'+3' refers to the operation "sameing". For example,

+ *3 = *3, +-8 = "8, +-3 = -3, and +(5 - 9) = 5 - 9.

V/hen the plus sign is used in nanning a positive number as it is

in '47', you could read it as 'positive', or not pronounce it at all.

When it is used as a sameing sign, as it must be in '+(3 - 5)' [why


'must'?], read it as 'plus', or don't read it at all.

We have said that oppositing is the operation which takes you


frona a real number to its opposite. Can you describe the operation
sameing? Recall the principles for the real numbers. Do any of

these principles tell you about an operation which is really sameing?


Describe the inverse of sameing.

EXERCISES
A. You have seen that there are several ways of naming real num-
bers. It is innportant that you beconne familiar with all of these
ways, and that you develop skill in simplifying expressions
which contain the various kinds of numerals. The expressions
given below should be read and simplified.

Sample 1_. -8 - 6 + -
Solution . Read as 'the opposite of 8, minus 6, plus the
opposite of 3', or as 'negative 8, minus 6,

plus negative 3'.

Simplify, converting the subtraction to addition -of-


the -opposite :

-8 - 6 + -3 = -8 + -6 + -3
= -14 + -3 = -17.
ri-89]

1/102

On page 1 -88 we take the final step of introducing the student to


standard notation for positive and negative nunnbers. Just as there
are three meanings which are denoted by the minus sign, there are
three meanings denoted by the plus sign. The "strangest" of these
is that the plus sign denotes the operation sameing. In conventional
courses, we explain this by telling the student that he should pretend
that the plus sign is not there. We say in this course that the
plus sign in such cases stands for the operation of sameing. [The
technical name for such an operation is: the identity operation.]

An answer to the question 'why 'must'?' near the middle of page 1-88
is:

Because '*(3 - 5)' is nonsense since '3-5' must be a numeral


for a real number. There is no number whose name is 'positive
negative two'.
An answer to the question in the paragraph just before the exercises
on page 1 -88 is:
The operation adding 0, and the operation multiplying by '''1,

ajid the operation sameing, are all the same operation.

The following exercise was included in the 1958-59 edition and received
mixed reactions. It can be used very successfully in class to point
out the three uses of the minus sign and of the plus sign. If you use
it, we suggest that you introduce the six signs [or, of course, any
other six that you prefer] one at a time taking time in each case to
,

fix the meaning of the sign by several examples of its use.

You have seen that the nninus sign is used in three ways, and
that the plus sign is used in three ways. In order to see if we
understand these different uses, let's invent six signs for the
six uses and practice a bit in working with them.
•ir
'
for the minus sign of subtraction [7 -iV 5 is 2. ]

* '
for the minus sign of oppositing [*3 is the opposite of 3. ]

~' for the minus sign of direction [~3 is negative 3.]


® '
for the plus sign of addition [9 ® 5 is 14. ]

9
'
for the plus sign of sameing [<79 is 9. ]
'
Q' for the plus sign of direction [?? is positive 7. ]

TC[l-88]a
[1-88]

MORE,
1/103

1. Using these new signs rewrite the following expressions,


Saniple U *5 + "2 - *8 - -"3

Solution. 'JS © ~2 Ti- *8 -si- * ~3

(a) 8 - "5 + "6 (b) +2 - +7 + 9 - -37


(c) -5 9 (d) 9 + ++ 9

Simplify.
Sample Z_. ~5 © * ~7
Solution. Read as 'the sum of negative 5 and the
opposite of negative 7'. So,

~5 ® * ~7 = ~5 e '5'7 = 'Q'2.

Sample 3^,

Solution. Read as 'negative 5, the opposite of


negative 7'. This is not the name of
a number; it's just nonsense.

(a) *7 -3 (b) '3 iz ~5 (c) ^9 Ti- *~4


(d) ~5 9®*6 (e) <Q>10 ^ ~5 ® *~7
'1

Here are answers for the preceding exercises.

1. (a) 8 ilr ~5 ® ~6 (b) <^2 * ^7 ® 9 * *37


(c) *5 -tr **9 (d) 9 ® g 9***9

(a) Read as 'the sum of positive 7 and negative 3'. So,


'?7 ® ~3 = $4.

(b) Read as 'the difference of negative 5 from negative 3'. So,


~3 T^ ~5 = ~3 ® * ~5 = ~3 ® -JB = *2

TC[l-88]b
[1-88]

MORE,
1/104

(c) Read as 'the difference of the opposite of negative 4 fronn


[plus] 9'. So,

<^9 ^ *~4 = «'9 **~4 = <?.9 © ~4 = ,5.5.

[See lines 13, 14, page 1-88 in regard to reading the


"sameing" sign as used here. ]
(d) Read as 'negative 5, plus plus the opposite of 6' or as 'the
sum of negative 5 plus and the opposite of 6'. This is not
the name of a number; it's just nonsense.

(e) Read as 'the sum of the difference of negative 5 from


positive 10 and the opposite of negative 7*. So,
*10 Ti- * ~7 = 'Q'lO ® * ~5 ® * ~7
= <?10 ® <Q-5 ® •?>7

= 022.

On TC[l-82]c we asked that you have students practice reading


expressions aloud in a way that conveys their meaning. After such
practice, there is no harm in students' adopting the more conven-
tional reading of symbols suggested in the Samples for Part A on
pages 1-88 and 1-89.

TC[l-88]c
[1-88]

MORE,
[1.10] [1-89]

Sample 2_. +8 5 + —6 - +12

Solution . Read as 'positive 8, minus negative 5, plus


negative 6, minus positive 12'.

Simplify.

+8 - -5 + -6 - +12 = +8 + +5 + -6 + -12
+13 + -6 +-12
+7 +-12
-5.

Sample 3. 2-5+6-3 + 9-6-5


Solution . Read as '2, minus 5, plus 6, minus 3, plus 9,

minus 6, minus 5'.

Simplify.
2-5 9-6-5
+ 6-3 +
= 2+-5+6+-3 + 9+-6+-5
--3 + 6 + -3 + 9 + -6 +-5
3 + -3 + 9 +-6 +-5
= + 9 + -6 +—
= 9 + -6 +-5
3 +-5
-2,

Here is a second method of simplifying. [Note


that the associative and commutative principles for

addition are used in this second method. ]

We see that

2-5+6-3+9-6-5
= 2+-5+6+-3+9+-6+-5.
So, the given expression:
2-5+6-3+9-6-5
can be thought of as naming the sum of positive and

negative nunnbers. The positive numbers are 2, 6, and


9, and the negative numbers are —5, —3, —6, and -'S.

(continued on next page)


.

[1-90] [1.10]

Add the positive numbers, add the negative num-


bers, and then add the two sums.

2-5 + 6-3 + 9-6-5


= (2 + 6 + 9) + (-5 +-3 +-6 +-5)
17 + -19
-2.

[Note : In doing the following simplifications you may wish to

put in nnore steps than you feel you really need. It is

a good idea to do this at the beginning. Then, as you


do more problems, you will discover short cuts which
will enable you to do many problems without writing

more than one or two steps.]


1. 9-3-7 2. -5+6-11 3 -12 - 4 + 19

4. -6 + 5+9 5. +7-3-4 6 +8 + 12 + 17

7. +4 - +7 +-3 8. +5 - -3 + -4 9 +5 - -6 - -5

Simplify

1. +12 - +6 --7 +-8 +-13 - -7

2. -3 +-6 - -4 + -7 --7 + -13

3. 3 +-7 + 3 --7 - 5 + 12 +-3


4. -4 + +-8 --3 + 11 +-17 + 16

5. 7-3+-3-+8+17--1--6 + 4

6. -10 + 17 +-3 --10 - -7 - -3 -


-1

7. -10 - — 10 +—10 + -3 - —3 + + +2 - +2

8. -19 + -3 --4 +-6 +-17 +-4 - -3 - -19

9. - 7 +-3 + 7 --8 + 9 --10

10. 3-5 + 6-7 + 9 + 8-3


11. + 5-6 + 8-0 + 9-5
[1-91]

+ 7

Answers for Part A [which begins on page 1-88],

1. -1 2. -10 3. 3 4. 8

6. 37 7. -6 8. 4 9. 16

Answers for Part B [on pages. 1-90 and 1-91].

1. -1 2. -18 3. 10 4. 1 5. 21
6. 25 7. -10 8. -23 9. 24 10. 11

11. 11 12. 4 13. 4 14. 18 15. -18


16.

21.
4
-14.837
17. 13 18.
4 19.
12
20. -1.07

Answers for Part C [on page 1-91].

1, negative number 2. negative number 3. positive number


4. negative number 5. positive number 6. negative number
7. negative number 8. positive number 9. negative number
10. negative number

Part D is difficult. The first 18 exercises in Part L of the Supple-


mentary Exercises are of this type but much easier. You may want
to assign them first. Another helpful device is to suggest that the
students unabbreviate the expressions before trying to simplify.

Answers for Part D [on pages 1-91 and 1-92],

1. -9 2. 17 3. -20 4. 43 5, 46
6. 49 7. -5 8. 4 Q 25 10. 12
11. -19 12. -43 13. -67 14. -11 15.

16. 17. -10 18. 230 19. -191

TC[l-90, 91, 92]


[1-90]
[1.10] [1-91]

12. 2-7+8-6+4-f3 13. 1-1+2-2 + 3-6+7


14. 10-8+7-5+6+8 15. 5-15-20 + 18+2-8
16. 4+7-3-8+9-5 17. 6+4-3 + 12 + 10-16

18. 5^
2
-i-3 - Zj 22 - T^- 19. 6^ 34263
- 2 J - 57 + 3^ -
2|

20. 4.83 - 10 - 3.8 + 7.9 21. 16.75 - 11.3 + 40.72 - 61.007

[More exercises are in Part K, Supplementary Exercises .


]

C^. Examine each of the following expressions, and tell whether it

stands for a positive number, a negative number, or 0.

1. -^5 2. -7 3. +4 4. +"5

5. +(3 - 2) 6. +(7 - 9) 7. -(8 - 3)

8. -(4 - 7) 9. 5 10. + 5

D^. Simplify,

Sample 1. (3 - 8) X (7 - 1 1) - (5 - 12) X (6 - 10)

Solution . (3 - 8) X (7 - 11) - (5 - 12) X (6 - 10)

= (-5 X-4) - (-7 X-4)

20-28
-8.

Sample 2^. 6 - 3 X (4 - 7)

Solution . 6 - 3 X (4 - 7)

= 6 - (3 X-3)
= 6 - -9
= 15.

1. 5+7X(3-5) 2. -8 +-5 X (-2 +-3)

3. -9X(5+-2)+7 4. -3X(-7-8)+-2
5. 6 X (3 -—8) - 2 X (5 - -5)

6. (-2 - 3) X (-7 - 5) + (-3 + -8) X (-2 + 3)

(continued on next page)


[1-92] [1.11]

7. {5 - 8) X(9 - 12) - (6 - 13) X (7 - 9)

8. (4 - 3) X(2 - 7) - (12 - 15) X (8 - 5)

9. (6 - 1) X(7 - 2) - (12 + 1) X (3 - 3)

10. 7 - 2 X (5 - 8) - 3 X (4 - 2) - 5 X (6 - 7)

11. -2 - 6 - (3 + 5) - (7 + 2) - (8 - 3) - (-9 - 2)

12. +6 + -2 - 3 - 8 - 7 X (5 - 2 - 1) + 2 X (-3 - 6 - 2)

13. -2 X [5 + -(3 - 4)] - 5 + [(7 + 3) - 6 X (2 + 8)]

14. [(8 - 2) - (7 - 5)] X +4 - [(2 - 7) - 7 X (3 - 5)] X 3

15. [(9 - 5) + (7 - 24)] X 83 + [(24 - 7) + (5 - 9)] X 83

16. (18 - 27) X (5 3 - 41) + (27 - 18) X(5 3 - 41)

17. 17 - {5 - [2 - 3 X (4 -!-
1)] - 6 X (7 - 5) + 21}

18. -7 - 3 X {4 + 2 X (3 - 5) - 3 X [7 - 2 X (4 - 10) + 5] - 7}

19. 5 X (3 + -9) - 7 X [-6 - 3 X (2 - 3) + 4 X (3 -f 4) - 2]

[More exercises are in Part L, Supplementary Exercises .


]

1.11 Division of real numbers .


- -In grade school you learned that
to do the division problem:

24 ^ 6 = ?

you had to search for a number which when multiplied by 6 gave


24. This nunabe r is 4 because 4X6 = 24. Look at the following

sentence:
(4 X 6) V 6 = 4.

Does it illustrate the idea that dividing by 6 is the inverse of

multiplying by 6 ?

On the following page, at the left, is a list of pairs which belong


to the operation multiplying by "3. The list on the right is made by
reversing each pair in the list on the left.
[1-93]
1/106

In Unit 2we give a much more extensive treatment of division. Here


we wish only to bring out the fact that, just as subtracting ~3 is the
operation which is inverse to adding "3, dividing by "3 is the opera-
tion which is inverse to multiplying by "3.

The ordered pairs which are listed in the upper right quadrant of page
1-93 belong, of course, to the operation dividing by "3.

Answers for exercises in the middle of page 1-9 3.

-5 -3 *6 ^4 "7
(1) (2) (3) (4) (5) (6)

Let the student solve the division problems listed at the bottoin of
page 1-93 and at the top of page 1-94 by thinking in terms of inverse
operations. In short order, he will develop the rules for himself.

Answers for exercises at bottoin of page 1-93, and top of page 1-94.

1. "6 2. "6 3.
"6 4. "6 5. "7

6. *2 7. "3 8. *3 9. "9 10. 3

"3 12. "8 13. *8 "8


1. 14. 8 15.

6. "9 17. 18.

TC[l-93]
[1.11] [1-93]

V-- ^/v// ////// // ZZ>^- ^/ ////,/ , //////// A


; multiplying by "3 \

y / / / / y> , / / ^ ' / ; / /.^tV

(^9, -27) (^4, -12) X ("27, "9) (-12. "4r


(-3, ^9) (n, -3) ('9. -3) (-3, "D
/

(0, 0) (^5, -45) (-1, ^3) '


(0, 0) (-45, "15) ("3, -1)

(-7, "21) ("5.1. -15.3) ("21, -7) (-15.3, "5.1)

("6. -18) (-5. "15) (-18, "6) ("15. -5)

(-18, "54) (-12, "36) ("54, -18) ("36, -12)

(^21, -63) (-63, "21)

Can you give a name for the operation to which the pairs in the right

list belong? This operation is the inverse of multiplying by "3.

Use the lists to solve the following problenns in division.

(1) "15 4- -3 = ? (2) "9^-3=?


(3) -18 -^ -3 = ? (4) -12 3 = ?

(5) "21 ^ -3 = ? (6) '3 = ?

Now, suppose you wanted to solve the division problem:

-63 -^ "7 = ?

To solve this problem you could use a list for the operation dividing
by "7, or a list for the operation multiplying by "7. But, even with-

out such lists, you could still use your knowledge of multiplication
to solve this division problem. You could imagine yourself searching
through a long nnultiplying by "7 list for a pair in which the second

number was -63. The first number in the pair gives you the answer
to the division problem. What is the answer?

Do these division problems.

1. "18 V "3 = ? 2. "18 V -3 = ? 3. '18 T "3 = ?

4. -18 -f "3 = ? 5. "14 4- "2 = ? 6. "2 -^ -1 = ?

(continued on next page)


[1-94] [111]

7. "9 -^ *3 = ? 8. "12 -i-


"4 = ? 9. *27 T -3 = ?

10. 18 V 6 = ? 11. 18 T-6 = ? 12. 24 ^ -3 = ?

13. -24 T-3 ^ ? 14. 24 T 3 - ? 15. -24 T 3 = ?

16. 36 T-4 = ? 17. T 5 = ? 18. T -3 = ?

WAYS OF NAMING A QUOTIENT


The quotient of a first number (dividend) by a second number
(divisor) inay be named by putting a divide -by sign between

numerals for the numbers, the dividend numeral being placed on


the left. So, the quotient of 8 by —2 is nanned by:

8 T -2.

A simpler name for this quotient is :

-4.

In grade school you learned that a fraction can also be used to


name a quotient. Thus, the quotient of 8 by -2 is named by:

8/-2 or by: .
-J
The part of the fraction which nanaes the dividend is called the
numerator of the fraction, and the part which names the divisor
'

—8j
'

is called the denominator of the fraction. In the fraction ,

the numerator is '8' and the denonainator is '—2'.

EXERCISES
A. Simplify.

1. 12 -f 3 2. -17 ^1 3. -6 T-2

4. 8^-2 5. 10 T -1 6. -7 ^-1
7. -=--3 8. 9 T 3 9. 16 ^-4
10. 17 T-1 11. -27 T -3 12. 9 v-9

13.
^ 14.
^ 15. -^^
[1-95]

1/107

Notice that on page 1-94 we state that a fraction is a type of numeral.


The fraction has three parts, numerator, fraction bar, and denonr^i-
nator. If we want to refer to the number named by the numerator or
the number named by the denominator, we use expressions such as
'numerator-numiber* and 'denominator-nuinber*.

There are good reasons for using the word 'fraction' to denote a
numeral rather than a number. For exannple, we would want to say
' '

1 '
2 1
that y is a different fraction from ^ ; however, y is the same
number as -j • Thus, each nunriber has an infinite number of fraction

names. [The expressions -j and '2/4' are abbreviations for '(2 -r 4)'.
See page 2-87.

In the article in The Arithmetic Teacher previously referred to [See


TC[1-A, B, C].Ji Frank L. Wolf asks the reader to tell what is wrong
with the following reasoning. [This should give you another reason
why we wish to regard a fraction as a numeral rather than as a number].
Since 5 is a divisor of 10 we know that 5 is a

divisor of the numerator of -r-E • Hence, because


-p^ = rr- and because we may substitute equals for

equals, 5 is a divisor of the numerator of y.


Therefore, 5 is a divisor of 2.

Answers for Part A [on pages 1-94 and 1-95].

1. 4 2. -17 3. 3 4. -4 5. -10
6. 7 7. 8. 3 9. -4 10. -17
n. 9 12. -1 13. -2 14. 3 15. -11
16. 17. -2 18. 3 19. -6 20. 6

21. -6 22. 4 23. 4 24. -4 25. 3

26. -3 27. 3 28. 4 29. 4 30. -2


31. -7 32. 33. -5 34. \_
35. _i
36.
_J

TC[l-94, 95]a
[1-94]
[1-95]

-6

1/108

After the very brief introduction to the notion of the impossibility of


division by zero on page 1-65 in connection with the question: What
is the reciprocal of 0?, the student is now ready for a somewhat
more formal discussion of this problemi in Part B on page 1-95.
The student will discover when he tries to do the three "division
problems" in Exercise 3 that the list he nriade in Exercise 2 is a
"good-for-nothing" list. You should not engage in a formal dis -
cussion at this point all one needs to say here is that you cannot
;

do problems which involve division by zero. In Unit 2 we treat


this matter in nnuch greater detail.

Answers for Part B [on page 1-95].

1. (no, 0), (-8, 0). (li, 0), (|. 0), (.732. 0), (".OOl, 0), (6, 0),
(-35.5, 0), (j, 0), (19827, 0), (0, 0) . .. .

2. (0, nO), (0, -8), (0, Ij), (0, |), (0. .732), (0, ".001), (0, 6),

(0, -35. 5). (0, |), (0, 19827), (0,0)... .

The list of pairs a student makes in answer to Exercise 1 may be


captioned 'multiplying by 0', and his list for Exercise 2 may be
captioned 'dividing by 0'. But, if so, point out to him that the set
of ordered pairs which he is calling dividing by is not an operation
on the set of all real numbers. So, since all the similar phrases
'dividing by *1', 'dividing by "3. 5', etc. do name operations, the
caption 'dividing by 0' may be misleading. And this is a good reason
for deciding against using it.

3. Division by is impossible.

The answer to the bracketed question at the bottom of page 1-95 is that
ifyou find that, in comparing a first number of arithinetic with a
second number of arithmetic, you can get the second number by add-
ing to the first number, you are comparing just one number with
itself.

TC[l-94, 95]b
1/109

Here is a quiz which tests the student's ability to perform all five
operations with real numbers.

I. Simplify.

1. *80 -^ "20 2. "100 -=• "5 3. n? ^ "17 4. -f


-3

5. (*6 •= -2) + ("8 ^ -4) 6. ("12 ^ *Z4) + ("9 =- -18)

7. ("5 X -12) 4- (*2 X -3) 8. ("7 - -3) - (^0 f -5)

9.
^0 - S 10.
8 X "2
^ + "7 "1 X "4

11.
no - '30
12.
"8 X "5
no + "30 "9 + no
13. -(^6 4- -3) + -ri5 -f ^5) 14. -(-2 ^ -I) - -(-8 V -8)

15. % X ("5 + *3) X{-2 + -3) 16. -2 X (*8 - ~3) ^ (*8 + -6)

II. Fill the blanks to make true sentences.

1. *9 -^ = *3 2. -4 =• =n 3. *30 ^ = -6

4. *4 + (-60 ) = *8 5. *4 + (-60 ^ )
8
6. ns -
( V -2) = *20 7. -18 - ( ^ ^) 20

III. Multiple -choice . Draw a loop around the correct answer,

1. The quotient of a negative number by a negative number


is .

2. (A) a positive number (B) (C) a negative number

2. The opposite of the quotient of a negative nximber by a positive


number is .

(A) a positive number (B) (C) a negative number

3. The reciprocal of a negative number is .

(A) a positive number (B) (C) a negative number

TC[l-94, 95]c
[1-94]
Answers for quiz.

I. 1. "4 2. *20 3. "1 4.

5. "1 6. 7. -10 8.

9. "5 10. *4 11. -2 12. *40

13. "I 14. "1 15. ^60 16. "11

II. 1. *3 Z. *4 3. "5 4. -15

5. *5 6. *4 7. *4

III. 1. a positive number 2. a positive number


3. a negative number

TC[l-94, 95]d
[1.12] [1-95]

16. 0/-17 17. 34/- 17 18. -18/-6


-18
19.
if 20.
f 21.
3

22.
4
Y 23. ^
—4
24.
-4
1

-12
27.
4

28. +4/+1 29. 4/ + 1 30. 6/-

-K^
-15

34. n -^ ^2 35. n V "3 36. '1^1


[More exercises are in Part M, Supplementary Exercises .
]

B_. 1. Construct a list of ten pairs which belong to the operation


multiplying by 0.

2. Make another list of pairs by reversing the pairs in the


first list.

3. Use the lists to try to solve these problems.

(a) 6 V = ? (b) =^ = ? (c) V = ?

1. 12 Comparing numbers --Who has more stamps, Richard with


.

2350 or Al with 1820? This is an easy question. Richard does,


because 2350 is a larger number than 1820. And, how do you know
2350 is larger than 1820? You may have decided this by remember-
ing that in counting 1, 2, 3, etc. , 1820 comes before 2350.
Another way of telling that 2350 is larger than 1820 is to recog-
nize that you can get number [not 0] to 1820. And,
2 350 by adding a
as long as we are talking about numbers of arithmetic this is a ,

perfectly good method to use. To decide which of two numbers is


the greater, just decide which of the two numbers you would have
to add to in order to get the other. The one you have to add to is the
smaller, and the other is the larger. [If you find that you can get
one of the numbers of arithmetic by adding to the other, how many
numbers were you comparing in the first place?]
[1-96] [1-12]

EXERCISES
A. True or false ?

1. 9 is greater than 7 2. 3 is less than 17

3. 6 is greater than 4. 19 is less than 38 t 2

5. 11. 1 is greater than 11. 09 6. 7. 38 is greater than 7. 379

B. Each of the following exercises contains a pair of numbers of

arithmetic. Compare the numbers in the pair, and write your


result as a sentence.

Sample. (15. 17)

Solution . I can add a number of arithmetic (not 0) to 15

to get 17. So, I'll write:

15 is less than 17.

[l could have written '17 is greater than 15'.]

1. (9,4) 2. (16,61) 3. (10002, 100002)

4. (3^,3^) 5. (9-^,94) 6. (0.10002, 0.100002)


5 4 o I

[Note: Let's agree to abbreviate 'is greater than' by '>' and


'is less than' by '<'-]

7. (21, 17) 8. (93,39) 9. (74, 73.5)

10. li- 12. (.304. .208)


(Yf'T^) (^'M^
C^. Fill in the blank with a numeral for a number of arithmetic to
make a true sentence, and then read the sentence aloud.

1. 19 < 2. 34 > 3. _>55.1

4. 6^ < 5. y^ > 6. < .0003


[1-97]

1/111

AnSTwers for Part A.


1. T 2. T 3. T 4. F 5. T

'1^

Ans>wers for Part B.

1. 9 is greater than 4 [or: 4 is less than 9]

Z. 16 is less than 61 [or: 61 is greater than 16]

3. 10, 002 is less than 100, 002 [or: 100, 002 is greater than 10, 002]

4. 3t
5
is greater
B than 41-4
3-r [or: 3 -r is less than 3t]J
5

5. 9 Q- is less than 9 •= [or; 9 y is greater than 9-q-]

6. 0, 10002 is greater than 0. 100002 [or : 0. 100002 is less than


0. 10002]

7. 21 > 17 8. 93 > 39 9. 74 > 73.5

^°'
IT ^ IT ^^-14^^ ^^- -304 >. 208

Answers for Part C.

1. A numeral for any number greater than 19 will inake the sen-
tence true [e.g., 19.5].

2. A numeral for any number smaller than 34 [e . g. , 3 3]

3. A numeral for any number greater than 55.1 [e.g., 55.2].

4. A numeral for any number greater than 60- [e.g., 6^].


9 8
5. A numeral for any nunnber smaller than y:,- [e.g., yr]-

6. A numeral for any number smaller than . 000 3 [e.g., . 0002].

TC[l-96]
[1-96]
[1-97]
[1-96]

1/112

Here is another case in ^which we use a familiar phrase but with a


new meaning. The students know the meaning of > with respect ' *

to numbers of arithmetic. We now want to use this sanne symbol


to denote a relation among real numbers. Clearly, this relation
for real numbers ic different from the one for numbers of arith-
metic, just as multiplication as an operation for real numbers is
different from multiplication as an operation for numbers of
arithmetic. So, the only way in which we come to use the symbol
> with correctness in connection with real numbers is to have
' '

a definition which tells its nrieaning. V/e give this definition on


pages 1-97 and 1-98, trying to show some of the desirable character-
istics of the definition. But, it is an arbitrary matter just as the
definitions of addition and multiplication for real numbers are
arbitrary matters.

Answers for Part A [on page 1-98].

1. *8 < *11 2. ""S > "1 3. "10 < '7

4. "5 > '30 5. *15 > *3 6. -7 < -4


7. -98 < 2 G. 5 > 2 9. 5 > 1

10. 5 > 11. 5 > -I 12. 5 > -2

13. 5 > -5 14. 5 > -7 15. -7 =-7


16. -7 < -6 17. -7 > -8 18. -7 > -9
19. -20 < 19 20. -19 < 20

"I-

Answers for Part B [on page 1-98].

1. A numeral for any number greater than -19 will make the
sentence true [e.g., —18.9].
2. A nuiTieral for any nuniber greater than 19 [e.g. , 20].

3. A numeral for any number less than 19 [e.g., 18.9].


4. A numeral for any nunnber less than -23 [e.g., -24].
5. A nunneral for any number less than [e. g. , -.5].
6. A numeral for any number greater than [e.g., . 25].

7. A numeral for any number less than -30 [e.g., -60],


8. A numeral for any number greater than -30 [e.g., -15],
9. A numeral for any number less than -= [e.g., ~Z"]«

TC[l-97, 98]
[1.12] [1-97]

COMPARING REAL NUMBERS


How shall we proceed in comparing real numbers? Is *5 < ^17?
Is '•4 > "2? Is ~3 < "2? Is "100 > ""1 ? There are some clues around
which can help us understand why mathematicians do decide as they
do about which of two real numbers is the larger.
Consider the sentence:
4 < 19.

Is this a sentence about numbers of arithmetic, or is it a sentence


about real numbers? Actually, we can't tell, because the numerals
'4' and ' 19' are ambiguous. Up to now, this ambiguity did not bother
us because no matter what interpretation we used, sentences such as
'5 + 7 = 12' and '6X4 = 24' made sense. So, in order to keep this
freedom of interpretation, let's agree that if we see the sentence:

4 < 19,

it could be telling us either that the number 4 of arithmetic is less


than the number 19 of arithmetic, or that "^4
is less than ^19. So,
which is smaller, ""7 or '20? ""15 or ''2? *1000 or 0?

Let's review how we answer questions like these for numbers of


arithmetic. All we do is find out which number you have to add to to

get the other. The one you add to is the smaller.


But, does this simple test work for real numbers? Take the
pair of real numbers *Z and *\b. Is there a number you can add to
"^2
to get *15? Is there a number you can add to ''lb to get *2 ? The
answer to both questions is 'yes'. So, the sinnple adding test is no
help. How could we change the test so that it would help? The
accepted answer is: find out which number you would have to add
a positive number to to get the other; the one you have to add a

positive number to is the smaller. Use this test to check the follow-
ing sentences:

'
^s < n7, "101 >noo, < %.9.

Now we have a test for deciding which is the larger of two non-
negative numbers. And this test is such that when we ask the
question '
Is 4 < 19?', we get the same answer whether we are
thinking of numbers of arithmetic or of real numbers.
[1-98] [1.12]

What about dealing with the rest of the real numbers? Once
again, the accepted answer same test is
is that the to be used for
all pairs of real numbers. The number of the pair to which you
can add a positive number to get the other is the smaller. For
example, "3 < *1 because if you add *10 to "3, you get *7. The
sentence '"12 < '1 '
is true because we can add *1 1 to "12 to get "1.

Use this test of adding a positive number to check each of the fol-

lowing true sentences.

"5 < n2 "11 > "13 "100 < n "73 <

EXERCISES
A. Use one of the signs '
> ', ' < ', and ' = ', and write a true sent-
ence which compares the given real numbers of each pair.

Sample . ("15, "19)

Solution . To which of these numbers can I add a positive


nunnber to get the other? Since

"19 + *4 = "15,

"19 is the smaller. So, I write:

"15 > "19.

[Could you write something else?]

1. ('8, ni) 2. (^3, "D 3. ("10, "7) 4. ("5, "30)

5. (^5, ^3) 6. (-7, -4) 7. (-98, 2) 8. (5, 2)

9. (5, 1) 10. (5, 0) 11. (5,-1) 12. (5,-2)

13. (5, -5) 14. (5, -7) 15. (-7,-7) 16. (-7,-6)

17. (-7, -8) 18. (-7, -9) 19. (-20, 19) 20. (-19, 20)

B^, Fill in the blanks to make true sentences.

1. —19 < 2. 19 < 3. 19 >

4. -23 > 5. > 6. >

7. < -30 8. _> -30 9. -<-\


1-99]

'v
1/114

On the other hand, one could think of blobs of chalk


organized into infinitely long streaks, as fornaing an approxiniate
model for geometry. So, it would be appropriate to refer (in the
less precise sense) to such streaks and blobs as lines and points,
['would be' because infinitely long streaks of chalk don't exist.]
And, it is customary to speak of any moderately straight and
reasonably long streak as a line, and to speak of a small bit of it
as a point. [Incidentally, this use of 'point' and 'line' in the
teaching of geometry leads to serious misconceptions for students.]
We shall, as stated on page 1-99, use such streaks as pictures of
parts of the number line, and use bits of them as pictures of points.
To avoid confusion with 'point of the number line' [which we shall
ordinarily abbreviate to 'point'], we shall use 'dot' when referring
to a "point" of a "streak line". [On occasion, we shall use 'point'
and 'line' to refer to the physical objects when the content makes
clear what is going on.]

Notice that many conventional texts use 'nuniber line' for what we
call a 'picture of part of the nunnber line'. In our terminology it
doesn't make sense to say:
Draw a nunnber line and mark the nunnbers and 1 on it.

Rather, we would say:


Draw a picture of the number line and naark dots on it
corresponding with the numbers and 1.

Answers for questions in the text.

(1) •

li 2

(2) Yes (3) because < ly < 2

(4) 2 is "to the left of" 5 (5) 8 is "to the right of" 1

(6) 2 is **to the right of" -4 (7) -2 is "to the left of" -1

(8) because -I < - 1 <


•J,

Note that the answer to (1), above, is a picture, not of a line, but of

TC[l-99]b
^1-99]
1/113

Notice that the appropriateness of the picture on page 1-99 of the


number line is a consequence of the way in v^hich the relation > is
defined for real numbers. While doing the exercises on page 1-9 8,
students \will probably construct similar "mental pictures" for them-
selves.

Although you will probably find that your students have no difficulty
with the terminology introduced on pages 1-99 and 1-103, you may
yourself experience some confusion due to conflicts with your pre-
vious use of the terms 'number line' and 'point'. The words 'line'
[or, more explicitly, 'straight line'] and 'point' are used in many
ways. In abstract deductive geometry these words are primitive
ternas and, properly, neither is assigned a referent. When one does
assign appropriate referents to these words [and to the other primi-
tive terms--for example, 'congruent'] one obtains a nnodel of abstract
deductive geometry. Because of this it is customary to use 'line' and
'point' to refer to the corresponding entities in any model of geome-
try; and also, to use these words in a less precise sense, to refer to
similar entities which occur in what are, in some sense, "approxi-
mate" models of geometry. And, stennming from such uses, there
are nrxetaphorical usages like these in 'Line up! and 'What point are
'

you driving at?'.

Now, from the point of view of elennentary geometry, an important


characteristic of a "line" is that it consist of "points" which are
"ordered" in buch a way that it is possible to set up a one-to-one
correspondence between them and the real numbers so that a point
is "between" two others just if the real number which corresponds
with it is greater than the number which corresponds with one of the
two points and less than that which corresponds with the other. Since
the set of real numbers, when ordered by > [or by <], certainly,
itself, satisfies this requirement, it is appropriate to refer to its
members, the real numbers, as points. For definiteness, we call
the ordered set of real numbers the number line and speak of each
,

real number as a point of the number line. The main advantage in


using this geometric language is that doing so helps one to apply
one's geometric intuition to the organization of relationships among
real numbers. [In fact, mathematicians are likely to refer to the
members of any set as points whenever this cue to geometric intui-
tion seems likely to be helpful.]

TC[l-99]a
[1-13] [1-99]

1.13 The number line --Many people think of the nunribers of


.

arithmetic as being "lined up m order", starting with the smal-


lest nunaber, 0, and with larger nunibers to the "right" of

smaller nunibers.

(1) Draw a picture in the space above to show this arrange-


ment. Ivlake dots on the picture for the numbers 0,

1 — , and 2.

(2) Is the dot for I y between the dots for and 2?

(3) V.'hy should the dot for 1 — be between the dots for and 2?

(4) Make a dot for the number 5. How does your picture show
that 2 < 5 ?

(5) Make a dot for the number 8. How does your picture show
that 8 > 1 ?

One of the advantages of our agreement for comparing real


numbers is that we can also think of all the real numbers as lined
up in order.

^ « ( • » > «. «^ ^^ • —
-5 -4 -3 -2 "1 n '2 ""3 •'4

(6) How does this picture show that 2 > —4 ?


(7) How does this picture show that— 2 < —1 ?

3
(8) Make a dot on the picture for -j . Why should this dot be
placed between the dots for and — 1 ?

In comparing real numbers it is helpful to think of them as being

lined up. So, we often call the ordered set of real numbers the

line of real numbers , or just the number line , for short. When
you draw a picture like the one just above question (6), you have
a picture (of part ) of the number line .
>

[1-100] [1.13]

EXERCISES
A, Here is a picture of the number line.

< 1

"5
AD
4
"4 "3
1 •
(

"2 -1
H
B •

n
1
'

^2
C (

*3
1

*4
E• '

''5 %
1—

Give the real number which corresponds with each labeled dot.

A: B: C: D: E:

B. Here is another picture of the number line. Mark dots on the


picture which correspond with the listed real nunnbers.

' 1 1
'
H \
1 — j '
!
i —
1
^>

"4 "3 "2 -1 n *Z *Z *A *5 *6

A: 5 B: 2 C: -3 D: \-
2
E: -2^3 F: -3.2 H: -\-
4

C_. A mental picture of the number line makes it easy for you to

compare real numbers. Write a comparison sentence about


each pair.

1. (-3, +17) 2. (+5.5, +3)

3. (-6. -5) 4. (-152, -2, 176)

5. (+.0012, -.0138) 6. (+.001, +.0001)

7. (-6.382, +\) 8. (^, ^)

9. (-1,428, +.0052) 10. (-.00016, -43,213)

[More exercises are in Part N, Supplementary Exercises .


]

D. True or false? [Just as '


/ '
is used for 'is not equal to', so
are ' </.
'
used for 'is not less than' and '/ '
for 'is not greater
than'. ]

1. 5 / 10 2. 4 / 3. -10 ^ -20
4. -10 / -10 5. . 5 / . 75 6. 2 / 2
ri-101]

1/115

Answers for Part A.

A: -4 B: -4 E: 4-

Ansxwers for Part B.


FC E H D A
> • -•— I- -«
*5'
n *3 *4

Answers for Part C.


1. -3 < +17 [or: +17 > -3] 2. +5.5 > +3 [or: +3 < +5. 5]

3. -6 < -5 [or: -5 > -6] 4. -152 > -2,176 [or: -2, 176 < -152]

5. +.0012 > -.0138 [or: 6. +.001 > +.0001 [or:


-.0138 < +.0012] +.0001 < +.001]
11 21 21 11
7. -6. 382 < +j [or: 8. __>_^
. r
[or: -^< .

-y]
-,

+ j > -6. 382]

9. -1428 < +.0052 [or: 10. -.00016 > -43213 [or:


+ .0052 > -1428] -43213 < -.00016]

Answers for Part D [on pages 1-100 and 1-101].

1. T 2. T 3. T 4. F 5. T
6. T 7. T 8. T 9. F 10. T
11. F 12. T 13. No

Answers for Part E [on page 1-101].

1. T 2. F 3. T 4. T 5. F
6. T 7. F 8. F 9. T 10. T

TC[1-100, 101]
[1-100]
[1.13] [1-101]

7. 5 / 10 ^r 2 8. -3/0 9. -13/5/3
10. 5 = 5 11. 5 < 5 IZ. 5/5
'1^

13. Do '/' and '< '


tell you the same thing? Compare
Exercises 11 and 12.

Suppose each of two students, Alice and Rachel, picks a real


number and each whispers her choice to Ned. Ned tells you that
Alice's number is not less than Rachel's. Can you conclude that

Alice's is greater than Rachel's?


When you write the comparison sign '/' between numerals and
have a true sentence, the numeral on the leftname a number
can
which is greater than or equal to the number named by the numeral
on the right. So, sometimes, instead of using '/' we use '>', a
combination of '
> '
and '
= '. Similarly, we son^etimes use '<'
instead of '
/ '. [Read '
> '
as 'is greater than ot_ equal to' and '
< '

as 'is less than or equal to'.]


So, for example, the sentence:

(1) 5/6
says the same thing as does the sentence:
(2) 5 < 6 or 5 = 6,

and (1) and (2) say the same thing as does :

(3) 5 < 6.

E. True or false ?

1. 5 > -6 2. 10 < -26 3. 4 < 4

2-2 5. _3<-- 6.
5 ^ 6

7. 1.53 < 1.053 8. -1. 542 > +0.001

9. -^ 1~|| 10. 972 - 846 > -(972 - 846)


)

[1-102] [1.13]

F. For each of the following exercises, make as many true sent-


ences as you can by inserting the signs '
= ', '
/ ', '
< ', '
'/ ',

'
> ', '
/ ', '
< ', and '
> '.

Sample . 6 4

Solution. 6 > 4, 6/4, 6 > 4, 6/ 4

1. 5 3 2. -4 -4 3. 6 -3
1
-10 -9
4. 5.
-f ^
6.
782
3 -3 8 - 2 2 -
8
7. 8. 9.
7 -7 3 - 7 3 -
7

10. Suppose we abbreviated:

is less than or is equal to or is greater than


by:
<
>

In which of the Exercises 1-9 above could you use this sign
to make a true sentence? Do you think anybody uses a
sign like this ?

[More exercises are in Part Q, Supplementary Exercises .


]

vl^ ^"^ v'^


'1^ '1^ 'p

None of the pictures of the number line which we have drawn


looked like this.

^ — 1 , ,
1 1
1 1
^
"3 -2 "1 *1 *Z *3 *4

This picture would be perfectly all right if we were interested


''^3
only in using it to check a sentence like > "1". But, we
can draw our pictures so that they also enable us to check a
sentence like '
4 - 2 = "1 - "3 ' just by looking at the picture.
The picture above doesn't help in this case. V/hy not?
[1-103]

1/116

Answers for Part F.

1. 5 / 3, 5 5<! 3, 5 > 3, 5 > 3

2. -4 = -4, -4 ;^ -4, -4 / -4, -4 > -4. -4 < -4

3. 6/-3, 6 > -3, 6 > -3, 6 / -3

4. -10 /-9. -10 < -9, -10 < -9, -10/ -9


5. 4/2, -^<2, -iO,,^ .10^,
6- -TftT
782 f- 0' -7Q
782
< 0, -
tIi
<o, -yl^/o
7. = 0, ;^ 0, / 0, < 0, >

3 -3 -3 3 -3 3 -3 -3
8.
7 -7 » 1^--7 '
7
;^
-7 >
7
<
-7 l> -7

8 2 - 8 8 - 2 2 - 8 8 - 2 8 8
3 ^^ 3 - 7
'
3 - 7
<
3 - 7
4
3 - 7 iff 7 '
3 -
- 2
7
^^
2 - 8
3 - 7

10. The symbol rr could be used to make a true sentence in each


of the Exercises 1-9.

Answer to question on last line of text:


The distance, on paper, between the dots for 2 and 4 is not the
same as the distance between the dots for "3 and ~4.

You can make a true -false quiz on Part F by taking the sentences
given above and making slight modifications in some of them. [For
example, use '5 / 3' as it is, but change *5 / 3' to 3 ^ 5* or to '

•5;^ 3'.]

TC[1-102]
[1-102]

F. For
[1-103]

X
[1-102]

1/117

We have found it convenient to use geometric terminology in talking


about the system consisting of the set of real numbers together with
the relation >. Thus, we call such a system the number line and ,

we each element in this system a point of the number line


call We .

extend the use of geometric terminology by introducing the phrases


*the distance between real numbers* and 'the distance between points
of the number line*. [See TC[l-99]a. ]

Answers for Part A [on pages 1-103 and 1-104].

1. 3 2. 3 3. 10 4. 5

7. 45 8. 21 10. 1313

12. 524

In either case, after subtracting the two real numbers the girls
shoxild think of the number of arithmetic to which the real num-
ber corresponds. This number of arithmetic is the distcuice
between Ruth's number and Rachel's number.

TC[l-103]
[1. 13] [1-103]

The picture didn't help because the distance betv^een the


dots for Z and 4 is not the same as the distance between the
dots for ~3 and "1. Pictures of the number line such that
sentences like '
4 - 2 = "1 -
"3 ' can be checked by comparing
distances betv/een dots are said to be drawn with a uniform
scale . V/hen you did the exercises in Parts A and B on
page 1 - 100, you probably took it for granted that the pictures

were drawn with a uniform scale.

THE ABSOLUTE VALUE OPERATION


You have seen that it is helpful to think of the set of real num-
bers as a line, each real number being a point of the line, VtTien

thinking about ordinary lines, we have a notion about what we mean


by 'distance between points'. Can we develop such a notion for the

number line? That is, can v/e make sense out of '
the distance

between real nunnbers '


?

Think of a picture of the number line which has been drawn with
a uniform scale. The length of the segment betv/een the dots for ~4

and ~1 is 3 units, no matter what unit was used in drav;ing the pic^are.
V.laatever unit was used in drawing the picture, the distance (v.'ith

respect to this unit) between the dots is 3. So, let us agree that the
distance between the real numbers ~4 and "1 is 3. [As you saw in

our earlier discussion of trips, distances are numbers of arithmetic.


V/hat is the distance between "1 and ~4?]

EXERCISES
A. For each of the listed pairs of real numbers, give the number
of arithmetic which is the distance between the real numbers.

1. (^7, nO) 2. (no, "7) 3. (-5, *5)

4. ('7, -12) 5. (n2, *7) ' 6. ("8, "24)

(continued on next page)


[1-104] [1-13]

7. ("10, *35) 8. (-72, -72) 9. (-38, "59)

10. ("724, ^589) 11. ("57, -804) 12. ("72. "596)

13. Ruth and Rachel each pick a real number. To find the

distance between these real numbers, Paul suggests that


Ruth subtract her number from Rachel's. Ned protests,
and says that Rachel should subtract her number from
Ruth's. Arthur, who understands that a distance is a

number of arithmetic, says that both are right as far as


they go, but each needs to take one more step.

What is that step?


0-- vl, x',
'1^ -t" 'I''

You have seen in the exercises in Part A that you can find the

distance between real nunnbers, say, ~5 and -9, by first subtracting


either from the other.
-5 - -9 = "4 or -9 - -5 = -4

Then, the distance between the real numbers is the number of arith-
metic which corresponds with both of these differences.

4 is the number of arithmetic which corresponds with "4

and

4 is the number of arithnnetic which corresponds with "4.

So,

the distance between -5 and -9 is 4.

V/e call the number of arithnnetic which corresponds with a real


number the absolute value of the real number . For exannple,

the absolute value of "7 is 7,

the absolute value of "3 is 3,

the absolute value of is 0,

the absolute value of -4 is 4,

and the absolute value of (9 - 15) is 6.

So, the distance between two real numbers is the absolute value of
the difference of either one from the other.
[1-105]

veen

1/118

From here, through page 1-110, it is essential to keep clear the


distinction between numbers of arithmetic and nonnegative real
numbers. So, we have been very careful to avoid creating contexts
in which the meaning of a numeral nnight be ambiguous.

'1-

Our definition of 'absolute value of a real nunnber differs from


'

definitions given, for example, in college algebra textbooks.


[Definitions of the latter kind make the absolute value of a real
number a nonnegative real number, rather than a number of
arithmetic. ] We have experimented with both definitions and
believe the one given on page 1-104 to be the more satisfactory
to use at this level. On page 1-110 we point out that, just as
names for numbers of arithmetic such as '5' and *9' can be used
annbiguously to nanae nonnegative real nunnbers [*5 eind *9, re-
spectively], so can names of numbers of arithnnetic like ' *5
|
|

and ' "9


I
be used cimbiguously to denote nonnegative real numbers
I
'

[*|*5| and *I-9l, respectively].

TC[1-104]
[1-104]
[1-105]
1/119

Answers to questions on page 1-105.

Top of page: Z7, 27, Z7; 573, 573, 573; 700, 20, 73, 48. 100, 0.

List of pairs for absolute valuing: (~3, 3), (*3, 3), ("142, 142),

(-1, i), (-5|, 5|), (-192, 192), (-1.92. 1.92), ("17, 17), (^, i)
Bottom of page : (1) "2, '"2
(2) '3,^3 (3) "527, *527 No
(4)

(5) Yes, the absolute value of the real number is the number
of arithmetic.

'I*

Answers for Part B [on page 1-106].

1. 9 2. 17 3. 12 4. 20 5. 12
6. 142 7. 18 8. 9. 3 10. 15
11. 34 12. 15 13. 9 4. 7 15. 7

16. 12 17. 18. 219 19. 20.

Answers for Part C [on page 1-106].


1. *3, or: -3 2. *5, or: ~5 3. *5, or: "5
4. *4. or: "4 5. *5, or: "15 6. *12, or: *Z
7. *14, or: "6 8. *3

Answers for Part D [on pages I-IO6 and 1-107].


1. > 2. > 3. > 4. > 5. < 6. <
7. > 8. > 9. = 10. = 11. > 12. >
13. = 14. = 15. = 16. > 17. <

TC[1-105, 106]
[1.13] [1-105]

What is the absolute value of "27? V/hat is the distance between


"27 and 0? V/hat is the distance between and "27?
V/hat is the distance between *400 and ""973? V/hat is the absolute
value of (^400 - ^973)? Of (*97 3 - -^400)?

What is the absolute value of (^634 - ~66)? Of ("5 X ^4)? Of ^73?


Of "48? Of "100? Of 0?
Notice that absolute valuing is an operation. It takes you from
each real number to a single corresponding number of arithmetic.
We say that it is an operation on^ the set of real numbers to^ the set

of numbers of arithmetic. Up to now, we have used a list of some


of the pairs in an operation in order to "visualize" it. Make such
a list [about ten pairs] for absolute valuing.

Here is another way to visualize absolute valuing.


/N\ \
\ \
\
/ /
f3
/

/
/
/
/
-n
/ \ \ \
\ ^ \
/ / / +2
\ \ \
/ / / / / \
/ ^ / / /
\ \
^ X - \ ^
\
/ / / / / , . .
\ \ \
/ / / / / *0 s
/ / / / / T
-\- -+- -ij -I
-t- -f- -t-

"4 "3 1 n *2 ^3 M *5 "6 "7

(1) What two real numbers each have absolute value 2?


(2) What two real numbers each have absolute value 3?
(3) What two real numbers each have absolute value 527?
(4) Does any real number have absolute value —3?
(5) Does any real number have absolute value 0?
[1-106] [1.13]

As in the case of other operations, it is convenient to have a


sign for absolute valuing. The standard sign consists of two vertical
bars. Thus,
'
I
5 I
'
nneans the absolute value of "5,

and
'
1*17 I' means the absolute value of *17.

[V/e pronounce '


|
. . . |
'
as 'the absolute value of . . .
'.
]

^1, »i^ vi^


'I- '1^ "i-

B. Simplify.

Sample .
"3 + |
*?
| |
|

Solution .
"3 + "7 = 3 + 7 = 10
I I I I

1. *7 + -Z 2. "5 + n2 3. -8 + "4

4. "9 + "11 5. "2 + no 6. "71 + "71

7. ^21 - -3 8. "15 - jns 9. -103 - noo


10. "5 X "3 11. "2 X "17 12. '5 X "3

13. % + ^3 14. "9 - '2 15. "2 - *9

16. "5 + '7 17. "5 + ^5 18. 1-102 - ni7


19. '4 - '3 + "5 - "7 - ^4 - "2

20. "2 + "3 X M - "5 - ^6-^2 X *5 - "5

C. Fill in the blanks to m ake true sentences.

1. 7 + =10 2. _ + 4 = 9

3. 8 - - 3 4. 9 X = 36

5. + ^5 = 10 6. _ + "7 = 5

7. - M =10 8. - ^3 =

D. Complete each to a true sentence by inserting one of the signs


'
= '. ' < ', or ' > '.

1. rsi 1^31 2. |-5| 1^31 3. |"7| 1^21

4. I
-"7 I I
-"2 I
5. |0| |-7| 6. |0i 7

"2
7. r±
'
10
2
5
8.
3
9. I
ill
8
If3
[1-107]
[1-106]

si ar

1/120

Up to now we have not quite told students how we use the word
'operation'. On page 1-107 we nearly do so. In the nnore sophis-
ticated language of Unit 4, an operation is a set of ordered pairs
no two of which have the sanne first component. At present we
take the word 'ordered* as understood, and avoid using the word
'component'.

Prior to this time it has probably not been quite clear to students
in what cases we say that an operation has an inverse [although it
should be clear that, if it does, then the inverse is obtained by
reversing each ordered pair which belongs to the given operation].
Having nnade precise the notion of operation we can now character-
ize, as we do on page 1-108, those operations which have inverses,

Finally, we can categorize operations in a useful way by paying


more explicit attention to the sets on which they are "defined".

TC[1-107, 108]
[1.13] [1-107]

10. ^7 - ^5
I
+ I
^5 - "2
I
\n - '2

11. M - %| + 1^6 - ^3| 1^4-^3

12. ^4 - -2
I
+ I
-2 - "8
I i
M - "8

13. "2 - n I
+ I
n - ^3
I I
"2 - ^3

14. "2 X "3 ^2 X -3 15. |"3| X |-4| "3 X "4


I I I I I I

16. ^7 --3| 1^71 - |-3| 17. 1^9 + "21 |"9| + i-2

DOES ABSOLUTE VALUING HAVE AN INVERSE?

,-, I am thinking of a number. If I add *Z to it,

I get ^12. What number am I thinking of?

I am thinking of a number. The absolute value of


(11)
this number is 7. What number am I thinking of?

Do you see a difference between these problems ? You can tell what
number was thought of in the first problenri because the operation
adding *Z has an inverse. In the second problem, although you know
that the number must be either *1 or "7, you can't tell which. The
operation absolute valuing does not have an inverse. Let's look into
what this means.
To begin with, we ought to be clear on what an operation is. The
sets of pairs which we have been calling 'operations' have this impor-
tant property --none of them contains two pairs that have the same first

number [but different second numbers]. That is, an operation operates


on a number to produce a unique result. [When the operation adding "^2

operates on "8, the result is "6, and nothing else; when the operation
absolute valuing operates on "4, the result is 4, and nothing else. ]

This notion of uniqueness of result is the essence of the idea of opera-


tion. So, let us agree that a set of pairs is an operation just if no two

pairs in the set have the same first number.


Now, from any operation you can get a second set of pairs by
reversing each pair which belongs to the given operation. For example,
when you reverse each pair in the operation adding '^7, you get a set of
pairs. If you think a moment, you will see that this second set of pairs
is an operation. In fact, it is the operation we call 'subtracting *7'.
[1-108] [1.13]

But, is it always the case [as here] that when one reverses the pairs
in an operation, the new set of pairs is also an operation? Let's try

it with the operation absolute valuing. Here is a list of some of the

pairs in the new set.

/ (7,-7) (7, ^7) (0, 0) (5, *5) \


\

(5, -5)
/

Is this set of pairs an operation? No, because it contains pairs with


the same first number but different second nuinbers. Since this set
of reversed pairs is not an operation, we say that absolute valuing does

not have an inverse. Can you think of another operation which does not
have an inverse? In general, an operation has an inverse just if the
set of its reversed pairs is an operation.
Let's look again at the set of pairs we get by reversing the pairs in
absolute valuing. Even though this set of pairs is not itself an operation,

we can "split" the set into two sets each of which is an operation.

(7, -7) (5, -5) (6, -6) (7, ^7) (5, ^5) (6, *6) ^
I
/ \

\ (104, -104) (1, -1) (0, 0) I (104, n04) (1, n) (0, 0)

\ ... y
Notice that the operation listed on the left is an operation on the set of
numbers of arithmetic. So is the one listed on the right. [If (0, 0) were
not included in one of these sets, the set would still be an operation,
but not on the set of numbers of arithmetic. Why?] When the first
operation operates on a number of arithmetic, the result is the corre-
sponding nonpositive real number. What is the result of operating on
numbers of arithmetic with the second operation? Do you recognize
these operations? You learned about them when we first mentioned
real numbers. These operations do not have standard names, but we
have been using two signs almost as though they were signs for the
operations. These are the signs '
" '
and ' * '. If you apply the opera -
tion - to the number 7 of arithmetic, you get the corresponding
nonpositive real number "7. If you apply the operation * to the number
58 of arithmetic, you get the corresponding nonnegative real number *58.
[1-109]

'
and

k
. .

1/121

1. ?al msntber
3. ir-.i.-. — etic
5.

7. t.

9. 10.

11.

-: -5 - ?zr: z :" rage l-llOj.

1. .-. :5;.-.r ;..-.-; i; r .. es only to real mooibers.

2. T r. r rrfriT'-C". iTr.irs :~..'" t c> T^iamb'c r 5 zi a-mr-rr-cTic.

3. T.-e :r r r i:i : - 177'. -f £ :-'.:: --—.rsrs af arithmetic.

s. - " - rrtrE.t'. rr. itt./. tf oaiLxj' to ~i'i.sm d£ rs ci 3.71"^.^.; tic.

6. T"-T :rrri:.;~. izr-iss OHily to nDOBmr t r ; : 1 1r .


:
".
~ e : : : ,

l^fi.Ti.", \; * -i -i i-wC i aaiaae for a. mad; e r ;; 1? .:.". ~. rticj.

7. Al>£ : 1 . .
- : L - 7 e 3 oaaly to rea.'. r. . —. : i r 5

51 tc ;.zi-. .:- i: ;:-.;— ::' page 1-110:

*||~2||* caa be assterpreted as an abbrevxatiom fox *^|*|~2||*


or for * \*'^~2 "* and so may be a laame for eiltiber *Z or tbs
itT ^^ ' iti]unetic»
•- j3 - 4 j* can be interpreted as an abbreviatian f or '-*]*^3 - *4|'
s^i :'
F D iiaterpreted, is a ::^£.— ^ :'
: r ~1

TCjl-109, 110)
[1.13] [1-109]

Notice that \wj Cy belongs to each of these operataons. To use '"' and
'** as sLzr.s frr the operations, we must define *~0' and '"^0'
to he
numeraiS i-r "..- - r : il num.'ber 0. f V/aming Even tiioiigla "we are
claiming that "C' - and *0 = 0, is still neitlieT negative nor positive.
But, it is btrh r-rrtsitive and ::' ;./.egative. ]

EXERCISES
A. For each r _-:_ : _- r : : : : v d, tell by checking in the appropriate colunan
whether it is a. rs^L-r.-^s.r of arithmetic, a positive real number, a
negative real number, or the real number 0. [Since iiaroughout
this section on ah; ;h--e valuing it has been essential to distinguish
betv.e^r. r.i~/icrs cd arithmetic and real numbers, we h.ave not used
'S:~ : :r.-' ;:".:: jn according to -which a numeral for a numl>er of arith-
r:.-;-. . _ :: _. ^ _ ::
- : ; ;.me the corresponding nonnegative real number.
IVfc 1 : r_: _ r_ ^ _ :.-.: ; z zlicy in the table. ]

!M ^ Jl »H
'
Ei o a
w jrs O XI J3

1^ 13
>B
^ -3
> ?
=!
a
3
43
w
O
Z tJ
20
4

JX
_^ —
^
^
, ts
BJ ^
2; tS
tB
N\imber ;
J2J XS^ ci

1. 17^3)
z. ^ - no '
!

-173
{1
!

I
1

45 - 45
,1

*45 - "455

6- i 1
1

7- '1
J 1

1 i

'
"1 r
-

^1 1
1
f

10. »
1
i

II. 11^1 1

1
[1-110] [1.13]

B. Each of the following marks looks like a numeral but isn't because
it doesn't stand for a nuniber. Explain why.

Sample 1_. | 3 |

Solution. Absolute valuing is an operation which is applied


only to real numbers. Since '3' stands for a num-
ber of arithmetic [recall that we are not using
ambiguous numerals in these exercises], '
|
3 |
'
is

nonsense.

Sample 2. ^("3)

*
Solution . The operation applies only to numbers of arithmetic.

1. 12 2. -("5) 3. ^("2) 4. |"2|


I I I I

5. *(^3 - *4) 6. "(3-4) 7. -|3 - 4|

Some of the expressions in Part B do make sense if they are


interpreted according to the convention that a nunieral for a number
of arithmetic stands for the corresponding nonnegative real number.
[Even with this convention, there are still soiTie expressions in Part B
which do not make sense. ] In Exercise 1, if we regard '12' as stand-

ing for *12 then '


|
12 |
'
is a numeral for a number of arithmetic. But,

since this is the case, we can use the convention again and regard

I
12 I
'
as standing for *12. So, if you see '
|
12 |
'
in a place where it

is intended to make sense, you will know that '12' is being used as a
name for "12. But, you will have to look further in order to decide

whether '
|
12 |
'
is being used as a name for the number 12 of arithmetic,

or as a name for *12. For example, if you come upon the sentence:

I
12 I
- 3 = *9

asid you believe that it is intended to make sense, you will interpret

I
12 I
'
and ' 3' as names for nonnegative real numbers. But, if you see:

I
12 I
-3 = 9,

then, without additional information, all you can be sure of is that ' 12'

stands for *12, and that either '


|
12 |
',
'3', and '9' all stand for numbers
of arithmetic, or all stand for nonnegative real numbers.
Two other expressions in Part B which can be interpreted to nnake
sense are those in Exercises 4 and 7. Tell how to make sense out of them.
[1-111]
1/122

Answers for MISCELLANEOUS EXERCISES.


A. 1. 1 2. -9 3. 5 4. 5 5. -20
6. -21 7. 33 8. 8 9. 2 10. -12
11. -.3 12. -2 13. 3. 3 14. -5.4 15. 14.1

16. -6.4 17. 1 18. -1 19. 18 20. 19

21. 4 22. -8.3 23. 5 24. 36 25. -63


26. -66 27. 72 28. 42 29. -132 30.

31. 4 32. -9 33. -3 34. -6 35.

36. 37. -9 38. -4 39. -4 40. -66

1. F 2. T 3. T 4, T 5. T
6. F 7. F 8. T 9. T 10. T
11. F 12. F 13. F 14. F 15. T
16. T 17. T 18. T 19. T 20. T
21. F 22. T 23. T 24. T

C. 1. (a) n (b) *9 (c) "9 (d) no


(e) -2 (f) * 32.5 (g) *2.2 (h) -112 »

2. "3 3. Q corresponds with -3; W corresponds with

4. Points A C s
(a) "19 "11 -29

(b) -25 "17 -35

(c) "22 "14 "32

(d) "21. 5 -13.5 -31,.5


-
(e) "127. 2 119.2 -137 .2
(f) 3964. 7 3972.7 3954,.7

5. -2

TC[1-111, 112, 113]


[1-111]

MISCELLANEOUS EXERCISES
A. Simplify.

1. +3 +-2 2. -3 - +6 3. -3 --8
4. 6 +-1 5. - 1 2 + -8 6. -16 - 5

7. 27 --6 8. 5 - -3 9. -5+7
10. -15 + 3 11. + 1.3 - +1.6 12. -5+3
13. 5. 6 +-2. 3 14. -7.8 + 2.4 15. 10. 7 --3.4

16. -3. 5 +-2.9 17. + 5 --3 +-7 18. -2 --4 - +3

19. 7+9 --2 20. 24 +-6 --1 21. -13 + 8 --9

22. 1. 2 +-1. 7 +-7.8 23. 0. 5 X 10 24. -3 x-12


25. -7 X +9 26. 11 X -6 27. 6X12
28. -2 x-3 X +7 29. 11 X -3 X 4 30. X— 3 X —
31. -12 ^-3 iZ. 18 v-2 33. -24 ^ +8

34. -150 V 25 35. +-5 36. T +250

37. -27 ^ 3 38. 32 ^-8 39. + 48 +-12

40. 198 -f-3

True or false ?

1. |-4| + |+4| = 2. |+4| + |-4| - |-8|

~^3. |-3| - 1 + 3] = 4. |-2| - -3 = 5

5. -4 +-2 > -7 6. I
-1
I
+ -3 - j-4| = -8

7. | + 6{ + |-6| < | + 6| + |+6| 8. |


+ 5| + i
+ 5| > | + 5| + |-5

9. 3 + |-2| - |-1 I
- 4 10. -7 + |-2j + =-5
11. -10 =-7 + |-3| 12. |-5! > |-6|

13. +10 =-11+1 14. 3 - 2 < 2 - 3

15. | + 3| - "| + 2| = 3 + 2 16. |4| - |-2| < 4 - -2

17. 1 + 1
I
- |-2| < +1 --2 18. |-2| X |-3| =-2 x-3
(continued on next page)
[1-112] [MISCELLANEOUS EXERaSES]

19. -2x+3^-2x+3 20. |


+2 1 x |
+ 3 j
= +2 x +3

21. |4-7|.-|7-4| 22. | +2 -


-1 3 i
= |-1 3 - +2 i

23. |-5 - -3| = |5 - 3| -i24. |-I 5 - 2 |


= |
2 + I 5 {

When we think of the real numbers as points of a line, the number


line, we can also think of trips on the number line. We have already
decided how to find the distance between two real numbers, so we
can also use real numbers to ineasure trips on the number line
once we have chosen a positive direction. It helps in understanding
many problems to translate them into problems concerning trips
on the number line. For this purpose it is most convenient to choose
the direction fronn to *1 as the positive direction. [For example,
the nneasure of a trip from * 7 to "^1 is 6. ] We shall use this choice

in each of the following exercises.

1. What is the measure of each of the following trips?

(a) from to *! (b) from *6 to *15

(c) from *15 to *6 (d) from '3 to *1

(e) from "6 to "8 ( f) from ""5.7 to *38.2

(g) from -7.6 to "5.4 (h) from "127 to "239

--^2. [Draw a picture of the nunnber line and label some of its points

to help you with this problem. ] A trip from point M to point N


is measured by +2, a trip from N to P is measured by -6, and
a trip from P to D is measured by— 2. If the point N is +5,

what real number is point D?

3. A trip from W to Q is measured by +9, and a trip from Q to

A is measured by +7. If point A is +4, what real numbers are


the points W and Q?
[1-113]

V
.

1/123

D^. 1. Using the positive direction for a profit, the answer is


*2.75 dollars.
2. Using the positive direction for points gained, the player's
score at the beginning of a round could be represented by
**5', so the change is measured by "7, and his score at
end of final round was "2.
3. -12"

4. "10

5. Using the positive direction for temperatures above zero,


the answer is ^1"

6. 26, 100 dollars.

7. "16"

8. Using the positive direction for an increase in water volunne,


the answers are:
(a) *17 gallons

(b) There must have been at least 25 gallons of water in the


tank, but there could have been many more. The largest
amount would depend, of course, on the size of the tank.
(c) 25 gallons
(d) 33 gallons

TC[1-113, 114]
[MISCELLANEOUS EXERCISES] [1-113]

4. The measure of a trip

from C to R is 14, from R to S is — 32,


from S to A is +10, and from A to C is +8.

Give the real numbers which are the points A, C, and S

(a) if point R is 3. (b) if R is —3.

(c) if R is 0. (d) if R is | .

(e) if R is —105.2. ( f) if R is 3986. 7.

5. A trip from A to Z is measured by —5, and a trip from Z to

C is measured by +9. If point A is— 6, what is point C?

D. E^ch of the following problems involves changes, [in Exercise 1

the change is in financial status, in Exercise 2 the cliange is in

point standing, etc.] Choose a direction of change for the positive


direction, and use real numbers to measure changes.

1. Bill is in business and loses $2. 00 on one day and makes a


profit of $4. 75 on the second day. What is the change in his
financial status over the two-day period?

2. A player has 5 points at the beginning of a round, loses 3 points


during that round, gains 7 points the next round, loses 12
points the next round, and gains 1 point the final round. What
is the change in his point standing from the beginning of the first
round mentioned to the end of the final round? What was his

point standing at the end of the final round?

3. If the temiperature is +15° Fahrenheit and it drops 27°, what


is the temperature at the end of the drop?

[Note: In doing these problems does it help if you think of "trips"


along the number line ? ]
[1-114] [MISCELLANEOUS EXERCISES]

^4. Let us say that the main floor in a department store corresponds
with the real nuinber 0, that the floor 3 levels below corre-

sponds with the real number -i 3, and the floor 2 levels above

the main floor corresponds with -Z. What real number corre-
sponds with the floor at which the elevator stops after moving
11 floors up from floor f 1?

5. The highest temperature on January 3 in Chicago was 10" above


zero. On January 4 the highest temperature was 2° higher
than on January 3, and on January 5 it was 5° lower than
January 4. What was the highest temperature on January 5?

6. At the end of one year, a firm had a balance of $-10, 70C,


and at the end of the next year, the balance was $15, 400.
How much better off was the firm at the end of the second year?

7. The temperature at 7 p.m. on a certain aay in New York City


was 7°. By 3 a.m. of the next day the temperature had
dropped to —9°. What was the change in temperature from
7 p.m. to 3 a.m. the next day?

-^8. During one day 12 gallons of water was pumped out of a tank,
and that night 17 gallons was pumped into the tank. During the
next day 30 gallons was pumped out, and 42 gallons was pumped
into the tank that night.

(a) What is the change in volume of water in the tank from


the morning of the first day to the morning of the third

day?

(b) How much water was in the tank on the morning of the
first day?

(c) What is the least amount of water which could have been
in the tank on the morning of the first day?

(d) How much water was there in the tank on the morning of the
first day if there were 50 gallons in it on the morning of the
third day?
[1-115]
[1-114]

1/124

E. 1. 21 2. 10 3. -108 5. -1
6. 1, -1 7. -2

F. Here are the lists.

1
2
1

'z
6 1
! ^¥
12
31 5 ^ 121
-3 12
62 11 ^ 15
0.5 -.0042
|x(-i) 40 - -4
-.0042
350% of i 12

-2 1
20% of (100% of 5)1
2
" 4 -8
1

4
-4 -16 ^^2
-2X -i
22
-8
3 + 7
T
6 + 4
3 6 150% of 2
1
-1 - -4
38 X ,\ 1

19

1. 10 2. -2 3. 30 4. 5. 2
6. -4 7. 6 8. 12 9. 12 10. -5
11. 25 12. 9 13. 6 [or: -6] 14. 9
15. 9 16. 3 17. -1 18. -1 19. -14
20. oppositing [or: subtracting 14, or: multiplying by -1,
or: dividing by -1, or: adding -14]
21. 7

TC[1-115. 116]
[MISCELLANEOUS EXERCISES] [1-115]

E. Guess the number.

1. A certain number is added to— 5, and the result is 16.

2. —5 is subtracted from a certain number, and the result is 15.

3. A certain number is divided by 6, and the result is — 18.


4. A certain number is added to its reciprocal, and the result
is 2.

5. If I add a certain number to its reciprocal, the sum is —2.

6. If I add the opposite of a certain number to its reciprocal the


sum is 0.

7. If I add a certain number to + 1 , I get— 1.

F. Rearrange this list into columns with all the numerals for the
same number in one column.

1_ 6
2 2
-16
62
22
150% of 2 11
0. 5 6
12

— . 0042 i —3
— . 0042
350% of
TT >^
W-

--4 -2
40
4 -1 --4 20% of (100% of 5)
12

—4
1 1
+3
3 + 6
X (-1)
f

.2X-- 3+7
6+4 38 x^ 16
32 ^ °
[1-116] [MISCELLANEOUS EXERCISES]

Fill in the blanks to make true sentences.

Sample . (9, )
belongs to the operation adding 5.

Solution . Since 9 + 5 = 14, we can nnake a true

sentence by writing a '


14 '
in the blank.

1. 3, . ) belongs to adding 7.

2. ;-8, _) belongs to adding 6.

3. -5, ) belongs to multiplying by— 6.


4. -7, ) belongs to subtracting —7.

5. ,
11) belongs to adding 9

6. , 36) belongs to multiplying by— 9.


7. 8, )
belongs to the inverse of adding 2.

8. 9, )
belongs to the inverse of adding "3.

9. |— 12, )
belongs to oppositing.

10. ,
— 5) belongs to sameing.

11. [—5, )
belongs to squaring. [To square a number is to

multiply it by itself. ]

12. + 3, ) belongs to squaring.

13. , +36) belongs to squaring.

14. —27, ) belongs to dividing by— 3.


15. "•27, )
belongs to multiplying by the reciprocal of —3.

16. —3, ) belongs to adding the opposite of —6.

17. 7, •*-7) belongs to multiplying by .

18. 7, —7) belongs to dividing by .

19. 7, —7) belongs to adding .

20. 7, —7) belongs to .

21. ""7*
) belongs to absolute valuing.
[1-117]

W
•w
[1-llb]

1/125

H. 1. -6 2. -28 3. -31 4. 43
5. -67 6. -46 7. -36 8. 105

9. (a) 24.366 (b) 7.7825 (c) -4j (d) 1 (e) -^


10. (a) -5 (b) -6 (c) -7j|- (d) -12 (e) -17

11. 74

To complete the table:

Feb. 68.6°, March 69.0°, April 70. 1 °, May 72.0°,


J\ine73. 4°, July 74.3°, Aug. 75.0°, Sept. 75.7°,

Oct. 74.6°, Nov. 73.7°, Dec. 70.3°.

Answers to questions on p. 1-119.

1. 72. 13° [Draw a line at '72. 13' to simplify Exercise 2. ]

2. Jan. 3.33, Feb. 3.53, Mar. 3.13, Apr. 2.03,


May 0.13, June "1. 27, July "2.17, Aug. "2.87,
Sept. "3.57, Oct. "2.47, Nov. "1.57, Dec. 1.83.

3. The positive differences total *13.9, and the negative dif-


ferences total "13. 9.
4. Yes 5. Yes!

J. 1. 32°, 18°, 6°, "7°, "15°, "23°. 2. 10°


3. Nome, 3° 4. 1 1
°
5. Ketchikan 6. 1, 12

7. This will need some discussion; we merely wauited students


to do some speculating about the uses for information of
this kind.

TC[1-117. 118, 119, 120]


[MISCELLANEOUS EXERCISES] [1-117]

H. Solve these problems.

1. The average of two numbers is 12. One of the numbers is 30.

What is the other number?

2. Four numbers average— 7. What is their sum?


3. Two numbers average —22. One number is — 13. What is the

other number?

4. Three numbers average +14. One number is —26; a second


number is +25. What is the third number?

5. Two numbers average —25; one number is +17. What is the

other number ?

6. Two numbers average —33; one number is -*20. What is the

other number?

7. There are four numbers whose average is— 9. What is their


sum ?
8. Two numbers average +27; one number is — 51. What is the

other number?

9. Find the average for each set of numbers.

(a) {3.5, 6.09, 2.37, 100.12, 9.75}


(b) {3.89, 12. 1, 7. 14, 8.0}
(c) { + 5. -14}
(d) { + 3, -101, +105, -3}
,

(e)
r
1-g
3
i _ 3 j_5 ,

, + 2 .
4 ' ""24

10. Fill in the blank so that the average for each set is —2.

(a) {-6, -9. +12, }

(b) {-9, , 9}
, . r
1 _i _ 3,
\^i ^ Z ' '
3 '
4
(d) {-3.0, -3.4, , +2.0, +6.4}
(e) {2, -2. -1, 0, , 3, 1}

11, The average of three numbers is 47. One of the numbers is

67. What is the sum of the other two?


[1-118] [MISCELLANEOUS EXERCISES]

Below is a chart which gives the mean (average) temperature


for each month of a certain year in Hawaii. Use the chart and
complete the following table. Then answer the questions which
are at the top of the next page.

Mean Mean Mean


Month Temperature Month Temperature Month Temperature

Jan. 68. 8" May- Sept.

Feb. June Oct.


March July Nov.
April Aug. Dec.

Monthly Mean Temperatures in Hawaii

76.

75.

74.

73.

72.

71.

70.

69.0

68.0
Jan Feb Mar Apr May June July Aug Sept Oct Nov Dec
[MISCELLANEOUS EXERCISES] [1-119]

1. Find the mean of the mean monthly temperatures for the year.

2. Subtract the mean temperature for each month from the mean
for the year.

3. Add all the differences obtained in Exercise 2.

4. Did you get for Exercise 3?

5. Suppose you subtract the mean of a set of numbers from each


of the numbers. The resulting differences are called deviations
from the mean . Do you think that, for each set of numbers,
the sum of the deviations from the mean is 0?

J. The bar chart on the next page shows mean January temperatures
in Fahrenheit degrees computed over a period of years at some of
the Alaska weather stations.

1. Give the mean of the January temperatures at the following

weather stations: Ketchikan, Valdez, Susitna, Ruby, Tanana,


and Dawson.

2. According to the graph, how many degrees colder was


Anchorage than Seward?

3. What station had the temperature closest to ? What was


this temperature?

4. How many degrees warmer was Ruby than Barrow?

5. What was the warmest station?

6. How many stations had their mean January temperature


above freezing? How many below freezing?

7. The mean temperature for the thirteen stations is +2.8°.


What practical use could be made of this fact?
I

[1-120] [MISCELLANEOUS EXERCISES]

-cr

t/1
+ 40' -4>
(U
CO

+ 30' -^3-
(1)

0)
M
I— O
iTi

+ 20' ->- ..U-. nJ" -ts-


o
0) " c 3
?
+ 10'
— co- -S
o
^
-^^ >
rt
5— -.§. ....GM
nJ
C vi
o
•to-
?
*->


n[.- 1 ci ;d
IXJ

H
nJ

m Q
nJ O

-10°

-20'

30'

Fahrenheit Temperatures at

Various Alaskan V/eather Stations

K. Punctuate to make sense.

1. Ray was making a list of the class. I"irst he wrote Jim. Beside
Jim he wrote 1. Then he wrote Helen and beside Helen he
wrote 3. He looked at the 3 and realized that he had made an
error. He erased the 3 and this time put 2 beside Helen.

After writing 13 names he wrote 14. When he finished the


list he had 23 names and 23 nunnerals.

2. Harry in the sentence in which Chester had had had had had
had had had had had had the approval of the examiners.
1/126

K. Here is one way to punctuate the sentences.

1. Ray was making a list of the class. First he wrote 'Jim'.


Beside 'Jim' he wrote 1'. Then he wrote 'Helen' and
'

beside 'Helen' he wrote '3'. He looked at the '3' amd


realized that he had made am error. He erased the *3' and
this time put '2' beside 'Helen'. After writing 13 names
he wrote 14'. When he had finished the list he had 23
'

names and 23 numerals.


2. Harry, in the sentence in which Chester had had 'had had',
had had 'had'; 'had had' had had the approval of the exami-
ners.

Answers for TEST.

I. 1. -5 2. 2 3. 12 4. -9 5. 3

6. -11 7. -13 8. 10 9. -5 10. 606


11. 19.2 12. -14 13. -51 14. 15.

16. 14 17. 30 18. -50 19. -48 20. 8

II. The book was on the table but it wasn't on a 'table'. If the
book were on a 'table' that was on a paper on the table then
you couldn't see that a 'table' was on the paper on the table.

m. (b) IV. (d) (d) VI. (a) VII. (c)

VIII. 1. > 2. > 3. < 4. > 5. =

6. > 7. > 8. > 9. = 10. >

TC[1-120, 121. 122]


[1-120]
[1-121]

TEST
I. Simplify.

1. -4 + "1 2. 7 + "5

3. 10 + 2 4.-9+0
5. + 3 6. 2 + "13

7.-9-4 8. 7 - "3

9. +2 - ^1 10. 317 - '289

11. 11.2-16X J 12. 87|-%of-16

13. 13X-3+4X"3 14. (9 X "2 + "3 X "6) -^ (5 - 8)

15. ~\^(^'\^\) ^^- "7 X 2 X -1

17. -2X[-3X5] 18. 5 X {-2 X [-3 + 8]}

19. -3 X [-5 X -2 + (-1 + 7)]

10 X -

II. Use single quotes to punctuate the following paragraph so that


it makes sense.

The book was on the table but it wasn't on a


table. If the book were on a table that was on a
paper on the table then you couldn't see that a
table was on the paper on the table.

III. You can convert the sentence:

5 4 + 5 ^ 9 = 5 ^ (4 + 9)
J

into a true one by replacing each '


^
'
in it by which of the
following synnbols ?

(a) + (b) X (c) V (d) - (e) /


3 5
'

[1-122] [TEST]

IV. Which of the following is a name for -8 + 13?

(a) 8 4 -13 (b) 8 f 13 (c) -8 + "13

(d) 13 - 8 (e) -(-8 - 13)

V. Which of the following is an instance of the corrimutative principle


for multiplication?

(a) 10 X \^
,
X i^
f 1 .
:r
Uo
10 X - iV<X 4
-

(b) ^ =<-7 = ~| X 7 (c) ^7 + -2 = -2 + -7

(d)
I
X ^7 =
~f X ^ (e) '12 + (8 X -3) =: {'12 + "3) X 8

VI. Which of the following is an instance of the associative principle


for addition?

(a) (-8 + -9) + 2 = -8 4 (-9 + 2)

(b) *2 + (-8 + -9) = ^2 + ("9 + -8)

(c) (-8 + 2) + "9 = (-8 ( -9) + 2

(d) 2 X (-8 + -9) = 2 X -8 + 2 X -9

(e) (-8 X "9) X 1 = -8 X -9

VII. Simplify.

*2 X (*3 X "2 J -5 X -^4) + "10

(a) -42 (b) +42 (c) "62 (d) "72 (e) "18

VIII. Between the two numerals given in these exercises insert one
of the symbols '
> ',
'< 'or '= '
so that the resulting sentence
is true.

1. .52 .498 2. "3.45 -3. 72

1 2
3. 4. 756. 784.
7 9
1 -
5. .0025 6. . 032 . 033
400
4 2
7. " 8.
5 11

9.
7+3 167
0.
1 1
T -1
334 -4

.4 . 5
1-123]

\
[1-122]

1/127

1. F 2. F 3. F 4. F 5. T
6. F 7. F 8. T 9. F 10. T
1. T 12. p 13. T 14. F 15. F
6. T 17. T 18. T 19. F 20. F

(0, -7), ("2, -9), (3, -4), (5, -2), (10, 3) . . . .

2. (2, 5), (0, 3), (-1, *Z), (-10, -7), (21, 24) ... .

3. (20, -5), (4, -1), (2, -j), (-8, +2), (-16, +4) . . . .

4. /*8, "8), (-6, *6), (-3, *3), (^01. "101), ("2.5, *2. 5)

5. (10. -20), (1, -2). (-4, +8), (-10, +20), (-2, +4) . . .

XI. (c), (e) XII. (c), (d)

1. C 2. F 3. B 4. E 5. H
6. D 7. I 8. A 9. G 10. D
11. B 12. A 13. J

XrV. 1 . No! [The team had a net loss of 8 yards. ]

2. $ 1686.68

TC[l-123, 124, 125]


[TEST] [1-123]

IX. True or false?

1. -1000 > -2 2. -10 > 8

3 -^
71
>-^
71
4 -ii <-5
3

5. 0.016 > 0.0016 6. -0.016 >-0.00l6

9. -^ > 1 10. -14Z5 <

11. +|*7| ^ |-3i 12. l-lOOOlj > I'lOOOZ]

13. '!5.47| < |5.469| 14. [8 - J3.5| >'-|2.466|

15. 1-1278. 543| =-|l278.543|

16. -|H| < -|2 466| 17. f|i^ It


18. -i < -i^ 19. |87| > 92 -

20. . 0079 ^ -\. 00791 |

X. 1. List 5 pairs of real numbers which belong to the operation


adding —7.

2. List 5 pairs of real numbers which belong to the operation


which is the inverse of subtracting 3.

3. List 5 pairs of real n'ombers which belong to the operation


dividing by — 4.

4. List 5 pairs of real numbers which belong to the operation


oppositing.

5. List 5 pairs of real numbers which belong to the operation


which is the inverse of dividing by— 2.

XI. Which operations are the same as multiplying by — 3?

(a) dividing by -3
(b) multiplying by the reciprocal of —3
(c) dividing by the reciprocal of -'3
(d) the inverse of multiplying by —

(e) the inverse of dividing by —3


[1-124] [TEST]

XII. Which operations are the same as subtracting 6?


(a) adding the reciprocal of 6

(b) the inverse of subtracting 6

(c) adding the opposite of 6

(d) the inverse of subtracting -6


(e) the inverse of adding -6

XIII. Each of the following sentences is a consequence of one of the

principles you studied in this unit. Below them are the names of
these principles, each being preceded by a letter. In the blank
at the left of each statement write the letter corresponding to the

principle which is illustrated.

Sample: _I_ 0. 5 + -5 = [Note The letter


: T
is placed
alongside the statement because
it illustrates the principle of
oppo sites. ]

1. 3 + (4 + 7) = (3 4 4) + 7

2. (5 + 0) + 7 = 5 + 7

3. 4 + (7 X 9) = 4 + (9 X 7)

4. (6 X 2) + (4 X 2) = (6 + 4) X 2

5. (3 X 4) X 1 = 3 X 4

6. [(8 X 13) X 7] + 5 = [8 X (13 X 7)] + 5

7. -7 + -"7 =

8. 3 + (8 + 5) = (8 + 5) + 3
9. 6 X (3 X 0) = 6 X
10. ("587 X 169) X = '587 X (169 X
ylg j|^)
11. (7 X 0) + 5 = (0 X 7) + 5

12. 2 X [(8 + 4) + 3] = 2 X [3 + (8 + 4)]

13. 9 - 5 = 9 + -5

A. Commutative principle for addition


B. Commutative principle for multiplication

C. Associative principle for addition


D. Associative principle for multiplication
E. Distributive principle for multiplication over addition
F. Principle for adding
G. Principle for multiplying by
H. Principle for multiplying by 1

I. Principle of opposites
J. Principle for subtraction
[TEST] [1-125]

XIV. 1. In a football game,, the team from Zilchville High got


possession of the ball and gained 5 yards on the first
down. Then they lost 12 yards, gained 2 yards, and
lost 3 on successive plays. Did they keep possession
of the ball?

2. Below is a record of Mr. Sellars bank account for one


week. Determine his balance at the end of the week.

Monday -- Balance on hand: $1297.58; deposit: $415.00;


checks paid: $56.75, $32.19, $77.95.

Tuesday -- Checks paid: $41.68, $8.92, $13.12, $87.78.

Wednesday -- Deposits: $219.37, $682.46; checks paid:


$486. 39, $17.62.

Thursday -- Checks paid: $63.97, $39-76, $102.96.

Friday -- Deposits: $57.81, $43.55.


[1-126]

SUPPLEMENTARY EXERCISES

A. Use single quotes in punctuating each of the following paragraphs


in order to make sense out of it.

1. Marika, who just arrived in this country from Greece, went


to the First National Store to buy a box of Kleenex. She
walked up one aisle and down another until she saw a stack of

boxes, each one of which had a Kleenex on it. She looked for
the price and finally found 2/Z9 on the end of a box. She
wondered if this meant that you could buy Z9 boxes for 2

dollars or if it meant that 2 boxes cost 29 dollars. Neither


possibility seemed very reasonable to her. She tried cross-
ing out the 2s, but she wasn't sure whether she should get

0/9 or l/l9. She also thought about dividing both 2 and 29


by 2; since she wasn't sure, she decided to ask the clerk.

2. John was 8 years old. At his birthday party he had a cake


with a small 8 on it. Ke wrote ate on a piece of paper and
put the paper beside the cake. He put ate by the 8, but he
didn't have 88. He had 8 ate. He could find the sum of 8

and 8 but he couldn't find the sunn of 8 and ate. So, he ate
the 8. But, he didn't eat the ate. The ate was left because
it hadn't been eaten yet. John tried to eat the ate, but the
ate was too big to be eaten. John took the e off the ate [it

didn't hurt] and put it in front of the at. Then it spelled eat.
So he did. He ate the eat. It was awful.

3. Mary is quite confused. She is Mary but yet she is not Mary.
She said, "If my name is Mary then I must be Mary. But
you say I am not Mary. If I am not Mary, why does every-
one call me Mary?" I Mary that if she wrote Mary on
told

a piece of paper, that was Mary but not Mary. The reason
for this is that Mary is Mary but Mary is not Mary. Do you
think Mary will ever understand this? I hope so. I know
that Mary doesn't understand this because it can't.
1/128

Answers for SUPPLEMENTARY EXERCISES.


A. 1. In line 4, write single quotes about the 'Kleenex'; in line 5,
about '2/29'; in line 9, about '2'; in line 10, about '0/9' and
'1/19'.

2. In line 2, write single quotes about '8* and 'ate'; in line 3,


about 'ate' and '8'; in line 4, about '88' and '8 ate'; in
line 5, about '8' and 'ate'; in line 6, about '8' and both
'ate's; in line 7, about 'ate'; in line 8, about both 'ate's
and the 'e'; in line 9. about 'at' and 'eat'; in line 10,
about 'eat'.

3. In line 1, write single quotes about the third occurrence of


'Mary'; in line 2, about the first occurrence of 'Mary'; in
line 3, about both 'Mary's; in line 4, about the first and
third 'Mary's; in line 5, about the first 'Mary'. Line 6
may be punctuated
'Mary' is 'Mary' but Mary is not 'Mary'
or:
Mary is Mary but 'Mary' is not Mary
In line 8, write single quotes about 'Mary'.

TC[1-126]
[1-126]
\
1/129

11 11

5. "2 6. "1 7. -14 8. ns


9. "14 10. "8 11. "22 12. ns
13. *4 14. ns 15. "47 16. "70

17. "55 18. "20 19. n 20. "8

21. "5 3 22. "15 23. "46 24. ni


25. no 26. *5 27. "23 28. n
29. "20 30. *6 31. "17 32. "548

"7 " 11
33. *3 34. 35. 36.
6 3 35

* 1
"1 " 11
37. 38. 39. 40.
15 8 5 12

"3 * 31
41. 42. 43. *\Z. 32 44. "2.62
7 6

45. -17. 07 46. "0. 37 47. ~4. 07 3 48. *0. 93

i9.
i *5.02 50. "6.73

TC[1-127]
[SUPPLEMENTARY EXERCISES- -Part B] [1-127]

B. Simplify.

1. ^8 + "3 2. -7 + "9 3. -3 -f
"6

4. "2 + "9 5. "5 + -7 6. -8 + "7

7. '4 + "10 8. "12 + "3 9. -5 -f


-9

10. "12 + M 11. "17 + -5 12. "18 + -3

13. "17 + ^21 14. "33 + "15 15. -51 + "4

16. -62 + "8 17. -5 + -50 18. "3 + "17

19. {^2 + *3) + -4 20. ("5 4 -8) + "5

21. ("51 + *5) + "7 22. (-7 + "9) + -17

23. {"41 + -10) + ns 24. (-5 + -11) + "27


"
25. "6 f (-3 + "7) 26. "8 + (
"9 + 12)

27. "12 + (-3 + -8) 28. "10 + (-4 + -5)

29. ("4 + -7) +(-5 + -12)

30. ("6 + "9) +(-9 + "12)


"
31. (-71 + "40) + (-35 + 49)

32. ("124 + -72) + ("584 + -16)

33. *| + 'i 34. "il-rA 35. *| + '|

36. *|.ri 37. "i^l 38. '1^1


3,. ,i.-|v*i 40. ri.-i)r|
„. (-|.-iv^ «. r|.*if). 5
6

43. "4. 03 + "8. 29 44. -5.21 +"7.83 45. -7. 83 + -9. 24

46. "2.1 +"1.73 47. "3. 08 + -7. 163 48. "5. 9 + "6. 83

49. ("5.93 + -7. 12) + "6. 21 50. (-5. 08 + "0. 35) + -2


[1-128] [SUPPLEMENTARY EXERCISES--Part C]

C. Answer each of the following questions.

1. On the New York Stock Exchange, a certain kind of stock


whose par value is $50 (per share) was listed on Monday at
1 7 1
62 5- , on Tuesday at 61- , on Wednesday at 6I5- , on Thurs-
1 3
day at bl-^ , and on Friday at 61— .

(a) Use real numbers to list the changes from each day to

the next day.

(b) Mr. Brockman sold 25 shares of this stock on Tuesday,


and Mr. Stockert sold 25 shares same kind of
of the

stock on Friday. Mr. Brockman received how much


more money than Mr. Stockert?

2. The table below gives the T. V. weatherman's report of the

high, the low, and the normal niean temperatures in a

certain city for the first week of June.

Normal
Day High Low Mean Mean

Monday 87 62 72
Tuesday 85 57 70
Wednesday 80 49 68
Thursday 83 53 65
Friday 78 51 66
Saturday 75 48 68
Sunday 77 56 67

(a) Compute mean temperature for each day. [Weather-


the

men compute the mean temperature by averaging the high


and low temperatures. This does not give you the exact
mean but does come close. ]

(b) Use real numbers to indicate the difference of the mean


from the normal mean for each day.
1-129]

8' 8 '
8

(b) $3. 125

2. (a) Mon. 74.5, Tue. 71, Wed. 64.5, Thu. 68, Fri. 64.5,
Sat. 61.5, Sun. 66. 5.

(b) "2.5, -1, *3.5, "3, n.5, ^6.5, ^Ij

(c) -2, -5, ^3, "5, -3, *2

(d) -5. -8, "4. -2. -3. ^8

3. (a) -. 5, \ 8, -. 9, *. 2

(b) -1.3, -1.2, \4, "1.1

(c) 100.97, 100.47, 100.30, 100.17, 100

4. ns

5. Yes, 5 units Northwest of the pump. [Bob hikes three


times as fast as John. ]

6. 5 blocks from Nick's honne (going toward Bill's home)


[or: 1 block from Bill's home when going toward Nick's
home. ]

7. 14 blocks. South [Carol traveled 4 times as fast as Sally. ]

8. 16 miles. East [Betty hikes 5 times as fast as Jane. ]

TC[1-128, 129. 130, 131]


[1-128]

G.
[SUPPLEMENTARY EXERCISES--Part C] [1-129]

(c) Use real numbers to indicate the changes in the ''High'


readings from Monday through Sunday.

(d) Use real numbers to indicate the changes in the "Low"


readings from day to day throughout the week.

The temperature chart of a certain hospital patient showed


these readings.

Morning Noon Evening

Wednesday 99. 6 99.9 103.4


Thursday 99. 1 100. 2 102. 1

Friday 99.9 100. 1 100.9


Saturday 99.0 100. 2 101. 3

Sunday 99.2 99.6 100. 2

(a) Use real numbers to indicate changes in the morning


temperature readings for the successive days.

(b) Use real numbers to measure the changes in the evening


temperatures for the successive days.

(c) Find the average temperature reading for each day.

Bruce was playing a game with Randy. On the first play

Bruce gained 10 points. On successive plays he lost 6,

gained 8, gained 3, lost 7, gained 9, lost 4. What was his

standing after the final play?

(continued on next page)


[1-130] [SUPPLEMENTARY EXERCISES --Part C]

5.
Southeast Northwest

N
A
pump for *^ drinking water

On a hiker's trail, John starts from P which is 21 units north-

west of the spot where pure drinking water may be obtained,


and walks toward the pump. Bob starts at J (which is 11 units

southeast of the pump) at the same time, and also hikes toward
it. They pass each other at N, which is 13 units northwest of

the pump, and continue on their way. However, Bob becomes


thirsty and turns back at P to hike to the pump, and then on
to J. Does Bob overtake John before he arrives at the pump?
Hov/ far from the pump is John when Bob overtakes him?
[The boys hike at steady rates. ]

Nick's home
A
Bill's home

Playground

Swimming Pool

Bill leaves home at 10:00 a.m. to walk to the playground


which is 4 blocks away. His friend Nick leaves his home
at 10:00 a.m. to visit Bill. When Nick arrives at Bill's
home and finds he isn't there, he has a hunch that Bill may
[SUPPLEMENTARY EXERCISES- -Part C] [1-131]

have gone to the playground, so he goes on toward it. Mean-


while, Bill hasn't found anything of interest at the playground,
so he has started on to the swimnning pool, which is 8 blocks
farther. When Nick can't see Bill at the playground, he turns
around and starts home; however, when he gets there he
still wants to talk to Bill, and decides to return to Bill's
honne. But Bill is on his way toward Nick's home, since he
saw none of his friends at the pool. If Bill and Nick have
walked at the same rate (and you disregard any time lost
while they look around at playground and pool), can you dis-
cover how far from Nick's home the two boys finally meet?

7. Ann starts fromhome and bicycles north. At the


Sally's
sanne time, Carol starts from her own home, which is 22
blocks south of Sally's, and is also bicycling north. Ann
travels only 4 blocks when she turns around and starts south.
Carol is 18 blocks north of Sally's home when she turns back;
at that time Ann is midway between the point where Carol
turns back and Carol's home. When Carol gets home and does
not see Ann, she starts north again and travels until she meets
her. How far from Sally's home do they meet? In what direc-
tion from Sally's home is the meeting point?

8. Jane and Betty both start hiking at the same time, and both
travel in an easterly direction. Jane starts at M, where a sign
post pointing west reads '4 miles to Fish Hook' . Betty starts

at Q; the sign board there points east and reads, '8 miles to

Fish Hook' .

Betty passes Jane at the R signboard which points west


and reads '1 miles to Fish Hook' . She continues hiking
east until she reaches a certain apple tree along the road.
She turns here (after hastily picking an apple to munch on the
way homel ) and starts back to Q. On the way she meets Jane
at the W signboard. It points west and reads, * 10 miles to

Fish Hook' .

If both Betty and Jane are hiking at steady rates, what is

the distance of the apple tree from the village of Fish Hook?
What direction is the tree from the village?
[1-132] [SUPPLEMENT..RY EXERCISES--Part D]

D. Simplify.

1. *5 + "5 2. + *10

3. 2i+*2i
^2 ^ ^3

mi. -io| 6. *1 + -.05

7. *1001 101 8. "1, 237. 248 + *1. 237, 248

9, 10. *3t^ + *4i


°^^«tI)^"^iI 16 4

11. *6. 35 + -8^ 12. 'l, 000, 000 + -1, 000, 000

13. "2, 000,000 + *Z, 000, 000 14. -7 + *8

"8 '7 M)
15. + 16. (-1 + + (-1 + *1)

17. -423 + ""398 18. -9. 8 + -7. 6

19. 2 + "3. 16 20. (-9 + "9) f "9

21. ("'28 + *29) + *25 22. *i, 098, 762 + -1

23. "17, 098 + ^7, 097 24. *18, 607, 487 + "18, 607,487

2
25. (*6. 5 + '6. 5) + (*7. 5 + -7. 5) 26. *7 + "3

27. *18. 01 + *1. 6 28. (-3 + "8)+ ^7

-,1 ^ ..7 30.


29. ^2 + ^8
31. -7i+ -9^ 32. \03+ '\

33. ^
34. C9 + "5) + -23
9 "^TT
35. (-8 + ^3) + -12 36. *5 + *0. 02

37. -. 05 + -.28 38. "".


312 + -. 213

39. . 008 + -.082 40. (*5 ^ *2) + (-3 + -4)

41. (-10 + '2) + ("12 3) 42. ("10 + "7) + (*3 + -2)

^*l -1
43. (^12 + -32) + -12 44.

45. "18 + (*10 + -12) 46. ("25 + *b) + (-4 + *9)

47. (-17 + -8) + (-5 + *30) 48. (-97 + -3) + (*25 + -75)

49. (*302 + "201) + (-11 + *10) 50. (*76 + "16) + ("20 + *10)
1/131

- 1
D. 1. 2. no 3.
6
4. ^6
*3
5. 6. ^0.95 7. ^900 8.
8
* 3
9. 10. 11. "2.05 12. "12,000,000
4 ^16
13. 14. n 15. "1 16.

17. "25 18. -17.4 19. "5.16 20. -27

21. *82 22. n,098, 761 23. -1

24. 25. 26. *4 27. n9.6i

28. ~4 29.
- 5
30.
M5 31. -16M
8 48 ^^35

32. "0.22 33. .397 34. -19 35. "17


99
36. "•5.02 37. "0. 33 38. *0.099 39. "0.09

40. 41. -23 42. -16 43. "32

44. 45. "20 46. -15 47.


8
48. -150 49. noo 50, *50

TC[1-132]
[1-133]

i
[1-132]

D. Sim-

1/13Z

-1. "28 2. "28 3. *28 4. *28

5. "44 6. *44 7. ^5 8.

*5
9. n 10. "26 11.
4
12. •4
1 - 165
13.
-»l 14. *23 15.
8
16. no
17. nz 18. "21 19. *36 20. -15

21. "88 22. "108 23. "24 24.

25. 26. *49 27. nz 28. "10

29. 30. 31. *80 32. -72

33. *392 34. *240 35. "240 36. n920


37. *60 38. ~48 39. 40.
4
41. -125 42. *320 43. *240 44. -49

45. "1 46. *25 47. 48. *21

49. *24 50. *36

1. 15 2. 15 3. 9 4. 14

5. 33 6. 35 7. 22 8. 15

9. 360 10. 106 11. 16

TC[1-133]
[SUPPLEMENTARY EXERCISES- -Parts E and F] [1-133]

JE. Simplify.

"4 X '4
1. "7 X ^4 2. ""7 X 3. ^7

4. "7 X -4 5. ^8X"5|- 6. 'SXS^


7.
11
"7^ X'3-^ 8. "8 X 9.
~5
tX "4
I

10. -6xMj 11. no X |- 12. jx*z^

13. ^^jX-Z^- 14. -2^X-9j 15. ^2|-X-8^

16. -"S X *2 17. *3 X ^4 18. "7 X *3

19. '4 X '9 20. *5 X -3 21. -"SX-U

22. "12X^9 23. "4 X *6 24. "3X0


25. 0X^5 26. -7 X -7 27. "4 X "3

28. "5 X ^2 29. X "6 30. *4 X

31. {'5 X "4) X *4 32. ('2 X "9) X "4 33. -7X(^8X"7)

34. (*6X'10)X"4 35. (^6X"10)X''4 36. ("8 X "12) X '20

37. ^3X(-4X"5) 38. ("2 X "3) X "8 39. "10 X ("20 X 0)

X ("5X-5)X"5 X X "2
40. |- ( 7 X n) 41. 42. ("40 *4)

43. *30 X ("2 X -4) 44. ("35 X |-} X *7 45. "4 X ("1 X x)

46. -5X(nOX j) 47. ("10X0)X-8 48. ("7 X "9) X j

49. |x(-8X-12) 50. ("12 X j)X-9

F. Simplify.

1. 1+2X7 1+7X2 2. 3. 15 -3X2


4. 12+4^2 5. 6X5+3 6. 11X4-9
7. 7+8-3-2+12 8. 12X2 + 4 + 1X3 + 6

9. 3X2X(5X12) 10. 3 X 6 X 5 + (1 + 3) X 4

11. 13 X (2 X 4) + 8 + 3 X 5 X 2 + 10

(continued on next page)


[1-134] [SUPPLEMENTARY EXERCISES- -Part G]

12. X TJ X 13. 44 X 2 ^ 8 X 6 = 3 X 4|-


T" I
X ^ X I + X X "8
14. 36 -f ^
i D
+ 48
fa
2 15. ~2 ("8 + ~7)

16. (5+~4)X"3+8 17. -? X "3 X "4 X I


6

18. 8 X "3 + "2 X "3 19. 13 + "26 X ^ + "8


2

20. 10 + "8 + [7 + "3 + '2] + "9 + 12

21. "3 X "8 + 2 22. "7 X 3 + 4 X "5

23. 5 X ~7 + 5 X ~3 24. "2X5+5X6


25. "82 + ("3 + 82) 26. "2 X ("20 + 20)

27. 12 X "3 X "IX '2 28. "3 X + "4 X "1

29. ("18 + "12) X ("5 X 6) 30. (48 -f '16 + "32) X (8 X "9)

G_. Each of the following sentences is an instance of one of the prin-

ciples for the numbers of arithmetic. Tell which principle.

1. 8 X 8 + 2 X 8 = (8 + 2) X 8 2. 1 X (5 + 2) = (5 + 2) X 1

3. (3 + 4) + 5 = 3 + (4 + 5) 4. 73 + = 73

5. 18 + 32 = 32 + 18 6. 392 X 1 = 392

7. 618X0 = 8. 17 X (8 + 3) = 17 X 8 + 17 X 3

9. (16 X 15) X 4 = 16 X (15 X 4) 10. 397X18=18X397


11. 798. 3 + = 798. 3 12. 62 + 13 + 7 = 62 + (13 + 7)

13. 3^+5^
2 4
= 5^+3^
4 2
14. 357.25X0 =

15. . 7361 = . 7361 X 1 16. 14 X 5 X 18 = 14 X (5 X 18)

17. I
o
X ^
I
= I-
I
X I-
b
18. 373. 8 = 373. 8+0

19. 16 X 5^ = (16 X 5) + (16 X ^)

20. 7.23 + . 77 = . 77 + 7. 23 21. | X | X f = | X ( | X f )

22. = 19. 3 X 23. yy X 1 = yy

24.
7 ^ 14 '
42 " 7 ^ i^l4 42
1/133

12 ^ 13. 99 14. 60 15. "240

16. 5 17. 70 18. "18 19.

20. 7 21. 26 22. -41 23. "50

24. 20 25. "3 26. 27. "72

28. 4 29. *900 30.

G. 1. dpma 2. cpm 3. apa 4. paO

5. cpa 6. pml 7. pmO 8. idpma

9. apm 10. cpm 11. paO 12. apa

13. cpa 14. pmO 15. pml 16. apm


17. cpm 18. paO 19. idpma 20. cpa

21. apm 22. pmO 23. pml 24. apa

TC[1-134]
[1-134]
[1-135]
[1-134]

IZ

1/134

25. paO 26. cpa 27. Mpnia 28. apm


29. pmO 30. pml 31. apa 32. apm
33. dpma 34. paO 35. cpm 36. pml
37. apm 38. cpa 39. pmO 40. apa

41. dpiTia 42. cpm 43. pml 44. paO

H. 1. 120 2. 10,000 3. 620 4. 60

5. 8000 6. 70 7. 91 8. 210

9. 10. 11. 108 12. 2562


13. 450 14. 1776 15. 720 16. 540

TC[1-135]
[SUPPLEMENTARY EXERCISES-- Part H] [1-135]

25. 57. 3 = 57. 3 + 26. . 32 + . 73 = . 73 + . 32

27. 35 X 7 + 35 X i = 35X7i
35 X 7^ 28. . 4 x ( . 8 x 1 . 5) = {. 4 X . 8) X 1 . 5

29. 93yX0=:0 30. 475.8 = 475.8x1

/^inl ^^^ cl inl l\ 3


Jl
31. [10y
'y2 + 3x+5-=
4/ 4
10vt-i3-+
2\^4 5x4
r: 1

32. . 92 X . 34 X 5 = . 92 X (. 34 X 5)

33. (. 90 + . 02) X 1. 7 - (. 90 X 1. 7) + (.02 X 1. 7)

34. 27|- + = 27|- 35. 187 X 37 = 37 X 187

36. 4343 X ^--3


1= 43 I- "'-6
37. ^ x ,^^ X
^7
4^
i
J
=
~
I-
^ 73
X I X i

38.
il
+
f
=
I + j| 39. = 1297.8 xo
40. 827 + (73 -\ 769) = (827 + 73) + 769

41. 25t X 12 = 25 X 12 + 4 X 12
4 4
2 3
X 4 2
21 14

9 9
44. +
17 "'
17

H. Simplify. Do as much of the computation as possible without


writing.

1. 2 X {[{5 X 3) + (5 X 2)] + [(5 x 5) + (5 x 2)]}

2. 10 X {[50 X 6] + [50 X 7] + [(50 x 5) + (50 x 2)]}

3. . 5 X {[(62 X 8) + (62 x 4)] + [(62 x 5) + (62 x 3)]}

4. X {[(5 X i) + (i X 7)] + [(340 x i) + (1 x 8)]}


I
5. (8 X 40) X 25 6. 5 X (2 X 7)

7. (5 X 7) + (7 X 8) 8. (15 X 7) f (7 X 15)

9. 125 X 10. 75 X X 9

11. (8 + 75) + 25 12. 752 + (48 + 1, 762)

13. (45 X 5) X 2 14. X (9 X 222)


I
15. (36 X 17) + (36 X 3) 16. (18 x 27) + (3 x 18)

/continued on next page)


'

[1-136] [SUPPLEMENT-VRY EXERCISES--Part I]

17. (51 X 1,763) + (51 X 237) 18. 17j x 12

19. 102% of 35 20. -j^ X 200

23. (972. 75 X 37) + (37 X 27.25) + 490

24. (29 X 51) + (62 x 51) + (9 x 51)

25. J X {[33 X 18] f [18 x 7] + [18 x 60]} ^ 100

r. Simplify.

1. "8 - *3 2. "8 - "3 3. *8 - "3

4. *3 - 5. - *3 6. *8 - *3

7. "6 - *14 8. -7 - *8 9. *9 - "2

10. - '4 11. "6 - "7 12. "13 - "13

13. 2 - "2 14. 7 - "2 15. *9 - *15

16. "17 - -8 17. •"23 - 31 18. 17 - 29

19. 4 - 9 20. 9 - "4 21. "36 - "43

22. 5 - "8 23. "8 - "5 24. 5 - 8

25. 8 - 5 26. - 47 27. 17 - 17

28. 638 - 635 29. "638 - "635 30. 635 - 638

31. 7 - "31 32. "5 - "21 33. *6 - "31

34. 8 - 6 35. 10 - 18 36. °-^


37. "2 - "13 38. *8 - "6 39. *10 - *28

40. -15 -
41. *5 - *5 42. ^6 - M4
43.
*1 M *13 "5
" 44. *Z - 45. *5 -
2 3

46. *16 - -14 47. "3 - ni 48. "2 - *13

49. -5 - *5 50. "10 - *8 51. *3 -

52. *2 - "13 53. -5 - "5 54. "16 - *14

55. "10 - *28 56. ". 3 - *. 02 ".


5 57. *. - 09
[1-137]

*724

'^. 3

1/135

17. 102,000 18. 208 19. 35.7 20. 144

21. 1.7 22. 24.8 23, 37,490 24. 5100

25. 1000

1. -11 2. "5 3. ni 4. *3

5. "3 6. *5 7. -20 8. "15

9. ni 10. *4 11. n 12.

13. 4 14. 5 15. -6 16. -9

17. -54 18. -12 19. "13 20. 13

21. *1 22. *3 23. "3 24. -3

25. 3 26. -47 27. 28. 3

29. "3 30. 3 31. 38 32. n6


- 1
33. *37 34. 2 35. "8 36.
8
37. ni 38. n4 39. -18 40. -15

41. 42. *2 43. 44. "11


6

45. no 46. *30 47.


*'l 48. -15

49. -10 50. 2 51. *3 52. ns


53. 54, "30 55. -38 56. -0.8

57. *0. 11

TC[1-136]
[1-136]

17.
[1-137]

*724

Q. 3

X
[1-136]

17.

1/136

58. n55 59. "55 60. "826 61. *4.8

62. "20.9 63. *36.4 64. -492 65. -46

*19
66. "7.4 67. 68. 69.
2 8 15
101 * 3 * 5
70.
63
71.
56
72.
16
73. *,|

74. "13 75. •4 76. -ao| 77.


M7
15
78. "18.9

J. 1. -17 2. "7 3. n 4. *2

5. "8 6. n3 7. *21 8. -26

9. "3 10. "4 11. 12. -19

13. *8 14. *25 15. *z 16. -3

-10
17. "5 18. -29 19. 20. -1
K. 1. 17 2. -23 3. -2 4. -26

5. 5 6. -4 7. 8. -85

9. -54 10. -39 11. -14 12. -10

TC[1-137]
[SUPPLEMENTARY EXERCISES -
-Parts J and K] [1-137]

58. n03 - "52 59. "92 -


• "37 60. no2 - ^724

61. *8. 5 - *3. 7 62. "9.6 - ni . 3 63. n7. - "19. 3

64. 983 - 1475 65. 683 - 729 66. 75. 3 - 82. 7

67.
*2 n 68.
"3 "7
69.
^3 2
3 6 4 8 5 3

1 '8 *3 *3 -
1
3
70. 71. 72.
7 9 7 8 16 8

73. *6^ - -3^ 74. -4 -•i 75. -4- -1


76. n7^ - "3^ 77. 4 78. -'4' - *6.4

J_. Simplify.

1 "8 + -3 - % 2. "4 - "3 - %


3. *7 - "3 H-
"9 4. *5 - "2 + "5

5. '6 - '7 - "8 + "3 6. no - "3 - *7 •


"7

7. ni + "12 - "13 h "91


8. "4 - "5 - 12 - 15

9. 15-6-3-7-2 10. 19+5-3-2 - 23

11. (*4 - -3) + ("5 + -2) 12. (-7 - ^7) + (-8 - "3)

13. (*7 - -2) - ("3 - -4) 14. (no - -3) - (-15 +-7)

15. (9 - 2) - (8 - 3) 16. (12 - 14) + (7 - 8)

17. (5 - 12) - (9 - 11) 18. (6 - 61) - (17 - 43)

19. ('6. 3 + n. 4) - ("4. 8 + *7. 7) + ("5. 4 + *3. 2)

K. Simplify.

1. 8 - 3 + 12 2. 7 - 18 - 12

3.-5+9-6 4.-3-8-15
5.-6+9+2 6.-5+3-2
7. -12 -3 + 15 8. -61 +11 - 35

9. -10 - 4 - 40 .10. -17 + 3-25


11. 10 - 11 - 13 12. -9-8+7
(continued on next page)
[1-138] [SUPPLEMENTARY EXERCISES -
-Part K]

13. -7+9 + 12-8-5-10 + 15

14. +12+8-9-6-5+7 + 3-8


15. -25 -5-20 + 42+5-6+4

16. -^70-20-7-6 + 3+4-14-27


17. +13+3-5-3+2-5-6
18. -27 -13+32+4-9-3+5

19. -12 -19-3 + 6-2+4 + 12+8

20. -13 -2+9+8+7-10-3+7


21. +19 + 6-11-4+7
3-10-8 +

22. +17-4-3+6-8-12+5
23. -23 - 4 - 3 -i 12 - 2 + 8 + 14 - 2

24. -31 +27-3-8 -. 2 + 14

25. +13+9-5-11-8+7-12
26. +17-8-11+9-12-5 + 7

27. -12 16-5+9-8-11


+7 +

28. +7-8-12 + 15-5-11+9


29. -11-9+6+14+10-12-4
30. -5+9-8 + 7 + 13-11-12
31. +17 + 4-8-9-3-9
32. -23 + 19-3 + 10-5-7
33.

34. +. 5
^444
-i. 1-1,31-51
+
2

. 3 - . 8 - . 7 - . 2 + 1. 2

35. -7I + 2I + ll - sl - 3|
2 4 2 4 4

36. +. 12 + . 11 - . 10 - . 02 + . 03 - . 13 - . 01

37. -6.3 + 1.4+4.8-7.7 + 3.2-5.4


[1-139]

1/137

13. 6 14. 2 15. -5 16. 3

17. -1 18. -11 19. -6 20. 3

21. 2 22. 1 23. 24. 1

25. -2 26. -3 27. -4 28. -5

29. -6 30. -7 31. -8 32. -9

33. -4 34. .3 35. -15 3 36.


4
37. -10

TC[1-138]
[1-138]
[1-139]
[1-138]

13.

1/138

L. 1. -12 2. 14 3. -25 4. -20

5. 54 6. -39 7. -11 8. -13

9. -4 10. 60 11. 12. -18

13. -24 14. 14 15. 27 16. -56

17. -10 18. -3 19. -11 20. -55

21. 57 22. 44 23. -27 24. -25

25. 17 26. 27. 14 28. -28

29. 69 30. 32 31. 166 32. 320

33. -12 34. 2500

TC[1-139]
[SUPPLEMENTARY EXERCISES- -Part L] [1-139]

L^. Simplify.

1. -3X(5-1) 2. -7X(8-10) 3. -5X{7-2)


4. 5 X (4 - 8) 5. 6X(7+2) 6. -3X(8--5)
7. 4 + (9 - 12) X 5 8. 5 + 3 X (15 - 21)

9. 6 - 2 X {10 - 5) 10. 11 - 7 X (2 - 9)

11. -9-3X(8-ll) 12. -12-3X(4-2)


13. (?-8)X(7-3) 14. (12-5)X(7-5)
15. (6 -,'•
3) X{6 - 3) 16. (5-9)X(5-f9)
17. -3 +(7 - 5) + (6 - 8) + (2 - 5) + (7 - 11)

18. -9 - (4 - 7) + (12 - 3) - (6 - 8) - (15 - 7)

19. -5 + 2 X (8 - 3) + 5 X (6 - 2) + 4 X (8 - 17)

20. -11 - 3 X (4 - 1) - 2 X (9 - 8) - 3 X (9 + 2)

21. (9 - 3) X (7 - 5) + (6 - 1) X (11 - 2)

22. (5 - 7) X (11 - 3) + (12 - 2) X (15 - 9)

23. (6 + 4) X (7 - 2) - (8 - 15) X (9 - 20)

24. (12 - 7) X (9 - 18) - (8 + 2) X (15 - 17)

25. [5 + 3 X (8 -
4)] X [9 - 4 X (8 -
6)]

26. [-6 - 2 X (7 - 10)] X [21 + 5 X (6 - 10)]

27. 11 - [6 + 2 X {7 - 3)] + [5 - 3 X (6 - 10)]

28. -19 + [-3 - 9 X (11 - 12)] - [-15 - 5 X (2 - 8)]

29. 15 - 2 X [3 + 4 X (7 - 2)] - 5 X [8 - 2 X (5 -J-


9)]

30. -83 - 2 X [-4 - 7 X (9 - 2)] - 3 X [-2 - (9 - 8)]

31. 100 - 6 X {3 - [5 - 2 X (7 -
3)] + [-2 - 3 X (7 -
2)]}

32. [17 - (8 - 3) X (4 -
7)] X [18 - (7 - 9) X (8 - 12)]

33. [51 - (6 -i- 5) X (7 -


2)] X [-17 - (5 - 9) X (8 -
3)]

34. -26 X (57 - 82) + -31 X (57 - 82) - 43 X (57 - 82)


[1-140] [SUPPLEMENTARY EXERCISES- -Parts M and N]

M. Simplify.

1. *7 -r *Z 2. "18 T "3 3. "21 -= 7


4. *42 •=- -7 5. *8 T "6 6. "8 T "6

7. "6 ^*8 8. "6 V "8 9. +10 V "20

10. '20 ^ *10 11. "20 V -10 12. + 10 V *\5

13. *15 V -3 14. "15 v*3 15. *16 ^ "32

16. *Zb T "13 17. "34 V '17 18. ^27 V "54

19. '.08 ^ *4 20. -. 106 ^ ". 3 21. ^ 873 V ".09

'
22. "1,000,005 ^ *15 23. '9873234 ^ 1234 24. *225 -
r "15

25. "196 ^ ^14 26. *169 ^ "13 27. "256 V -16

28. *289 ^ "17 29. *324 T +18 30. "361 V "19

*72 "54 -38


31. "8 32.
*2
33.
"19 34. 98/14

35. 132/12 36. "108/9 37. 56/-8 38. "84/12


128 "147 "156
39. -91/"13 40. -4 41. -3 42.
4

999.999 "98762 63
43. -33 44. "23
-
45. 6Z5/-25 46. -9

-171 -112
47. 48. - O

N. Write a comparison sentence for each of the pairs of nunnbers listed.

1. (*5, *3) 2. {*7, "4) 3. (-5, *4)

4. (-7, -12) 5. ("20, "15) 6. (*4, M 7)

7. ("17, -16) 8. (-16, -17) 9. ("16, ^6)

10. (*16, -16) 11. (*16, -17) 12. ("4, "4.1)

13. (*5 r "7, -5 i *7) 14. (-2 \ "3, 2 x 3)

15. (2 + i
3^, 7 - i
i) 16. (9 - 17, 17 -
9)
2' 2

17. (5 X (3 - 5), -5 X (5 -
3)) 18. (-3 - 1^, 5 -
9^)
4 4
[1-141]

1/139

*7
M. 1.
2
2. % 3. "3 4. "6

~4 *4 "3 *3
5. 6. 7. 8.
3 3 4 4
"1 *2
9. 10. "2 11. *i 12.
2 3
"1
13. "5 14. "5 15. 16. "2
2
'1 * 53
17. *2 18. 19. "0.02 20.
2 150
21. 9.7 22. "66,667 23. "8,001 24. "15

25. "14 26. "13 27. n6 28, "17

29. na 30. M9 31. "9 32. "27

33. *2 34. 7 35. 11 36. "12

37. "7 38. "7 39. *7 40. "32

41. *49 42. "39 43. "30,303 44. *4,294

45. "25 46. "7 47. "19 48. M4

N. [There is more than one way in which these comparison sentences


could be written. We have listed several, but not all, possibili-
ties for each exercise. ]

1. *5 > "3. ^5 ^ *3, *5 / *3, *5 > *3

2. *7 > *3, *7 ;^ *3, ^7 / *3, *7 > *3

3. "5 < *4, "5 ;»^ *4, "5 / *4, "5 < *4

4. "7 > "12, "7 /^ "12, "7 / "12, "7 > "12

5. "20 < "15, "20 i-


"15, "20 / "15, "20 < "15

TC[l-140]a
[1-140]

M. Si'
[1-141]

1/140

6. *4<*n, M;^n7, M/ n7, ^4<n7


7. -17 < "16, "17 ;^ "16, "17 f^ -lb, "17 < "16

8. -16 > "17, "16 /. -17. "16 / "17, "16 > "17

9. -16 < n6, -16 ^ 16, "16 / n6, "16 < 16


10. 16 > -16, n6 ft -16, 16 / -16, 16 > "16

11. 16 > -17, 16 / -17, 16 / -17, 16 > -17

12. -4 > -4. 1, -4 / "4. 1, "4 / -4. 1, -4 > "4. 1

13. "5 + "7 < -5 + ^, -"S + -7 ;^ -5 + "7, *5 + "7 / -5 + ^.


*5 + -7 < -5 + ^
14. -2 X -3 = 2 X 3, "2 X -3 / 2 X 3, "2 X "3 ;>^ 2 X 3,
"2 X -3 > 2X3

15. 2 + 3|- < 7


I' 2 + 4?^ "^-^ 2 + 3^/7-1-,
2

2 + 4^^- r
16. 9 - 17 < 17 - 9, 9 - 17 ;^ 17 - 9, 9-17/17-9,
9 - 17 < 17 - 9

17. 5 X (3 - 5) = -5 X (5 - 3), 5 X {3 - 5) / -5 X (5 - 3),

5 X (3 - 5) / -5 X (5 - 3), 5 X (3 - 5) < -5 X (5 - 3)

18. -3 -
1^ > 5 9^
^4 ' l^J^S -9|, -3- li/ 5 '
4
1 3
> 9x
4 —
J
1 5 -
4

TC[l-l40]b
[1-140]

M. Si
[1-141]

X
1/141

5. > 6. > 7. > 8. <

9. > 10. < 11. < 12. <

13. < 14. < 15. F 16. F


17. T 18. T 19. F 20. T

21. T 22. T 23. T 24. F


25. F 26. T 27. T 28. F
29. T 30. F 31. F 32. T
33. T 34. T 35. F 36. T
37. F 38. F 39. F 40. T
41. T 42. F 43. F 44. T
45. 46.

TC[1-141]
[SUPPLEMENTARY EXERCISES- -Part O] [1-141]

O. Insert an ' > ', an '


< '
or an ' = '
to get a true statement.

1
^-
-L-
0.9
-1-
0.8
2 • 7^
10
4i
5

3. ^ 4 4. -8 -8
6 /

5. 7 -9 6. 148 -14

'•
373 377

^8 ^4

9. 342 "342 10. -^ - yj

11. "1,999,999 "1,999,997


20 _ 20
12. -I?-
31
-1$ 32
13. .082 .820

14. "1.001 '1.0001

True or false?

-99 > -97 16. 8 > 8. 17. =-4.25


15. 1
V
18. -1.2 < 19. 7/8 20. -10 ^

21, -8 / -8 22. -8 ?^ 8 23. 8 ^' 8

24. 8/^-8 25. (-7 4 7) /O 26. •0->/|5


27. 1 < . 6001 28. -\ > - 142 29. •°«^«^ 2500
30. -5 > -2 31. 5 -^ 2 32. 5 /-5
33. -5 /-5 34. Z.2, Y 2.4 35 - 1

3.8
>
- 5.4
i

1
36. 37. -13 = 13 38. -. 0008 .008
200 - 2000 f^

_ . 198 ^ . 196
39. 40. 33j > 33,33 41. -. 082 < -.0082
2

10000 ^ 9999
42. \ 091 > *. 901 43.
-i>4 44.
29786' '
29787

45.
. 084 ^ . 087
. 3
« .is i .09
1
0/
»/
MUM
WM^ :mu>mmii>iktiiiiiMiMmtir

You might also like